You are on page 1of 127

50+ Bank PO | Clerk Previous Year’s Papers 2016 – 2020

1 Adda247 Publications For any detail, mail us at


Publications@adda247.com
50+ Bank PO | Clerk Previous Year’s Papers 2016 – 2020

Mock IBPS RRB PO Prelims 2019


31
REASONING ABILITY

Direction (1-5): Study the following information carefully 6. Statements: Only a few Palace is Home. All Home is
and answer the questions given below: Office. No Office is Building.
Conclusion
Nine persons i.e. P, Q, R, S, T, U, V, W, X were born on
I: All Palace is Home is a possibility.
different months i.e. January, March, April, May, July,
II: Some Palace is Building.
August, September, October, November but not necessarily
in same order. 7. Statements: All Men is Women. Some Child is
Women. No Men is Boy.
Four persons were born between P and T. P was born Conclusion
before T. Q was born in the month of 30 days after July. T I: Some Men is Child.
was born after Q and before R. There were as many persons II: No Men is Child.
born before X as after R. one person was born between U
and V. S was born before U and after W. 8. Statements: No Professor is Student. Only a few
Student is Lecturer. All Lecturer is Principal.
1. How many persons were born between X and V? Conclusion
(a) Two (b) Three (c) One I: All Professor is Principal is a possibility.
(d) Four (e) More than four II: All Student is Lecturer is a possibility.
2. Who among the following was born on August? 9. Statements: Only a few Palace is Home. All Home is
(a) R (b) S (c) T Office. No Office is Building.
(d) P (e) None of these Conclusion
3. In which of the following month S was born? I: Some Home is Building.
(a) March (b) April (c) June II: No Home is Building.
(d) October (e) None of these 10. Statements: No Professor is Student. Only a few
4. If W is related to April, V is related to July then, P is Student is Lecturer. All Lecturer is Principal.
Conclusion
related to which of the following?
I: Some Student is Principal.
(a) March (b) May (c) June
II: Some Lecturer is Professor.
(d) August (e) October
Direction (11-15): Study the following information
5. Four of the following five are alike in certain way
carefully and answer the questions given below:
based from a group, find the one which does not
belong to that group? Eight persons are sitting around a square table. Four
(a) R (b) S (c) T persons are sitting at middle of the sides of the square and
(d) U (e) V all are facing towards inside. Remaining four are sitting at
corners and they face outside.
Direction (6-10): In each of the questions below are given
some statements followed by two conclusions. You have to Two persons are sitting between P and U. R who is an
take the given statements to be true even if they seem to be immediate neighbor of P, sits opposite to S. T sits 3rd to the
at variance with commonly known facts. Read all the right of V. W sits immediate right of T. Q faces W.
conclusions and then decide which of the given conclusions 11. Who among the following sits opposite to T?
logically follows from the given statements disregarding (a) P (b) R (c) S
commonly known facts. (d) W (e) None of these
(a) If only conclusion I follows.
(b) If only conclusion II follows. 12. How many persons are sitting between P and V when
(c) If either conclusion I or II follows. counted from left of P?
(d) If neither conclusion I nor II follows. (a) Two (b) Three (c) Four
(e) If both conclusions I and II follow. (d) Either (a) or (c) (e) None of these

2 Adda247 Publications For any detail, mail us at


Publications@adda247.com
50+ Bank PO | Clerk Previous Year’s Papers 2016 – 2020

13. What is the position of Q with respect to R? F are seated (but not necessarily in the same order) and all
(a) Immediate right of them are facing North. Therefore, in the given seating
(b) Immediate left arrangement each member seated in a row faces another
(c) 2nd to the right member of the other row. P faces D. U does not face A, who
(d) 2nd to the left sits left to E but not immediate left. R sit at one of the ends
(e) None of these and diagonally opposite to B. Three persons sit between B
and F, who does not face U. C sits immediate left to D but
14. Who among the following person sit 3rd to the right of
does not faces S. Two persons sit between Q and U, none of
Q?
them sits at the end. The one who faces T sits 2nd right to A.
(a) P (b) U (c) R
(d) S (e) None of these 21. Who among the following faces A?
(a) S (b) T (c) Q
15. Four of the following five are alike in certain way
based from a group, find the one which does not (d) R (e) none of these
belong to that group? 22. How many persons sit to the right of R?
(a) Q (b) R (c) S (a) No One (b) one (c) two
(d) T (e) U (d) three (e) four
Direction (16-17): Study the following information 23. Four of the following five form a group ,who among
carefully and answer the questions given below: the following does not belongs to that group?
Eight members are living in a family. Q is the only son of P. (a) U (b) T (c) E
T is wife of U. T is sister of Q and R. V is daughter in law of (d) F (e) A
W. S is son of T. W is the mother of Q. 24. If in a certain way R is related to C, T is related to E,
16. How is S related to R? then who among the following is related to D?
(a) Son (b) Daughter (c) Nephew (a) U (b) T (c) E
(d) Niece (e) Can’t be determined (d) F (e) Q
17. How many male members are in the family? 25. Who among the following sit 3rd right to U?
(a) Four (b) Five (c) Three (a) R (b) T (c) P
(d) Six (e) None of these (d) S (e) Q
18. How many such numerals are there in the number Directions (26-30): Study the following information
‘254136987’ which will remain at the Same position carefully and answer the questions given below:
when arranged in ascending order from left to right? In a certain code language
(a) one (b) two (c) three ‘left right centre’ is written as ‘yo vo na’,
(d) four (e) None of these ‘ahead below behind’ is written as ‘sa ra la’,
19. How many pairs of letters are there in the word ‘above centre right’ is written as ‘ha vo na’, and
‘EDUCATION’, each of which have as many letters ‘behind below above’ is written as ‘ha ra la’.
between then in the word as they have between them
26. What is the code for ‘left’?
in the English alphabet?
(a) sa (b) ha (c) yo
(a) one (b) two (c) three
(d) four (e) more than four (d) na (e) None of these

20. If four letter word is formed from 1st, 3rd, 5th and 6th 27. ‘behind’ will be written as?
letter of TRANSLATE then what is the 3rd letter of (a) ra (b) ha (c) la
newly formed word? If more than one meaningful (d) Either (a) or (c) (e) None of these
word is formed, then the answer will be Z. 28. What is the code for ‘ahead’?
(a) L (b) T (c) A (a) sa (b) yo (c) la
(d) S (e) Z (d) ha (e) Can’t be determined
Directions (21-25): Read the following information 29. What does ‘ha’ stand for?
carefully and answer the questions given below: (a) behind (b) below (c) ahead
Twelve people are sitting in two parallel rows containing (d) above (e) None of these
six people each in such a way that there is an equal
30. What is the code for ‘centre’?
distance between adjacent persons. In row 1 – P, Q, R, S, T
(a) la (b) yo (c) sa
and U are seated (but not necessarily in the same order)
(d) ha (e)Can’t be determined
and all of them are facing south. In row 2 – A, B, C, D, E and
3 Adda247 Publications For any detail, mail us at
Publications@adda247.com
50+ Bank PO | Clerk Previous Year’s Papers 2016 – 2020

Directions (31-35): Study the following information and 34. Number of boxes above K is one less than the number
answer the questions given below: of boxes below ________?
(a) S (b) R (c) F
There are eleven boxes placed one above the other. Five
(d) D (e) None of these
boxes are placed between F and T. Not more than five
boxes are kept above T. Two boxes are kept between T and 35. How many boxes are there between M and H?
M. Three boxes are kept between M and S and M is kept at (a) One (b) Two (c) Three
one of the positions above S. There are only three boxes (d) None (e) More than three
kept above the box J. One box is kept between R and S. Two Directions (36-40): In each of the question, relationships
boxes are kept between R and H. Box D is kept at one of the between some elements are shown in the statements.
positions below box K and at one of the positions above box These statements are followed by conclusions numbered I
C which is not above R. Box E is kept immediately above K. and II. Read the statements and give the answer.
31. How many boxes are placed between J and R? (a) If only conclusion I follows.
(a) 5 (b) 6 (c) 3 (b) If only conclusion II follows.
(d) 4 (e) None of these (c) If either conclusion I or II follows.
(d) If neither conclusion I nor II follows.
32. Which of the following statement is true regarding C? (e) If both conclusions I and II follow.
(a) C is placed at one of the positions above D
36. Statements: C ≤ L = E ≤ R ≤ K = P ≥ O
(b) C is placed immediately below F.
Conclusions: I. P = C II. C < P
(c) R is placed just above C
(d) C is placed at the bottom most position 37. Statements: W > A = S ≥ H < I ≤ N ≤ G
(e) None of these Conclusions: I. H < W II. G > H
33. Which of the following is not true regarding J? 38. Statements: C < O ≤ D = S > A ≥ P ≥ Q
(a) J is immediately below box T Conclusions: I. Q < D II. C < A
(b) One of the boxes below J is D 39. Statements: F ≤ B = I ≤ C = A ≥ S > E
(c) Number of boxes between J and S is four Conclusions: I. S ≥ B II. F > E
(d) One of the boxes above J is K
(e) One box is kept between J and M 40. Statements: I ≥ N = T ≥ E > L ≥ G > M
Conclusions: I. G < N II. I ≥ L

Quantitative Aptitude

Directions (41-45): Study the table given below and 43. If total no. of employee in E is 25% more than D and
answer the following Question no. of employee in HR dept is same as in company C,
then employee other than HR dept in company E is
% of what % of other dept employee in company B.
Total Employee in
Company Female in (a) 60% (b) 80% (c) 75%
employee HR dept
HR dept (d) 50% (e) 55%
A 300 80 75 44. Find the difference between males of HR dept in
B 250 50 80 company C and D together and females of HR dept in
C 400 100 60 company B and C together ?
D 200 60 60 (a) 36 (b) 42 (c) 48
(d) 40 (e) 30
41. Find the average no. of Females in HR department
45. Find the average no. of employee other than HR dept.
together ? in A, B and C together ?
(a) 54 (b) 46 (c) 49 (a) 280 (b) 270 (c) 220
(d) 50 (e) 52 (d) 300 (e) 240
42. Females in the HR dept of company C is what % more 46. If there are total 150 females in company C then how
than male in HR department of company A ? many female employees are there other than females
(a) 250% (b) 200% (c) 100% of HR department
(a) 90 (b) 100 (c) 80
(d) 300% (e) 150%
(d) 110 (e) 120

4 Adda247 Publications For any detail, mail us at


Publications@adda247.com
50+ Bank PO | Clerk Previous Year’s Papers 2016 – 2020

Directions (47-51): Find the missing term in the following 54. If ratio of ages of P and Q before 4 year ago is 5 : 4 and
number series: after 12 years sum of their ages will be 68 years, their
what was P’s age 2 years ago ?
47. 1864, 1521, 1305, ? , 1116, 1089
(a) 24 years (b) 22 years (c) 18 years
(a) 1160 (b) 1180 (c) 1095
(d) 26 years (e) 20 years
(d) 1205 (e) 1220
55. If Pipes A and B can fill a tank in 15 min and 20 mins
48. 18, ?, 9, 18, 72, 576
respectively and pipe C empties the tank in 12 mins.
(a) 12 (b) 9 (c) 18
what will be the time taken by A, B and C together to
(d) 10 (e) 6
fill the tank completely?
49. 12, 6.5, 7.5, 12.75, 27.5, ? (a) 25 min (b) 30 min (c) 40 min
(a) 66.5 (b) 68.75 (c) 63.75 (d) 20 min (e)35 min
(d) 71.25 (e) None of these
Directions (56-60): Solve the given quadratic equations
50. 5 , 15, 50, ?, 1030, 6185 and mark the correct option based on your answer—
(a) 210 (b) 205 (c) 225 (a) x > y
(d) 200 (e) 195 (b) x < y
51. 130, 154, 186 , ? , 274, 330 (c) x ≥ y
(d) x ≤ y
(a) 216 (b) 220 (c) 240
(e)x = y or there is no relationship
(d) 226 (e) 230
52. If a boat travels 18 km more in downstream than in 56. (i) x² = 81 (ii) y² – 18y + 81 = 0
upstream in 3 hr. and if the speed of the Boat in still 57. (i) 4x² - 24x + 32 = 0 (ii) y² - 8y + 15 = 0
water is 20 km/hr. find the distance travelled by boat
58. (i) x² - 21x + 108 = 0 (ii) y² – 17y + 72 = 0
in downstream in 4 hr. ?
(a) 86 (b) 92 (c) 68 59. (i) x² – 11x + 30 = 0 (ii) y² - 15y + 56 = 0
(d) 96 (e) None of these
60. (i) x³ = 512 (ii) y² = 64
53. If A invested Rs. 12000 at some rate of interest of S.I
61. If a shopkeeper marks an item 50% above its CP and
and B joined him after 3 months investing 16000 at if 12% discount is given on the marked price and the
same rate of interest if A leaves before 2 month of shopkeeper makes profit of 256 Rs, then what will be
completion, then what will be the share of B’s profit the actual cost price of the item?
after 1 year if total profit is 22000 Rs. ? (a) 1000 Rs. (b) 800 Rs. (c) 750 Rs.
(a) 10000 (b) 14000 (c) 12000 (d) 1200 Rs. (e) 900 Rs.
(d) 8000 (e) 11000

Directions (62-67):The line graph shows the data of five seller selling an item(in units) on Monday and Tuesday.
450

400

350

300

250

200

150
A B C D E

Tuesday Monday

5 Adda247 Publications For any detail, mail us at


Publications@adda247.com
50+ Bank PO | Clerk Previous Year’s Papers 2016 – 2020

62. The no. of item sold by A and C together is how much 70. Total students in art stream in A is what percent more
more or less then items sold by B and D together on than total students in science stream in B?
both days ? (a) 75% (b) 70% (c) 90%
(a) 250 (b) 280 (c) 300 (d) 100% (e) 110%
(d) 320 (e) 350
71. Find the ratio of total students in commerce stream in
63. What is the average no. of items sold by all five sellers B to total students in science stream in A?
on Monday ? (a) 8 : 15 (b) 8 : 17 (c) 8 : 13
(a) 298 (b) 305 (c) 280 (d) 8 : 11 (e) 8 : 9
(d) 300 (e) 315
72. If in school C total students are720 students and total
64. Items sold by B and C on Tuesday together is what % students in science stream of school Care 25% more
more than same sellers on Monday together ? than total students in commerce stream in school B,
(a) 25% (b) 30% (c) 20% then find total students ofart & commerce stream in
(d) 15% (e) 24% school C is how much less than total students in art
and commerce stream in school A?
65. Find the difference between items sold by B, D, E on
(a) 120 (b) 110 (c) 150
Monday together items sold by B and E on Tuesday
(d) 100 (e) 140
together
(a) 150 (b) 180 (c) 160
73. Find the average number of students in science
(d) 120 (e) 200
stream in school A & B?
66. Item sold On Monday by C and E together is (a) 250 (b) 270 (c) 240
approximately what percentage of total items sold by (d) 200 (e) 225
A and B together on Tuesday ?
(a) 71% (b) 80% (c) 55% 74. If out of total students in art stream of school A & B
(d) 85% (e)65% ratio of boys to girl is 5 : 3 and 7 : 4 respectively, then
find difference between boys and girls in art stream of
67. Find the difference between the average items sold by
school A & Btogether?
A and B together on Monday and average of items sold
(a) 220 (b) 225 (c) 240
by B and C together on Tuesday?
(d) 248 (e) 224
(a) 45 (b) 35 (c) 25
(d) 40 (e) 50 75. P invested 60% more than Q and R invested 20%
more than Q. If ratio of investment time-period (P: Q:
68. If A start from P with speed 60 km/hr at 8:00 am and
R) is 2: 4: 3 and the sum of profit shares of Q and R is
B starts with speed 70 km/hr. at 8 : 30 am from Q and
Rs. 8550 then find the profit share of P.
total distance between P and Q is 680 km, find at what
time they will cross each other? (a) Rs. 3200 (b) Rs. 4000 (c) Rs. 2400
(d) Rs. 3600 (e) Rs. 3000
(a) 2 : 30pm (b) 1 : 30pm (c) 12 : 30pm
(d) 3 : 00pm (e) 4 : 00pm 76. When a person sold an article, his profit percent is
60% of the selling price. If the cost price is increased
69. If a person invested 6000 at T% S.I for 3 year and
by 75% and the selling price remains the same, then
same amount at (T + 5)% CI for 2 year and difference
find decrement in the profit is what percent of the
between both interest is 60 Rs. then find T ?(in %)
selling price of the article?
(a) 15 (b) 18 (c) 20
(a) 25% (b) 30% (c) 40%
(d) 24 (e) 25
(d) 27.5% (e) None of these
Direction (70−𝟕𝟒): Read the data carefully and answer
77. Area of Istcircle and circumference of IInd circle is
the question.
1386 cm2 and 176 cm respectively. There is a square
There are 1800 students in two school ‘A’ & ‘B’ and three 5
whose side is 35 7 % of twice of sum of the radius of
3
streams in each school i.e. art, science & commerce.18 % both the circles. Find the perimeter of the square (in
4
of total students in school A arein commerce stream and 28 cm)?
4
% of total students in school B arein science stream. Sum (a) 132 (b) 136 (c) 140
7
of total students in commerce stream in A & science stream (d) 116 (e) 124
1
in B is 420. 19 21 % of total students in school B are in 78. There are 5 red, 6 black and 5 blue balls in a bag. Out
commerce stream and 50% of total students in school A are of these balls, four balls are picked at random from the
in Art stream. bag. Then, what is the probability that one is red, two
are black and one is blue ball?
6 Adda247 Publications For any detail, mail us at
Publications@adda247.com
50+ Bank PO | Clerk Previous Year’s Papers 2016 – 2020
75 75 71 1
(a) (b) (c) (d) 63 3 % (e) 60 %
362 364 362
70 5 80. A train travelling at 72 km/hr. classes a platform of
(d) (e)
363 26
160 m in 18 second and another train travelling at 90
2
79. An article is marked 66 3 % above the cost price and km/hr crosses the same platform in 15 second. Find
loss incurred on selling that article is 25% of the the length of another train?
discount given on it. Then, find the discount % given? (a) 160 m (b) 180 m (c) 140 m
1 1 1 (d) 200 m (e) 215 m
(a) 48 3 % (b) 53 3 % (c) 58 3 %

Solutions

REASONING ABILITY

Direction (1-5): Direction (11-15):


Months Persons
January X
March W
April P
May V
July S
August U
September Q 11. (e); 12. (b); 13. (b);
October T
November R 14. (d); 15. (a);
1. (a); 2. (e); 3. (e); Direction (16-17):
4. (b); 5. (a);
Direction (6-10):
6. (d);

16. (c); 17. (a);


18. (b);
7. (c);

19. (e);
8. (a);

20. (e);
9. (b);
Directions (21-25):

10. (a);
21. (d); 22. (a); 23. (e);
24. (e); 25. (e);

7 Adda247 Publications For any detail, mail us at


Publications@adda247.com
50+ Bank PO | Clerk Previous Year’s Papers 2016 – 2020

Directions (26-30): H
M
Word Code D
Right/centre vo/na R
F
Left yo
S
Below/behind ra/la
C
Ahead sa
31. (c); 32. (d); 33. (c);
above ha
34. (c); 35. (d);
26. (c); 27. (d); 28. (a);
Direction (36-40):
29. (d); 30. (e);
36. (c); I. P = C (False) II. C < P (False)
Directions (31-35):
Boxes 37. (e); I. H < W (True) II. G > H (True)
E 38. (a); I. Q < D (True) II. C < A (False)
K
39. (d); I. S ≥ B (False) II. F > E (False)
T
J 40. (a); I. G < N (True) II. I ≥ L (False)

Quantitative Aptitude

41. (c); Average no. of females in HR dept therefore females other than in HR department
75 80 60 60
80× +50× +100× +60× = 150-60= 90
= 100 100
4
100 100

60+40+60+36 196 47. (b);


= = = 49
4 4
60
42. (b); Females in company C (HR) = 100 × = 60
100
25
Males in company A (HR) = 80 × 100 = 20
Difference = 60 – 20 = 40 48. (b); 18, ?, 9, 18, 72, 576
40
∴ % = × 100 = 200% more 18 × 0.5 = 9
20 9×1=9
125 9 × 2 = 18
43. (c); Total employee in E = 200 × 100 = 250
18 × 4 = 72
∴ employee of HR dept in E = 100
72 × 8 = 576
∴ other employee = 150
100
∴% of other employee = 150 × 200 = 75% 49. (d); 12 × 0.5 + 0.5 = 6.5
6.5 × 1 + 1 = 7.5
44. (a); Males in HR dept in C and D 7.5 × 1.5 + 1.5 = 12.75
40 40
= 100 × 100 + 60 × 100 = 40 + 24 = 64 12.75 × 2 + 2 = 27.5
Females in HR dept of B and C = 50 ×
80
+ 27.5 × 2.5 + 2.5 = 71.25
100
60
100 × 100 = 100 50. (b); 5 × 2 + 5 = 15
15 × 3 + 5 = 50
∴ Difference = 100 – 64 = 36
50 × 4 + 5 = 205
45. (e); Average of A, B, C =
220+200+300
=
720
= 240 205 × 5 + 5 = 1030
3 3 1030 × 6 + 5 = 6185
46. (a); Total females in company C = 150 51. (d);
females in HR department in company C
60
= 100×100= 60

8 Adda247 Publications For any detail, mail us at


Publications@adda247.com
50+ Bank PO | Clerk Previous Year’s Papers 2016 – 2020

52. (b); (Ds –Du ) 3 = 18 km 59. (b); x² – 11x + 30 = 0


Different in 1 hr. = 6km x² – 6x – 5x + 30 = 0
Ds and Du ∴ x(x – 6) – 5(x – 6) = 0
∴ Speed of boat in still water = 20 km/hr. x = 6, 5
Ds = 23 km/hr., Du = 17 km/hr. y² – 15y + 56 = 0
Distance travelled = 4 × 23 = 92 km y² – 7y – 8y + 56 = 0
53. (c); y (y – 7) – 8 (y – 7) = 0
∴ y = 7, 8
∴x<y
60. (c); x³ = 512
3
x = √512 = 8
y² = 64
6
∴ B’s share = 22000 × 11 = 12000 y = √64 = ± 8
∴x≥y
54. (b); P Q
–4 5 4 61. (b); Let CP = 100 x
+12 P + Q = 68 ∴ marked price = 150x
Age increased in 16 year = 32 years ∴ selling price after giving discount = 132x
Sum of Age of P and Q before 4 years = 36 ∴ 32x = 256
∴ 5x + 4x = 36 x=8
X=4 ∴ CP = Rs 800
P’s age 2 years ago = 5x + 2 = 22 years
62. (b); Item sold by A and C = 550 + 570 = 1120
55. (b); Item sold by B and D = 750 + 650 = 1400
∴ diff. =1400 – 1120 = 280
300+350+250+380+210 1490
63. (a); Average = = = 298
5 5

64. (c); Item sold by B and C on Monday = 350 + 250


∴ tank filled in 1 min = 2 units = 600
60
Total time = = 30 minutes Item sold by B and C on Tuesday = 400 + 320
2
= 720
56. (d); x² = 81 ∴ % increase = 120 ×
100
= 20%
x=±9 600
Y² – 18y + 81 = 0 65. (d); Items sold on Monday by B, D and E
(y – 9)² = 0 = 350 + 380 + 210 = 940
∴ y = 9, 9 Item sold on Tuesday by B and E = 400 + 420
∴x≤y = 820
57. (e); 4x² – 24x + 30 = 0 ∴ diff = 940 – 820 = 120
4x² – 16x – 8x + 32 = 0 66. (a); Item sold by C and E on Monday = 250 + 210
4x (x – 4) –8 (x–4) = 0 = 460
x = 4, 2 Item sold by A and B together on Tuesday
y² – 8y + 15 = 0 = 400 + 250 = 650
y² – 5y – 3y + 15 = 0 100
y(y – 5)–3 (y – 5) = 0 ∴ ? = 460 ×
650
∴ y = 5, 3 ≃ 71% (approx)
∴ No relation exists 650
67. (b); Avg. by A and B on Monday = 2
= 325
58. (c); x² – 21x + 108 = 0 720
x² – 9x – 12x + 108 = 0 Avg. of B and C on Tuesday = 2
= 360
x(x – 9) – 12 (x – 9) = 0 Diff. = 360 – 325 = 35
x = 9, 12 68. (b);
y² – 17y + 72 = 0
∴ y² – 8y – 9y + 72 = 0
y (y – 8) – 9 (y – 8) = 0
∴ y = 8, 9
∴x≥y
9 Adda247 Publications For any detail, mail us at
Publications@adda247.com
50+ Bank PO | Clerk Previous Year’s Papers 2016 – 2020
1 74. (c); Total boys in art stream of school A & Btogether
Dist travelled by A in 2 hr = 30 km
5 7
Remaining distance to be covered = 680 – 30 = 480 × 8 + 440 × 11 = 300 + 280 = 580
= 650 km Total girls in art stream of school A & B together
3 4
Relative speed = 60 + 70 = 130 = 480 × 8 + 440 × 11 = 180 + 160 = 340
650
∴ time taken = 130 = 5 hr Required difference = 580 −340 = 240
∴ time = 8 : 30 + 5 hr = 1 : 30 pm 75. (d); Let the investment of Q = 100x
69. (a); By going with the options Investment of P = 160x
6000×3×15 Investment of R = 120x
Interest received at SI = = 2700 Rs
100 Ratio of profit:
∴ T + 5 = 20% P Q R
6000×44
Interest received after 2 yrs at CI = 160x × 2 100x × 4 120x × 3
100
= 2640 8 : 10 : 9
ATQ,
∴ Difference = 2700 – 2640 = 60 Rs 19 unit = Rs. 8550
T=15% 8 unit = 450 × 8 = Rs. 3600
S (70−𝟕𝟒): 76. (b); Let the selling price be 250x
Let total students in A = x then, profit = 150x
And, total students in B = y CP=250x − 150x = 100x
175
75
Total students in school A in commerce stream = x × × Now, new C.P. = 100x × 100 = 175x
4
1 3x New S.P. = 250x
= New profit = 250x – 175x = 75x
100 16
200 150x−75x
Total students in school B in science stream = y × × Required % = 250x × 100 = 30%
7
1 2y
= 77. (c); Circumference of any circle = 2π × radius
100 7
3x 2y
Given, 16 + 7 = 420 … (i) 1386
Radius of 1st circle = √ π
= 21 cm
And x + y = 1800 … (ii) 176
So, from (i) and (ii), Radius of 2nd circle = = 28 cm

Total students in school A = 960 5
Side of square = × 2 × (21 + 28) = 35 cm
14
And total students in school B = 840 Perimeter of square = 4 × 35 = 140 cm
Total students in school B in commerce stream
400 1
= 21 × 100 × 840 = 160 78. (b); Ways to select 4 balls out of 16 balls = 16C 4
1 Ways to select one red balls = 5C1
Total students in school A in art stream = 2 × 960 = 480 Ways to select two black balls = 6C 2
Now, total students in school A in science stream Ways to select one blue balls = 5C1
3
= 960− × 960 − 480 = 300 ∴ Required probability =
5C 1 ×6C 2 ×5C 1
=
75
16
16C 4 364
And total students in school B in art stream
2
= 840 − 7 × 840 − 160 = 440 79. (b); Let the cost price be Rs 3x
Then the marked price= Rs 5x
Streams A B And let the discount given be Rs 4y
Art 480 440 Then loss incurred= Rs y
Commerce 180 160 ATQ
Science 300 240 3x − y = 5x − 4y
3y = 2x
480 −240 15
70. (d); Required percentage = × 100 = 100% Marked price=Rs 2 y
240
4y 1
71. (a); Required ratio =
160
= 8 : 15 Required discount %=15 × 100 = 53 3 %
300 y
2
5
72. (e); Total student art & commerce stream in C 80. (e); Speed of 1st train = 72 × 18 = 20 m/s
125
= 720 – 160 × = 520 ∴ Dist travelled by 1st train = 20 × 18 = 360 m
100
Required difference = (480 + 180) – 520 = 140 ∴ length of train (1st) = 360 – 160 = 200 m
5
300+240 Speed of 2nd train = 90 × 18 = 25 m/s
73. (b); Required average =
540
2 ∴ Distance travelled = 25 × 15 = 375 m
= = 270 ∴ length of 2nd train = 375 − 160 = 215 m
2

10 Adda247 Publications For any detail, mail us at


Publications@adda247.com
50+ Bank PO | Clerk Previous Year’s Papers 2016 – 2020

1 Adda247 Publications For any detail, mail us at


Publications@adda247.com
50+ Bank PO | Clerk Previous Year’s Papers 2016 – 2020

Mock IBPS RRB PO Prelims 2018


32
REASONING ABILITY
Direction (1-5): Study the following information 6. If point X is 6m south of point A then which point is at
carefully and answer the given questions: shortest distance from point X?
(a) E (b) A (c) F
Eleven boxes A, B, C, D, E, F, G, H, I, J, K are kept one above
(d) B (e) G
the other. Box G is kept at fifth position from the top. Two 7. What is the distance of point C from point H?
boxes are kept between G and H. Box D is kept just above (a) 9m (b) 5m (c) 4m
box H. There are as many boxes above box D as below box (d) 6m (e) 7m
B. Five boxes are kept between box F and box K, which is 8. Point B is in which direction with respect to point F?
kept at one of the positions below box G. Box A is kept at (a) South (b) South-east (c) North
one of the positions above box F. Only one box is kept (d) North-east (e) North-west
between Box G and Box C. Box I is kept above box E but Direction (9-13): Study the following information
not just above. Box E is not kept immediately above or carefully and answer the given questions
immediately below box C. Eight persons A, B, C, D, E, F, G, H are sitting around a
circular table such that five of them are facing towards the
1. What is the position of box I? center and the rest are facing away from the center. Three
(a) 8th from the bottom persons are sitting between F and H, who is facing center.
(b) 7th from the top C is 2nd to the right of F and faces opposite direction to F.
(c) 3rd from the top A sits 3rd to the left of C.G is one of the neighbor of E. Two
(d) 6th from the bottom persons sit between G and B, who is not neighbor of H.G
(e) none of these does not face C. G and A face same direction but opposite
to F.
2. How many boxes are kept between box E and Box H?
(a) seven (b)six (c) five 9. What is the position of E with respect to A?
(d) four (e) eight (a) immediate right (b) 5th to the left
(c) 2nd to the right (d) 2nd to the left
3. Which among the following statement is true (e) none of these
regarding box J?
(a) it is 7th from the bottom 10. How many persons are sitting between C and H, wen
(b) Box K is placed above box J counted from the left of C?
(c) only two boxes are kept between box B and box J (a) one (b) two (c) three
(d) four (e) none
(d) It is kept just below box H
11. Four of the five are alike in a certain way, which
(e) All are true
among the following does not belongs to that group?
4. Which of the following represents the boxes kept (a) C (b) B (c) F
between boxes A and I? (d) D (e) E
(a)C, B (b) A, K (c) F, G 12. Which of the following represents the immediate
(d) J, D (e) none of these neighbor of G?
(a) C (b) B (c) F
5. Which of the following box is kept just above box B?
(d) D (e) A
(a) C (b) K (c) F
(d) D (e) none of these 13. Which of the following is not true regarding F?
Direction (6-8): Study the following information (a) it faces towards the center
carefully and answer the given questions (b) E is immediate left to F
Point B is 14m east of point A. Point C is 9m north of point (c) Two persons sit between F and D, when counted
B. Point D is 12m east of point C. Point E is 15m south of from the right to D
point D. Point F is 30m west of point E. Point G is 10m (d) All are true
north of point F. Point H is 18 m east of point G. (e) no one sits between F and B

2 Adda247 Publications For any detail, mail us at


Publications@adda247.com
50+ Bank PO | Clerk Previous Year’s Papers 2016 – 2020
Direction (14-18): Study the following information Only one movie is released between the one which is of
carefully and answer the given questions. 75-minute duration and the one which is of 100-minute
duration. No movie released between the one which is of
Certain number of persons are sitting in a row facing 100 minute and B. Only one movie released after B. B
north. M sits 4th to the right of S. Five persons sit between released immediately after 100-minute duration movie.
M and X. T sits at one of the positions left to S. The Movie C released immediately after the one which is of
number of persons sitting between M and U are same as 130-minute duration. More than two movies released in
between S and T. Q is 2nd from one of the extreme ends. between C and D. The movie which is of 90-minute
Four persons sit between S and U. No one sits to the right duration released before E. One of movie was of 20
of N, who is immediate right to P. X is 3rd left to P. Not minutes more duration than E.
more than two persons sit between Q and U. 21. How many movies were released after E?
14. How many persons are sitting in the row? (a) One (b) Two (c) None
(d) Three (e) More than three
(a) 17 (b) 20 (c) 24
(d) 26 (e) 27 22. Which of the following movie was of 150-minute
duration?
15. How many persons are sitting between S and T? (a) E
(a) seven (b)six (c) five (b) A
(d) four (e) eight (c) There is no such movie
(d) C
16. What is the position of U from the left end?
(e) D
(a) 6th (b) 5th (c) 4th
(d) 2nd (e) 3rd 23. What is the total duration of movie D and E together?
(a) 135 (b) 225 (c) 165
17. How many persons are sitting between Q and M? (d) 175 (e) 190
(a) seven (b) eleven (c) ten 24. Which of the following statement is true regarding B?
(d) nine (e) eight (a) The movie released after B is of 120-minute
duration
18. Which of the following represents the person sitting (b) Two movies released in between A and B
at extreme end? (c) Movie B is of 100-minute duration
(a) M (b) U (c) X (d) Total duration of movie B and A is 225 minutes
(d) P (e) T (e) Movie A released after B.
19. If the second, forth, seventh and eighth letter of the 25. Which of the following statement is true?
word “FRACTION” are combined to form a meaningful (a) The movie released before A is of 130-minute
word, then what will be the 3rd letter from the left in duration
the so formed word. If more than one meaningful (b) Three movies released in between A and E
word is formed then the answer is X, if no such word (c) No movie released in between A and E
is formed then answer is Z? (d) Total duration of movie C and A is 230 minutes
(e) Movie C released immediately after E.
(a) O (b) X (c) R
(d) Z (e) C Direction (26-28): Study the following information
carefully and answer the given questions:
20. How many pair of digits have same number of digits F is the husband of G. K is the mother-in-law of G. H is the
between them in the number “573814269” as in the Father of F. M is the mother of H, P is the mother of K and
numeric series? B.
(a) five (b) four (c) six 26. If Y is the father of H then how is Y related to M?
(d) three (e) more than six (a) Mother (b) Father (c) Sister
(d) Brother (e) Husband
Direction (21-25): Study the following information
carefully and answer the given questions: 27. How is P related to F?
(a) Grandfather (b) Aunt (c) Mother
Movies of different duration released on different days (d) Grandmother (e) Wife
starting from Monday to Friday (starting from Monday).
Movie A was released On Tuesday. No movie released 28. How is B related to H?
between A and the one which is of 75-minute duration. (a) Sister (b) Brother (c)Husband
(d) Can’t be determine (e) Wife
3 Adda247 Publications For any detail, mail us at
Publications@adda247.com
50+ Bank PO | Clerk Previous Year’s Papers 2016 – 2020
Direction (29-31): Study the following information (a) Neither I nor II follows
carefully and answer the given questions:There are six (b) Only I follows
persons M, N, O, P, Q, R of different heights. N is shorter (c) Either I or II follow
than M but taller than Q. Only two person are taller than (d) Both I and II follow
M. R is taller than Q and O. Q is not the shortest. The one (e) Only II follows
who is second shortest is 154m. P is not the shortest 35. Statements: Some Door are Fan. No Door is Rose. No
person. Fan is Shelf.
29. If M is 19m taller than Q then what is the height of M? Conclusions:
(a) 190m (b) 181m (c) 175m I. All Door are Shelf is a possibility.
(d) 130m (e) 173m II. All Shelf can be Doors.
(a) Either I or II follows
30. If P is 181m than which of the following is true? (b) Only II follows
I. Only one person is taller than P. (c) Neither I nor II follow
II. The difference between the heights of P and Q is (d) Both I and II follow
27m (e) Only I follows
III. O is the shortest person.
Direction (36-40): Study the following information
(a) Only I (b) Only II and I
carefully and answer the given questions:
(c) All are true (d) Only III and II
Fourteen persons are sitting in two parallel rows such
(e) Only III and I that seven persons are sitting in each row. A, B, C, D, E, F,
31. How many persons are shorter than N? G are sitting in row-1 facing north while P, Q, R, S, T, U, V
(a) One (b) Two (c) None are sitting in row-2 facing south. G sits third to the left of
(d) Three (e) More than three A and neither of them sits at an extreme end of the row.
The one faces A sits immediate right to T. Only one person
Directions (32-35): Question consists of Some sits between T and Q. The one who faces Q sits third to the
statements followed by two conclusions. Consider the right of E. S sits to the immediate left of V. S neither faces
given statements to be true even if they seem to be at G nor E. D is an immediate neighbour of the one who faces
variance with commonly known facts. Read all the S. The one who faces C sits fifth to the left of P. B sits third
conclusions and then decide which of the given to the left of F. U sits at one of position to the right of R.
conclusions logically follow from the given statements
using all statements together. 36. Four of the following are alike in a certain way so
form a group which of the following does not belong
32. Statements: All Grills are Arrow. Some Hat are Grills. to that group?
Some Cell are Arrow. (a) U (b) B (c) T
Conclusions: (d) C (e) P
I. Some Cell are definitely not Grills.
37. How many persons sits between F and C?
II. Some Hat can never be Arrow.
(a) One (b) Two (c) None
(a) Only I follows (d) Three (e) More than three
(b) Only II follows 38. Which of the following is not true regarding U?
(c) Neither I nor II follow (a) No one sits to the right of U
(d) Both I and II follow (b) U sits third to the right of Q,
(e) Either I or II follow (c) P is an immediate neighbour of U.
33. Statements: All Grills are Arrow. Some Hat are Grills. (d) E is an immediate neighbour of the one who faces
Some Cell are Arrow. U,
Conclusions: (e) Only two persons sit between U and S
I. Some Hat are Arrow. 39. What is the position of C with respect to A?
II. Some Grills are Cell. (a) Second to the left
(a) Only II follows (b) Third to the right
(b) Only I follows (c) Immediate right
(c) Either I nor II follow (d) Immediate left
(d) Both I and II follow (e) Second to the right
(e) Neither I or II follow
40. What is the position of B with respect to D?
34. Statements: Some Door are Fan. No Door is Rose. No (a) Third to the left
Fan is Shelf. (b) Second to the left
Conclusions: (c) Forth to the left
I. Some Fan can never be Rose. (d) Third to the right
II. Some Rose are Shelf is a possibility. (e) Fifth to the right

4 Adda247 Publications For any detail, mail us at


Publications@adda247.com
50+ Bank PO | Clerk Previous Year’s Papers 2016 – 2020

QUANTITATIVE APTITUDE

Directions (41-45): Find the wrong number in the 43. 250, 260, 291, 314, 340, 370, 405
following number series ? (a) 370 (b) 314 (c) 260
(d) 405 (e) 250
41. 1, 3, 7, 15, 31, 64, 127
44. 750, 535, 411, 348, 322, 314, 315
(a) 1 (b) 3 (c) 15
(a) 315 (b) 750 (c) 411
(d) 64 (e) 127
(d) 348 (e) 314
42. 1, 15, 119, 475, 949, 947, 473 45. 2, 7, 27, 107, 427, 1708, 6827
(a) 947 (b) 475 (c) 15 (a) 107 (b) 1708 (c) 2
(d) 473 (e) 1 (d) 6827 (e) 7

Directions (46-50): Study the line-graph carefully & answer the question given below.
Line-graph given below shows the total no. of products for (kid + adult) in two different stores P & Q in five different
years.
Store P Store Q
80

70
No. of product

60

50

40

30

20
2000 2001 2002 2003 2004

46. What is the difference between total no. of products in (a) 150% (b) 40% (c) 125%
store P in year 2003 & 2004 together and total no. of (d) 100% (e) 50%
products in year 2000?
(a) None of these (b) 10 (c) 20 Directions (51-55): Solve the given quadratic equations
(d) 15 (e) 5 and mark the correct option based on your answer—
47. If total products in both the stores in year 2006 is (a) x ≥ y
increased by 20% as compared to year 2004. Then (b) x ≤ y
find total no. of products in year 2006? (c) x > y
(a) 102 (b) None of these (d) x = y or no relation can be established between x and
(c) 96 (d) 108 y.
(e) 92 (e) x < y
48. What is the ratio of total products in store Q in year 51. (i) 𝑥 2 − 20𝑥 + 96 = 0
2002 & 2003 together to total products in store Q in (ii) 𝑦 2 = 64
year 2000?
(a) 23 : 12 (b) 23 : 11 (c) 28 : 11 52. (i) 4𝑥 2 − 21𝑥 + 20 = 0
(d) None of these (e) 27 : 13 (ii) 3y2 − 19y + 30 = 0
49. What is the average no. of products in all the years 53. (i) x2 − 11x + 24 = 0
together in store P? (ii) y2 − 12y + 27 = 0
(a) 48 (b) 43 (c) 57
(d) None of these (e) 53 54. (i) x2 + 12x + 35 =0
(ii) 5y2 + 33y + 40 =0
50. Total no. of products in store P in year 2003 and in
store Q in year 2004 together is what percent 55. (i) 4x2 + 9x + 5 =0
more/less than total no. of products in store Q in year
(ii) 3y2 + 5y + 2 =0
2000?
5 Adda247 Publications For any detail, mail us at
Publications@adda247.com
50+ Bank PO | Clerk Previous Year’s Papers 2016 – 2020
Directions (56-60): Study the following paragraph (a) 240 (b) 260 (c) 300
carefully & answer the question given below. (d) 360 (e) 500
There are 1000 students in a college. Out of 1000 students
57. How many students appeared in two exams only?
some appeared in exams ‘X’, ‘Y’ and ‘Z’ while some not.
Number of student not appeared in any exam is equal to (a) 280 (b) 220 (c) 340
number of students appeared in exam ‘Z’ only. Number of (d) 300 (e) 260
students appeared in exam ‘Y’ is 360. Ratio of number of 58. How many students appeared in at most two exams?
students appeared in exam ‘X’ and ‘Y’ only to number of (a) 240 (b) 260 (c) 300
students appeared in exam ‘Y’ and ‘Z’ only is 2 : 3. Number
(d) 500 (e) 960
of student appeared in exam ‘X’ and ‘Z’ both is half of
number of students appeared in only exam ‘Z’. Number of 59. How many students not appeared in exam Y?
students appeared in exam ‘X’ only is 50% more than (a) 440 (b) 360 (c) 540
number of students appeared in ‘Y’ only. Number of
(d) 640 (e) None of these
students appeared in all the three exam is 4% of the total
number of students in the college. Number of students 60. How many students appeared in exam X or in exam Z?
appeared in ‘Y’ exam only is same as number of students
(a) 240 (b) 360 (c) 500
appeared in ‘Y’ and ‘Z’ only.
56. How many students appeared in at least two exams? (d) 680 (e)760

Direction (61-65): Bar chart given below shows Number of tigers in different National Parks i.e. A to D of a country in
two different years. Study the data carefully and answer the following questions

1998 2018
100

80
Number of tigers →

60

40

20

0
A B C D
National Parks→

61. Number of tigers in National Park B and C together in (a) 9 : 10 (b) 10 : 9 (c) 16 : 13
2018 is how much less more/less than Number of (d) 13 : 16 (e) 3 : 4
tigers in National Park A and D together in 1998?
(a) 40 (b) 44 (c) 52 64. Number of tigers in National Park ‘E’ in 2018 is 40%
(d) 60 (e) 72 more than number of tigers in National Park ‘D’ in
62. Number of tigers in National Park ‘D’ in both years 1998 while number of tigers in National park ‘E’ in
together is what percent of the Number of tigers in 1998 is 25% less than number of tigers in National
National Park ‘C’ in both years together? Park ‘C’ in 2018. Find total number of tigers in
(a) 60% (b) 160% (c) 140%
National park ‘E’ in 1998 and 2018 together?
(d) 120% (e) 180%
(a) 148 (b) 84 (c) 172
63. Find the ratio between number of tigers in National
(d) 160 (e) 136
Park ‘A’ in 2018 to number of tigers in National Park
‘B’ in 1998?

6 Adda247 Publications For any detail, mail us at


Publications@adda247.com
50+ Bank PO | Clerk Previous Year’s Papers 2016 – 2020
65. Average number of tigers in all National park in 2018 (a) Rs. 40,000 (b) Rs. 35,000
is how much less/more than average number of tigers (c) Rs. 32,000 (d) Rs. 30,000
in all National park in 1998? (e) Rs. 25,000
(a) 14 (b) 16 (c) 18
(d) 20 (e) 22 73. The sum of four times of an amount ‘x’ and (x – 9.75)
is Rs. 442. Find the approximate value of x.
66. The difference between downstream speed and (a) Rs. 85 (b) Rs. 90 (c) Rs. 100
upstream speed of boat is 6 km/hr and boat travels (d) Rs. 1100 (e) Rs. 75
72 km from P to Q (downstream) in 4 hours. Then
find the speed of boat in still water? 74. A and B entered into a partnership by investing some
(a) 15 km/hr (b) 18 km/hr (c)20km/hr amounts. The investment of A is twice of the
(d) 16 km/hr (e) 24 km/hr investment of B. Another person C joined them after 4
months. At the end of a year, the profit share of A and
67. In a vessel, there are two types of liquids A and B in C is equal. Then find the profit share of B is what
the ratio of 5 : 9. 28 lit of the mixture is taken out and percent of the profit share of C.
2 lit of type B liquid is poured into it, the new 1
(a) 50% (b) 333% (c) 40%
ratio(A:B) thus formed is 1 : 2. Find the initial
quantity of mixture in the vessel? (d) 60% (e) 75%
(a) 84 lit (b) 42 lit (c) 50 lit 75. The ratio of age of Ishu 8 years hence and that of
(d) 56 lit (e) 70 lit Ahana 6 years hence is 5 : 6. The age of Ishu 10 years
68. The average weight of 5 students in a class is 25.8 kg. hence is equal to the age of Ahana 6 years hence.
When a new student joined them, the average weight Then, find the present age of Ishu.
is increased by 3.9 kg. Then find the approximate (a) 1.5 yr (b) 2 yr (c) 3 yr
weight of the new student. (d) 4 yr (e) 5 yr
(a) 55 kg (b) 49 kg (c) 42 kg 76. What is the difference between 20% of P and 20% of
(d) 44 kg (e) 58 kg
(P + 5000).
69. A person has purchased two adjacent plots, one is in (a) 1500 (b) 1200 (c) 1000
rectangular shape and other is in square shape and (d) 2000 (e) 1600
combined them to make a single new plot. The
77. The ratio of the diameter of base and height of a
breadth of the rectangular plot is equal to the side of
the square plot and the cost of fencing the new plot is cylinder is 2 : 3. Find the radius of the cylinder if the
Rs. 390 (Rs. 5/m). Find the side of square if the length approximate volume of cylinder is 3234.01 cm³?
21 7
of the rectangular plot is 15 m. (a) cm (b) cm (c) 21 cm
2 2
(a) 10 m (b) 8 m (c) 12 m (d) 7 cm (e) 14 cm
(d) 9 m (e) 6 m
78. A train of some length passes the platform of length
70. A shopkeeper marked his article 50% above the cost 524 m in 55 seconds. Find the length of train if the
price and gives a discount of 20% on it. If he had speed of train is 72 km/hr.
marked his article 75% above the cost price and gives (a) 476 m (b) None of these
a discount of 20% on it then find the earlier profit is (c) 428 m (d) 526 m
what percent of the profit earned latter? (e) 576 m
1
(a) 50% (b) 60% (c) 333%
79. Efficiency of B is two times more than efficiency of A.
(d) 40% (e) 75% Both started working alternatively, starting with B
71. A person invested two equal amounts in two different and completed the work in total 37 days. If C alone
schemes. In first scheme, amount is invested at 8% complete the same work in 50 days then find in how
p.a. on SI for T years and SI received is Rs 2000 while many days A and C together will complete the work?
in second scheme, amount is invested at 10% p.a. for
2 years at CI and the compound interest received is (a) 24 days (b) 30 days (c) 36 days
Rs. 1050. Find the value of T. (d) 48 days (e) 18 days
(a) 4 yr (b) 8 yr (c) 6 yr
80. 7 men and 6 women together can complete a piece of
(d) 5 yr (e) 3 yr
work in 8 days and work done by a women in one day
72. Satish saves 20% of his monthly salary. And of the is half the work done by a man in one day. If 8 men
1 1
remaining salary 4th and 2th he gives to his mother and and 4 women started working and after 3 days 4 men
sister respectively and the remaining salary he left the work and 4 new women joined then, in how
submits as his EMI for the payment of his car. If his many more days will the work be completed
annual EMI was Rs. 60,000, then find his monthly (a) 7 days (b) 6 days (c) 5.25 days
salary? (d) 6.25 days (e) 8.14 days

7 Adda247 Publications For any detail, mail us at


Publications@adda247.com
50+ Bank PO | Clerk Previous Year’s Papers 2016 – 2020

Solutions

REASONING ABILITY

Direction (1-5): 19. (c); 2nd, 4th,7th and 8th letters are R, C, O, N
BOX The meaningful word formed is CORN
D 20. (e);
H
A
F
G
I Direction (21-25):
C Days Movies Duration
J Monday D 75
E Tuesday A 90
K Wednesday E 100
B Thursday B 130
1. (d); 2. (b); 3. (c); Friday C 120

4. (c); 5. (b); 21. (b); 22. (c); 23. (d);

Direction (6-8): 24. (a); 25. (c);


Direction (26-28):

26. (e); 27. (d); 28. (d);


Direction (29-31): R/P > R/P > M > N > Q > O
6. (c); 7. (b); 8. (d); 29. (e); 30. (d); 31. (b);
Direction (9-13): Directions (32-35):
32. (c);

9. (d); 10. (a); 11. (a);


33. (b);
12. (e); 13. (c);
Direction (14-18):

14. (c); 15. (e); 16. (b);


17. (b); 18. (e);

8 Adda247 Publications For any detail, mail us at


Publications@adda247.com
50+ Bank PO | Clerk Previous Year’s Papers 2016 – 2020
34. (d); Direction (36-40):

36. (e);
35. (b);
37. (b);

38. (b);

39. (c);

40. (c);

QUANTITATIVE APTITUDE

41. (d); 47. (a); Total no. of products in year 2006


120
= (55 + 30) × = 102.0 100

48. (c); Required ratio


80+60 140
= = = 28 : 11
55 55
42. (a);
49. (b); Required Average
25+40+65+55+30 215
= = = 43
5 5

50. (d); Required percentage


(55 + 55) − 55
= × 100
43. (c); 55
55
= × 100 = 100%
55
51. (a); (i) 𝑥 2 − 20𝑥 + 96 = 0
𝑥 2 − 12𝑥 − 8𝑥 + 96 = 0
𝑥(𝑥 − 12) − 8(𝑥 − 12) = 0
44. (e); (𝑥 − 12)(𝑥 − 8) = 0
𝑥 = 12,8
(ii) 𝑦 2 = 64
𝑦 = ±8
∴ 𝑥≥𝑦
52. (d); (i) 4𝑥 2 − 21𝑥 + 20 = 0
45. (b); 4𝑥 2 − 16𝑥 − 5𝑥 + 20 = 0
4𝑥(𝑥 − 4) − 5(𝑥 − 4) = 0
(4𝑥 − 5)(𝑥 − 4) = 0
5
𝑥 = 4,4
Alternate,
(ii) 3𝑦 2 − 19𝑦 + 30 = 0
3𝑦 2 – 9𝑦 − 10𝑦 + 30 = 0
3𝑦(𝑦 − 3) − 10(𝑦 − 3) = 0
(3𝑦 − 10)(𝑦 − 3) = 0
10
𝑦 = 3 ,3
46. (e); Required difference ∴ No relation can be established between
= (55 + 30) – (55 + 25) = 5 x and y

9 Adda247 Publications For any detail, mail us at


Publications@adda247.com
50+ Bank PO | Clerk Previous Year’s Papers 2016 – 2020
2 2
53. (d); (i) 𝑥 − 11𝑥 + 24 = 0 = 3 × 3𝑥 = 2𝑥
𝑥 2 − 8𝑥 − 3𝑥 + 24 = 0
𝑥(𝑥 − 8) − 3(𝑥 − 8) = 0
(𝑥 − 3)(𝑥 − 8) = 0
𝑥 = 3 ,8
(ii) 𝑦 2 − 12𝑦 + 27 = 0
𝑦 2 – 9𝑦 − 3𝑦 + 27 = 0
𝑦(𝑦 − 9) − 3(𝑦 − 9) = 0
(𝑦 − 9)(𝑦 − 3) = 0
𝑦 = 9,3
∴ No relation can be established between
x and y
54. (b); (i) 𝑥 2 + 12𝑥 + 35 = 0
𝑥 2 + 7𝑥 + 5𝑥 + 35 = 0 Now, 2x + 3x + 3x + 40 = 360
𝑥(𝑥 + 7) + 5(𝑥 + 7) = 0 ⇒ x = 40
(𝑥 + 7)(𝑥 + 5) = 0 𝑎
𝑎𝑛𝑑, 12.5𝑥 + 𝑎 + + 𝑎 = 1000
𝑥 = −7 , −5 2
5𝑎
(ii) 5𝑦 2 + 33𝑦 + 40 = 0 2
= 500
5𝑦 2 + 25𝑦 + 8𝑦 + 40 = 0 ⇒ a = 200
5𝑦(𝑦 + 5) + 8(𝑦 + 5) = 0
(𝑦 + 5)(5𝑦 + 8) = 0
8
𝑦 = − , −5
5
∴ 𝑦≥𝑥
55. (b); (i) 4𝑥 2 + 9𝑥 + 5 = 0
4𝑥 2 + 4𝑥 + 5𝑥 + 5 = 0
4𝑥(𝑥 + 1) + 5(𝑥 + 1) = 0
(4𝑥 + 5)(𝑥 + 1) = 0
5
𝑥 = −1 , −
4
(ii) 3𝑦 2 + 5𝑦 + 2 = 0
3𝑦 2 + 3𝑦 + 2𝑦 + 2 = 0
3𝑦(𝑦 + 1) + 2(𝑦 + 1) = 0 56. (c); Students appeared in atleast two exams = 80 +
(3𝑦 + 2)(𝑦 + 1) = 0 60 + 40 + 120
2 = 300
𝑦 = − , −1
3
∴ 𝑦≥𝑥 57. (e); Students appeared in two exams only = 80 + 60
+ 120 = 260
Solutions (56-60):
58. (e); Students appeared in atmost two exams = 180 +
Total students = 1000
120 + 200 + 60 + 80 + 120 + 200= 960
Let, students appear in exam Z only = a
Total students appeared in exam Y = 360 59. (d); Student not appeared in exam Y = 1000 – 360 =
Ratio of number of students appeared in exam X and Y 640
only to students appeared in exam Y and Z only = 2 : 3 60. (d); Students appeared in exam X or in exam Z
Students appeared in exam X and Z both = 180 + 60 + 40 + 80 + 200 + 120
= a/2 = 680
Number of students appeared in all three exams = 61. (d); Number of tigers in National Park B and C
4
× 1000 = 40 together in 2018 = 52 + 32 = 84
100
Number of students appeared in Y exam only Number of tigers in National Park A and D
= No. of students appeared in Y and Z only together in 1998
= 3x = 64+80 = 144
Number of students appeared in exam X and Y only Required difference = 144-84 = 60

10 Adda247 Publications For any detail, mail us at


Publications@adda247.com
50+ Bank PO | Clerk Previous Year’s Papers 2016 – 2020
62. (b); Number of tigers in National Park D in 1998 and Let the breadth of rectangular plot be y m and
2018 together = 80 + 48 = 128 length = 15 m
Number of tigers in National Park C in 1998 and ATQ,
2018 together = 48 + 32 = 80 30 + y + 3y = 390/5
128
Required % = × 100 = 160% ⇒ 30 + 4y = 78
80
⇒ 4y = 48 ⇒ y = 12 m
36 9
63. (a); Required Ratio = 40 = 10
70. (a); Let the CP be Rs. 100x
64. (e); Number of tigers in National Park E in 2018 = Then, MP = Rs. 150x
80
140
× 80 = 112 SP = 150x × 100 = Rs. 120x
100
Number of tigers in National Park E in 1998 = Profits = Rs. 20x
75 New MP = Rs. 175x
100
× 32 = 24 80
New SP = 175x × 100 = Rs. 140x
Number of tigers in National Park E in 1998 and
2018 together New Profit = Rs. 40x
20𝑥
= 112 + 24 = 136 Required % = 40𝑥 × 100 = 50%

65. (b); Total number of tigers in 2018 71. (d); Let the amount be Rs. x
= 36 + 52 + 32 + 48 = 168 CI at 10% in 2 years = 10 + 10 +
10 × 10
100
Total number of tigers in 1998
= 21%.
= 64 + 40 + 48 + 80 = 232
232 168 ATQ,
Required difference = − 𝑥 × 21
4 4 = 1050 ⇒ x = Rs. 5000
64 100
= = 16 And,
4
5000 × 8 × 𝑇
66. (a); Let the speed of boat in still water be x km/hr = 2000
and that of stream be y km/hr 100
⇒ T = 5 years.
ATQ,
(x + y) – (x – y) = 6 72. (e); Let the monthly salary be Rs. 100 x.
⇒ 2y = 6 ⇒ y = 3 km/hr EMI per month
72 1 1
Downstream stream = (x + y) = = 18 km/hr = 100x – (20𝑥 + 80𝑥 × 4 + 80𝑥 × 2) = Rs. 20x
4
⇒ x = 15 km/hr ATQ,
20x × 12 = 60,000
67. (d); Let the initial quantity of mixture in vessel be x
⇒ x = 250
lit
Monthly Salary = Rs. 25,000
ATQ,
5
𝑥× −10 1 73. (b); ATQ,
9
14
=
𝑥× −18+2
14
2 4x + x – 9.75 = 442
5𝑥−140 1 5x = 451.75
⇒ =
9𝑥−224 2
x = Rs. 90
⇒ 10x – 280 = 9x – 224
⇒ x = 56 lit 74. (a); Let the investment of B be Rs. x
∴ investment of A = Rs 2x
68. (b); Weight of new student = 6 × (25.8 + 3.9) – 5 ×
Ratio of profit,
25.8
𝐴 : 𝐵 : 𝐶
≈ 49 kg
12 × 2𝑥 : 12 × 𝑥 : 8 × 𝑦
69. (c);
ATQ,
24x = 8y
y = 3x
12 × 𝑥
∴ Required percentage = 8 × 3𝑥 × 100
= 50%

11 Adda247 Publications For any detail, mail us at


Publications@adda247.com
50+ Bank PO | Clerk Previous Year’s Papers 2016 – 2020
75. (b); Let present age of Ishu & Ahana be x year & y 5
78. (e); Speed of train in m/s. = 72 × = 20 m/s
18
year respectively
Let length of train be x m
∴ ATQ,
ATQ,
𝑥+8 5
= 524 + 𝑥
𝑦+6 6 = 20
55
6x + 48 = 5y + 30 x = 1100 – 524 = 576m
6x – 5y = – 18 … (i)
x + 10 = y + 6 79. (b); Lets efficiency of A is x unit/day and B’s
x–y=–4 … (ii) efficiency is 3x unit/day
So, B work for 19 days and A work for 18 days
∴ x = 2 years
ATQ—
∴ present age of Ishu is 2 years.
Total work = 19 × 3x + 18 × x = 75x
20 20 75𝑥
76. (c); quired difference = 100 (𝑃 + 5000) − 100 × 𝑃 𝐸𝑓𝑓𝑖𝑐𝑖𝑒𝑛𝑐𝑦 𝑜𝑓 𝐶 =
50
= 1000 = 1.5𝑥 𝑢𝑛𝑖𝑡/𝑑𝑎𝑦
75𝑥
77. (d); Let diameter of base be 2x cm & height of (𝐴 + 𝐶) 𝑡𝑜𝑔𝑒𝑡ℎ𝑒𝑟 = (𝑥+1.5𝑥)
cylinder be 3x cm = 30 𝑑𝑎𝑦𝑠
2𝑥
∴ radius = = 𝑥 cm 80. (d); One day work of women = half of work done by
2
We know, men in one day
Volume of cylinder = 𝜋𝑟 2 ℎ (r→ radius, h → Let efficiency of one women = w unit/day
Man’s efficiency = 2w unit/day
height) Total work = (7 × 2w + 6 × w) × 8 =160w unit
ATQ, 8 men and 4 women start work for 3 days
𝜋𝑟 2 ℎ = 3234 Total work done = (8 × 2w + 4 × w) × 3
22 = 60w
× 𝑥 2 × 3𝑥 = 3234 4 women replace 4 man
7
x = 7cm = (4 × 2w + 8 × w) =16w
100𝑤
Radius = 7cm Days required = = 6.25 days
16𝑤

12 Adda247 Publications For any detail, mail us at


Publications@adda247.com
50+ Bank PO | Clerk Previous Year’s Papers 2016 – 2020

1 Adda247 Publications For any detail, mail us at


Publications@adda247.com
50+ Bank PO | Clerk Previous Year’s Papers 2016 – 2020

Mock IBPS RRB PO Prelims 2017


33
REASONING ABILITY
1. What should come in place of question mark (?) in the 7. Which of the following boxes is kept at the top?
following series based on the above arrangement? (a) B (b) A (c) D
ZN XD UG QK ? (d) E (e) None of these
(a) LK (b) LO
(c) LP (d) KP 8. Choose the odd one out?
(e) Other than the given options (a) B (b) G (c) A
(d) D (e) E
2. How many such pair of numbers are there in the given
number “46579739” (Both backward and forward) 9. Which of the following boxes is kept between F and A?
same as far as according to numeric series? (a) B (b) G
(a) One (b) Two (c) C (d) H
(c) Three (d) More than three (e) None as box F is immediately above box A
(e) None of these. 10. How many boxes are there between C and A
3. If it is possible to make only one meaningful word with (a) Less than 2 (b) 4 (c) 5
the 1st ,2nd ,4th and 7th letters of the word ‘ECUADOR’ (d) 6 (e) None of these
which would be the second letter of the word from the Directions (11-15): In these questions, relationships
right? If more than one such word can be formed give between different elements are shown in the statements.
‘Y’ as the answer. If no such word can be formed, give
These statements are followed by two conclusions. Give
‘Z’ as your answer.
answer
(a) Y (b) E (c) I
(a) if only conclusion I follows
(d) Z (e) M
(b) if only conclusion II follows
4. If 1 is subtracted from each odd number and 2 is added (c) if either conclusion I or conclusion II follows
to each even in the number 9436527, then how many (d) if neither conclusion I nor conclusion II follows
digits will appear twice in the new number thus (e) if both conclusions I and II follow
formed?
(a) Only 8 (b) Only 8 and 6 (c) 8, 6 and 4 11. Statement: R ≥ S ≥ T > U > X; T < V < W
(d) 2, 4 and 6 (e) None of these Conclusions: I. R > X II. X < W

5. How many letter will be remain the same position in 12. Statement: E = F < G < H; G ≥ I
the word ‘MONSTER’ when they arranged in the Conclusions: I. H > I II. E > I
ascending order from left to right? 13. Statement: A > B > F > C; D > E > C
(a) One (b) Two (c) Three Conclusions: I. C < A II. B > D
(d) More than Three (e) None
14. Statement: K ≤ L ≤ M = N; P ≥ O ≥ N
Directions (6-10): Read the following information Conclusions: I. K < P II. K = P
carefully and answer the following questions.
Eight boxes A, B, C, D, E, F, G and H are place one above the 15. Statement: D < E < F < G; K > F
other in any particular order. Box no. 1 is at the bottom and Conclusions: I. K ≤ G II. K > D
box no. 8 is at the top. Three boxes are placed between A Directions (16-20): Read the following information
and B. Box H is placed immediately below A. There are two carefully and answer the following questions.
boxes between H and G. There are as many boxes between Seven persons A, B, C, D, E, F and G were born on different
C and D as between H and B. Box C is kept above D. Box E months viz. January, February, March, April, June, August
is kept immediately below box D. Three boxes are there and October of the same year, but not necessarily in the
between E and F. same order.
Only three persons were born before E and D is not one of
6. How many boxes are there above box D? them. F was not born immediately after E. B was born after
(a) 4 (b) 3 (c) 6 F. A was born immediately before the month in which G
(d) 2 (e) None of these was born. Only two persons were born between G and F.

2 Adda247 Publications For any detail, mail us at


Publications@adda247.com
50+ Bank PO | Clerk Previous Year’s Papers 2016 – 2020
16. How many persons were born between C and E? arrangement each member seated in a row faces another
(a) Three (b) Two (c) Four member of the other row. Q sits fourth to the left of A. The
(d) Five (e) None of these one facing A sits third to the left of S. Only one person sits
17. Who amongst the following is the oldest? between S and E. E does not sit at any of the extreme ends
(a) A (b) C (c) E of the line The one facing U sits second to the right of B. U
(d) B (e) F does not sit at any of the extreme ends of the line. Only two
people sit between B and Y. The one facing B sits second to
18. Who amongst the following was born between the the left of Z. F is not an immediate neighbour of U. P is not
months in which A and D were born? immediate neighbour of Q.
(a) E (b) G (c) C
(d) B (e) Both E and G 26. Which of the following groups of people represents the
people sitting at extreme ends of both the rows?
19. How many persons were born after D? (a) Q, Y, Z, R (b) F, Y, F, B (c) S, Y, Z, R
(a) One (b) Three (c) Four (d) Q, F, Z, B (e) Q, Y, Z, S
(d) Two (e) None of these
27. Who amongst the following faces, F?
20. Who amongst the following is the person who was
(a) Q (b) P (c) A
born in the month which has less than 30 days?
(d) X (e) B
(a) F (b) B (c) G
(d) C (e) A 28. Which of the following is true with respect to the given
Directions (21-25): Study the following information information?
carefully and answer the given questions: (a) B faces one of the immediate neighbours of Z.
In a certain code language (b) F sits exactly between R and E.
‘card win team time’ is written as ‘la ta ja sa’ (c) None of the given options is true
‘fight game play card’ is written as ‘ja pa ra da’ (d) A is an immediate neighbour of B
‘in win team fight’ is written as ‘da ta fa la’. (e) A faces U.
21. What is the code for ‘time’? 29. Which of the following is true regarding X?
(a) sa (b) da (c) ja (a) B sits second to the right of X.
(d) la (e) None of these (b) F is an immediate neighbor of the person who faces
X
22. ‘card fight in’ can be coded as?
(c) Both P and Y are immediate neighbours of X
(a) sa ja ra (b) fa ja da
(d) Only one person sits between X and A
(c) da ra ta (d) Can’t be determined
(e) None of the given options is true
(e) None of these
30. Who amongst the following sits second to the right of
23. What is the code for ‘game’? the person who faces P?
(a) ra (a) F (b) U (c) R
(b) pa (d) E (e) S
(c) Either ra or pa
(d) da Directions (31-35): Study the following information
(e) None of these carefully and answer the questions given below:
24. Which of the following is the code for ‘in’? Eight friends M, N, O, P, Q, R, S and T are sitting around a
(a) ta (b) da (c) la circular table with equal distance between them but not
(d) fa (e) None of these necessarily in the same order. Some of them are facing the
25. If ‘game in risk’ is coded as ‘Pa fa xa’ than what will be centre with some face outside (i.e. opposite to centre).
the code for ‘risk card fight’? O sits second to the right of R, R faces the centre. Only two
(a) Ja sa da (b) ja da ra (c) sa da fa people sit between O and N (either form O’s right or O’s
(d) xa ja da (e) None of these left). S sits second to the right of O. T sits to the immediate
Directions (26-30): Study the following information to right of N. S and N face opposite direction (i.e. if N faces the
answer the given questions centre then S faces outside and vice versa). Immediate
Twelve people are sitting in a two parallel rows containing neighbor of S face the same direction (i.e. If one neighbor
six people each in such a way that there is an equal faces the centre then the other also faces the centre and
distance between adjacent persons. In row 1 – A, B, P, Q, X vice-versa) Only three people sit between P and Q. Neither
and Y are seated (but not necessarily in the same order) P nor M is an immediate neighbor of R. Q sits second to the
and all of them are facing south. In row 2 – E ,F ,R ,Z ,S and right of M. Both T and Q face a direction opposite to that of
U are seated (but not necessarily in the same order) and all O (i.e. if O faces the centre then both T and Q faces outside
of them are facing North. Therefore in the given seating and vice-versa).

3 Adda247 Publications For any detail, mail us at


Publications@adda247.com
50+ Bank PO | Clerk Previous Year’s Papers 2016 – 2020
31. Who sits exactly between M and P? Give answer
(a) N (b) S (c) R (a) If only conclusion I follows.
(d) Q (e) None of these (b) If only conclusion II follows.
(c) If either conclusion I or II follows.
32. How many people in the given arrangement face the (d) If neither conclusion I nor II follows.
centre? (e) If both conclusions I and II follow.
(a) One (b) Three (c) Five
36. Statements: All bags are purses.
(d) Four (e) None of these No purse is black.
33. Who sits second to the right of T? All blacks are covers.
(a) O (b) Q Conclusions: I. All bags are covers
II. Some covers are purses.
(c) S (d) R
(e) Other than the given options 37. Statements: Some cats are rats.
Some rats are fishes.
34. Four of the following five are alike in a certain way All fishes are birds.
based on the given seating arrangement and so form a Conclusions: I. Some fishes are rats.
group. Which is the one that does not belong to that II. All cats being birds is a possibility
group?
38. Statements: Some flowers are roses.
(a) P (b) O (c) T No rose is red.
(d) M (e) Q All red are leaves.
Conclusions: I. Some flowers are definitely not red.
35. What is P’s position with respect to R?
II. Some leaves are definitely not roses.
(a) Second to the left (b) Third to the right
(c) Third to the left (d) Sixth to the right 39. Statements: All cards are sheets.
(e) Second to the right All files are cards.
Some sheets are papers.
Directions (36–40): In each question below are given
Conclusions: I. All files being papers is a possibility.
some statements followed by two conclusions numbered I
II. All files are not sheets.
and II. You have to take the given statements to be true
even if they seem to be at variance with commonly known 40. Statements: Some flowers are roses.
facts. Read all the conclusions and then decide which of the No rose is red.
given conclusions logically follows from the given All red are leaves.
statements, disregarding commonly known facts. Conclusions: I. Some flowers are not leaves.
II. No leave is a red.

QUANTITATIVE APTITUDE
Directions (41-45): What should come in place of the 45. 89, 88, 85, 78, 63, ?
question mark (?) in following number series problems? (a) 30 (b) 34 (c) 36
(d) 32 (e) None of these
41. 190, 94, 46, 22, ? , 4
46. There are 3 consecutive odd numbers and 3
(a) 12 (b) 14 (c) 10
consecutive even numbers. The smallest even number
(d) 8 (e) None of these is 9 more than largest odd number. If the square of
42. 5, 28, 47, 64, 77, ? average of all the 3 given odd number is 507 less than
the square of the average of all the 3 given even
(a) 84 (b) 86 (c) 89
number, what is the smallest odd number.
(d) 88 (e) None of these (a) 11 (b) 13 (c) 17
(d) 19 (e) 9
43. 7, 4, 5, 12, 52, ?
(a) 424 (b) 428 (c) 318 47. A can complete a task in 15 days B is 50% more
(d) 440 (e) None of these efficient than A. Both A and B started working together
on the task and after few days B left task and A finished
1
44. 6, 4, 5, 11, 39, ? the remaining 3 of the given work. For how many days
(a)159 (b) 169 (c) 189 A and B worked together.
(d)198 (e) None of these (a) 3 (b) 5 (c) 4
(d) 6 (e) 2

4 Adda247 Publications For any detail, mail us at


Publications@adda247.com
50+ Bank PO | Clerk Previous Year’s Papers 2016 – 2020
48. A boat can travel 9.6 km downstream in 36 min. If (a) 17 : 27 (b) 18 : 29 (c) 21 : 28
speed of the water current is 10% of the speed of the (d) 22 : 23 (e) 24 : 29
boat in downstream. How much time will boat take to
55. What is the difference between average of book sold by
travel 19.2 km upstream.
store A and E together and average books sold by store
(a) 2 hours (b) 3 hours (c) 1.25 hours
(d) 1.5 hours (e) 1 hour C and D together?
(a) 33 (b) 11 (c) 22
49. A started a business with a initial investment of Rs. (d) 44 (e) 20
1200. ‘X’ month after the start of business, B joined A
with on initial investment of Rs. 1500. If total profit Directions (56-60): In each of these questions, two
was 1950 at the end of year and B’s share of profit was equations (I) and (II) are given. You have to solve both the
750. Find ‘X’ equations and give answer
(a) 5 month (b) 6 month (c) 7 month (a) if x>y (b) if x≥y
(d) 8 month (e) 9 month (c) if x<y (d) if x ≤y
(e) if x = y or no relationship can be established.
50. Ratio between curved surface area and total surface
area of a circular cylinder is 3 : 5. If curved surface area 56. I. 𝑥 2 + 9𝑥 + 20 = 0 II. 𝑦 2 = 16
is 1848 cm3 then what is the height of cylinder. 57. I. 𝑥 2 − 7𝑥 + 12 = 0 II. 3𝑦 2 − 11𝑦 + 10 = 0
(a) 28 (b) 14 (c) 17
(d) 21 (e) 7 58. I. 𝑥 2 − 8𝑥 + 15 = 0 II. 𝑦 2 − 12𝑦 + 36 = 0

Directions (51-55): Given below is the pie chart which 59. I. 2𝑥 2 + 9𝑥 + 7 = 0 II. 𝑦 2 + 4𝑦 + 4 = 0
shows the percentage distribution of a book ‘XYZ’ 60. I. 2𝑥 2 + 15𝑥 + 28 = 0 II. 2𝑦 2 + 13𝑦 + 21 = 0
publishes in 5 different stores.
61. Train A completely crosses train B which is 205 m long
Total books = 550 in 16 second. If they are travelling in opposite direction
and sum of speed of both are 25 m/s. then find the
18% difference (in meter) between lengths of both trains.
22% (a) 5 (b) 6 (c) 8
(d) 10 (e) 12
12%
62. A trader mixes 14 kg rice of variety A which costs Rs.
60/kg with 18 kg of quantity of type B rice. He sells the
32% 16% mixture at Rs. 65/Kg and earns a profit of
100
%. Then
3
what was the cost price of type B rice.
(a) 30 (b) 20 (c) 40
51. If number of female who bought the books in store E (d) 50 (e) 45
are 21 more than number of males who bought books 63. Present age of A is 3 years less than present age of B.
from same store then find the number of females who Ratio of B’s age 5 year ago and A’s age 4 year hence is
bought book in store E. 3 : 4 then find present age (in years) of A.
(a) 75 (b) 78 (c) 71 (a) 20 (b) 17 (c) 23
(d) 68 (e) 73 (d) 26 (e) 29
52. Find the central angle for the book D. 64. A bag contains 6 Red, 5 Green and 4 Yellow coloured
(a) 117.5° (b) 115.2° (c) 112.8° balls. 2 balls are drawn at random after one another
(d) 108.5° (e) 118.8° without replacement then what is the probability that
53. If total books of another publisher ‘MNP’ is 20% more atleat one ball is Green.
2 4 3
than books of ‘XYZ’ publisher then what will be total (a) (b) (c)
3 5 8
books sold by store A and B for publisher ‘MNP’. 4 2
(d) 7 (e) 7
Percentage-distribution for different stores for MNP
remains same as for ‘XYZ’ 65. Cost price of B is 200 more than cost price of A. B is sold
(a) 200 (b) 178 (c) 181 at 10% profit and A is sold at 40% loss and selling price
(d) 186 (e) 198 of A and B are in the ratio 4 : 11. If A is sold at 20% loss
54. What is the ratio of total books sold by store A and C then what will be selling price of A.
together to the total books sold by store D and E (a) 320 (b) 400 (c) 240
(d) 160 (e) 360
together

5 Adda247 Publications For any detail, mail us at


Publications@adda247.com
50+ Bank PO | Clerk Previous Year’s Papers 2016 – 2020

Directions (66-70): Read the following table carefully and answer the following questions—
No. of students and % of students passed out of those who appeared are given for two subjects from year 2001 to 2005 in
a college XYZ.

66. Find the average number of students who were failed


70. Find the average number of students appeared in
in Economics in year 2002 and year 2003 together? Economics from year 2001 to 2004 together?
(a) 1435 (b) 1565 (c) 1720 (a) 3090 (b) 3015 (c) 3060
(d) 1590 (e) None of these (d) 3075 (e) 3850
Direction (71-75): What approximate value should come
67. Number of students failed in Statistics in the year 2003
in place of question mark (?) in the following questions?
is what % of the number of students failed in (Note: You are not expected to calculate the exact value)
Economics in the same year?
71. ? % of (5284.89 ÷ 7.08) = 986.01 – 533. 06
(a) 145.75% (b) 150% (c) 156.25%
(a) 42 (b) 39 (c) 74
(d) 158.25% (e) None of these (d) 65 (e) 60
68. Find the ratio between the total number of students 72. (1041.84 + ?) ÷ 3.02 = 1816.25 ÷ 4.01
appeared in Economics from 2002 to 2004 together (a) 442 (b) 337 (c) 385
and the total number of students appeared in Statistics (d) 268 (e) 320
from year 2003 to 2005 together? 73. 69.3% of 445.12 ÷ 14.06 = 623.08 ÷ ?
(a) 13 : 14 (b) 14 : 13 (c) 15 : 16 (a) 28 (b) 19 (c) 21
(d) 16 : 15 (e) None of these (d) 33 (e) 37
69. Find the difference between the total number of 74. ?2 + 114.09 – 24.06 × 5.14 = 163.19
students passed in Statistics from year 2002 and total (a) 7 (b) 13 (c) 11
number of students failed in Economics from year (d) 15 (e) 19
2005. 75. 768.16 ÷ 11.87 × √257 – 58.05 = ?
(a) 690 (b) 385 (c) 485 (a) 1033 (b) 1175 (c) 966
(d) 550 (e) 610 (d) 880 (e) 975
Directions (76-80): Study the following line graph carefully and answer the following questions.
Number of males and number of females are given. They are visiting a place from Monday to Friday.
Male Female
230

180

130

80
MON TUE WED THUS FRI

6 Adda247 Publications For any detail, mail us at


Publications@adda247.com
50+ Bank PO | Clerk Previous Year’s Papers 2016 – 2020

76. Find the ratio of the total number of males visited the (a) 30 (b) 60 (c) 40
place on Tuesday and Thursday together to the total (d) 50 (e) None of these
number of females visited the place on Monday and
79. If on Saturday the number of males and number of
Friday together?
females increased by 25% and 20% respectively as
(a) 29 : 30 (b) 30 : 29 (c) 25 : 26
compared to that on Friday then find the total number
(d) 26 : 25 (e) None of these
of males and females together visited the place on
77. Total number of males and females together visited the Saturday?
place on Tuesday are what percent more/less than the (a) 196 (b) 306 (c) 316
total number of male and females together visited the (d) 206 (e) 216
place on Thursday ?
12
(a) 26 13 %
3
(b) 25 13 %
3
(c) 26 13 % 80. Total number of males and females visited the place on
7 Monday and Tuesday together is how much more than
(d) 25 13 % (e) None of these
the total number of males and females visited the place
78. Find the difference between the total number of on Thursday and Friday together?
females visited the place from Monday to Wednesday (a) 175 (b) 125 (c) 150
and the total number of males visited the place from (d) 160 (e)130
Wednesday to Friday?

Solutions

REASONING ABILITY
1. (c); LP 12. (a); Only conclusion I follows.
2. (d); 13. (a); Only conclusion I follows.
14. (c); Either conclusion I or II follows.
15. (b);Only conclusion II follows.
3. (a); Race, Care Directions (16-20):
4. (c); Month Person
January C
February A
March G
5. (a); April E
June D
August F
October B
Directions (6-10):
16. (b); 17. (b); 18. (e);
Number Box
8 B 19. (d); 20. (e);
7 C Directions (21-25):
6 G
Word Code
5 F
4 A Card ja
3 H Time sa
2 D Win/team la/ta
1 E Fight da
6. (c); 7. (a); 8. (e); Game/Play pa/ra
9. (e); 10. (e); In fa
Directions (11-15): 21. (a); 22. (b); 23. (c);
11. (e); Both conclusion I and II follow. 24. (d); 25. (d);

7 Adda247 Publications For any detail, mail us at


Publications@adda247.com
50+ Bank PO | Clerk Previous Year’s Papers 2016 – 2020
Direction (26-30): 36. (d);

37. (e);
26. (a); 27. (e); 28. (e);
29. (b); 30. (e);
Direction (31-35):

38. (e);

39. (a);

31. (b); 32. (b); 33. (c);


40. (d);
34. (b); 35. (c);
Directions (36–40):

QUANTITATIVE APTITUDE
41. (c); Series is ÷2–1, ÷2–1 46. (a); Let a consecutive odd numbers
(22÷2)–1=10 = 𝑥 – 2, 𝑥 and 𝑥 + 2
42. (d); and consecutive even numbers
= 𝑦 – 2, 𝑦, 𝑦 + 2
So, 𝑦 – 2 = 9 + 𝑥 + 2
Adding prime No. 𝑦 – 𝑥 = 13 … (i)
77 + 11 = 88
and
43. (a); (7+1) × 0.5 = 4
(4+1) × 1 = 5 (𝑥)2 + 507 = (𝑦)2
(5+1) × 2 = 12 𝑦 2 − 𝑥 2 = 507
(12+1) × 4 =52 (𝑥 + 𝑦)(𝑦 − 𝑥) = 507
(52 +1)× 8 = 424 507
(𝑥 + 𝑦) = ⇒ 𝑥 + 𝑦 = 39 … (𝑖)
44. (c); (6×1)–2 = 4 13
(4×2)–3 = 5 Solving (i) and (ii) 𝑦 = 26 and 𝑥 = 13
(5×3) –4 = 11 so smallest odd numbers = 𝑥 – 2 = 13 – 2 = 11
(11×4) –5 = 39
(39×5) –6 = 189 47. (c); A complete work in 15 days.
45. (d); B will complete work in 10 days.
They together will complete whole work
15×10
= 25
= 6 𝑑𝑎𝑦𝑠
A and B together worked for = 6 × 2⁄3= 4 days
63 – 31 = 32
8 Adda247 Publications For any detail, mail us at
Publications@adda247.com
50+ Bank PO | Clerk Previous Year’s Papers 2016 – 2020
9.6 3𝑦(𝑦 − 2) − 5(𝑦 − 2) = 0
48. (d);𝑆𝑝𝑒𝑒𝑑 𝑜𝑓 𝑑𝑜𝑤𝑛𝑠𝑡𝑟𝑒𝑎𝑚 = 36 𝑘𝑚⁄𝑚𝑖𝑛
(3𝑦 − 5)(𝑦 − 2) = 0
= 16 𝑘𝑚/ℎ𝑟 5
Speed of current = 1.6 km/hr 𝑦 = 2,
3
Let speed of man in still water = 𝑥 ∴𝑥>𝑦
So, 𝑥 = 16 – 1.6 = 14.4 km/hr 58. (c); I. 𝑥 2 − 8𝑥 + 15 = 0
19.2
𝑅𝑒𝑞𝑢𝑖𝑟𝑒𝑑 𝑡𝑖𝑚𝑒 𝑖𝑛 𝑢𝑝𝑠𝑡𝑟𝑒𝑎𝑚 = 14.4−1.6 𝑥 2 − 3𝑥 − 5𝑥 + 15 = 0
= 1.5 ℎ𝑜𝑢𝑟𝑠 𝑥(𝑥 − 3) − 5(𝑥 − 3) = 0
(𝑥 − 3)(𝑥 − 5) = 0
49. (b);Ratio of profit of A and B = 1200 : 750 𝑥 = 3,5
= 24 : 15 = 8 : 5 II. 𝑦 2 − 12𝑦 + 36 = 0
So, 𝑦 2 − 6𝑦 − 6𝑦 + 36 = 0
1200×12 8
1500×𝑦
=5 𝑦(𝑦 − 6) − 6(𝑦 − 6) = 0
𝑦 = 6 months (𝑦 − 6)(𝑦 − 6) = 0
x = 6 month 𝑦=6
∴𝑥<𝑦
2𝜋𝑟ℎ 3
50. (d);2𝜋𝑟(𝑟+ℎ) = 5 59. (e); I. 2𝑥 2 + 9𝑥 + 7 = 0
5h = 3r + 3h 2𝑥 2 + 7𝑥 + 2𝑥 + 7 = 0
2h = 3r 𝑥(2𝑥 + 7) + 1(2𝑥 + 7) = 0
and (𝑥 + 1)(2𝑥 + 7) = 0
7
2πrh = 1848 𝑥 = −1, −
22 2 2
2 × × ℎ × ℎ = 1848 II. 𝑦 2 + 4𝑦 + 4 = 0
7 3
ℎ = 21 𝑦 2 + 2𝑦 + 2𝑦 + 4 = 0
𝑦(𝑦 + 2) + 2(𝑦 + 2) = 0
51. (c); Let male who purchased book from Store E = 𝑥 (𝑦 + 2)(𝑦 + 2) = 0
Then 𝑦 = −2, −2
22
𝑥 + 𝑥 + 21 = 100 × 550 ∴ No relation.
𝑥 = 50 60. (d);I. 2𝑥 2 + 15𝑥 + 28 = 0
Required number of females = 50 + 21 = 71 2𝑥 2 + 8𝑥 + 7𝑥 + 28 = 0
52. (b);
18
=
𝑥 2𝑥(𝑥 + 4) + 7(𝑥 + 4) = 0
5 32
18×32
(2𝑥 + 7)(𝑥 + 4) = 0
𝑥= = 18 × 6.4 = 115.2 7
𝑥 = (− ) , −4
5
2
53. (e); 𝑇𝑜𝑡𝑎𝑙 𝑏𝑜𝑜𝑘𝑠 𝑜𝑓 𝑠𝑡𝑜𝑟𝑒 𝑋𝑌𝑍 = 100 × 550
120 II. 2𝑦 2 + 13𝑦 + 21 = 0
2𝑦 2 + 7𝑦 + 6𝑦 + 21 = 0
= 660 𝑦(2𝑦 + 7) + 3(2𝑦 + 7) = 0
Total books sold by store A and B (𝑦 + 3)(2𝑦 + 7) = 0
= (18% + 12%) of 660 = 198 −7
𝑦 = −3, 2
54. (a); Required ratio = (18% + 16%) : (32% + 22%) 𝑥≤𝑦
= 34 : 54 = 17 : 27
61. (d);In 16 second distance covered by both
55. (c); Required difference = 16 × 25 = 400 m
1
= 2 [(32% + 16%) − (18% + 22%)]550 So length of A = 400 – 205 = 195
1
= 2 × 8% 𝑜𝑓 550 = 4% 𝑜𝑓 550 = 22 Required difference = 10 m
62. (c); Let cost price of mixture = 𝑦
56. (d);I 𝑥 2 + 5𝑥 + 4𝑥 + 20 = 0 4
𝑆𝑜, 𝑦 = 65
𝑥(𝑥 + 5) + 4(𝑥 + 5) = 0 3
(𝑥 + 4)(𝑥 + 5) = 0 𝑦 = 48.75
𝑥 = −4, −5 From mixture and allegation
II. 𝑦 2 = 16
𝑦 = ±4
∴𝑥 ≤𝑦
57. (a); I. 𝑥 2 − 7𝑥 + 12 = 0
𝑥 2 − 4𝑥 − 3𝑥 + 12 = 0
𝑥(𝑥 − 4) − 3(𝑥 − 4) = 0 7 48.75−𝑥
(𝑥 − 3)(𝑥 − 4) = 0 9
= 60−48.75
𝑥 = 3, 4 78.75 = 438.75 − 9𝑥
II. 3𝑦 2 − 11𝑦 + 10 = 0 360 = 9𝑥
3𝑦 2 − 6𝑦 − 5𝑦 + 10 = 0 𝑥 = 40 Rs./kg

9 Adda247 Publications For any detail, mail us at


Publications@adda247.com
50+ Bank PO | Clerk Previous Year’s Papers 2016 – 2020
63. (a); Let B’s age = 𝑥 50
= × 2200 = 1100
100
So A’s age = 𝑥 − 3
𝑥−5 3 Required difference = 1485 – 1100 = 385
𝑥+1
=4 Short trick = 55 × 27 – 50 × 22 = 385
𝑥 = 23
70. (e); Average no. of students appeared in Economics
A’s age = 23−3 = 20 𝑦𝑒𝑎𝑟𝑠
from year 2001 to 2004 together
64. (d);Probability that no ball is green 4200+3800+2600+4800 15400
= = = 3850
10𝐶 × 9𝐶 90 3 4 4
1 1
15×14
= 15×14 = 7 ?
3 4 71. (e); 100 × 750 = 450 ⇒ ? ≈ 60
Required probability = 1 − 7 = 7
(1042+?)
65. (a); Let C.P. of A = 𝑥 72. (e); 3.02
= 454 ⇒ ? = 320
So C.P. of B = 200 + 𝑥 310 625
73. (a); = ⇒ ? ≈ 28
According to question 14 ?
110
(𝑥+200)
100
=
11

𝑥+200
=
1 74. (b);?2 = 170 ⇒ ? ≈ 13
60
𝑥 4 6𝑥 4
100
75. (c); ≈ 64 × 16 – 58 ≈ 966
𝑥 = 400
If it is sold at 20% loss then selling price 76. (a); Total no. of males visited on Tuesday and
80
= 100 × 400 = 320 Thursday = 140 + 150 = 290
Total no. of females visited on Monday and Friday
66. (b);No. of students failed in Economics in year 2002 = 170 + 130 = 300
(100−45)
= × 3800 = 2090 Required ratio = 290: 300 = 29: 30
100
No. of students failed in Economics in year 2003 77. (a); Total no. of males and females together on Tuesday
(100−60)
= × 2600 = 1040 = 140 + 190 = 330
100
2090+1040 Total no. of males and females together on
𝑅𝑒𝑞𝑢𝑖𝑟𝑒𝑑 𝑎𝑣𝑒𝑟𝑎𝑔𝑒 = = 1565
2 Thursday = 150 + 110 = 260
55×38+40×26 330−260 12
𝑆ℎ𝑜𝑟𝑡 𝑡𝑟𝑖𝑐𝑘 = 2
= 1565 𝑅𝑒𝑞𝑢𝑖𝑟𝑒𝑑 % = 260 × 100 = 26 13 %
67. (c); No. of students failed in Statistics in year 2003 78. (d);Total no. of females visited from Monday to
100−35
= × 2500 = 1625 Wednesday = 170 + 190 + 140 = 500
100
No. of students failed in Economics in year 2003 Total no. of males visited from Wednesday to
100−60
= 100 × 2600 = 1040 Friday = 180 + 150 + 120 = 450
1625
Required difference = 500 – 450 = 50
𝑅𝑒𝑞𝑢𝑖𝑟𝑒𝑑 % = 1040 × 100 = 156.25%
65×25
79. (b);On Saturday —
𝑆ℎ𝑜𝑟𝑡 𝑡𝑟𝑖𝑐𝑘 = 40×26 × 100 = 156.25% Total no. of males visited the place
125
68. (d);Total no. of students appeared in Economics from = 100 × 120 = 150
2002 to 2004 Total no. of females visited the place
120
= 3800 + 2600 + 4800 = 11200 = 100 × 130 = 156
Total no. of students appeared in Statistics from Required males and females
2003 to 2005 = 150 + 156 = 306
= 2500 + 3200 + 4800 = 10500
Required ratio = 11,200 : 10,500 = 16 : 15 80. (c); Total males and females visited the place on
Monday and Tuesday together
69. (b);Total no. of students passed in Statistics in year = 160 + 140 + 170 + 190 = 660
2002 Total males and females visited the place on
55
= 100 × 2700 = 1485 Thursday and Friday together
Total no. of students failed in Economics in year = 150 + 120 + 110 + 130 = 510
2005 Required no. of persons = 660 – 510 = 150

10 Adda247 Publications For any detail, mail us at


Publications@adda247.com
50+ Bank PO | Clerk Previous Year’s Papers 2016 – 2020

1 Adda247 Publications For any detail, mail us at


Publications@adda247.com
50+ Bank PO | Clerk Previous Year’s Papers 2016 – 2020

Mock IBPS RRB PO Prelims 2016


34
REASONING ABILITY
Directions: (1–5): In these questions, a relationship 7. Statements: Some circles are triangles.
between different elements is shown in the statements(s). All triangles are squares.
The statements are followed by two conclusions. Give No square is a rectangle.
Conclusions:
answer I. Some triangles being rectangles is a possibility.
(a) if only conclusion I is true. II. All squares being circles is a possibility.
(b) if only conclusion II is true.
8. Statements: All pencils are Cutter
(c) if either conclusion I or II is true. Some Cutter are Scale.
(d) if neither conclusion I nor II is true. No Scale is a compass.
(d) if both conclusions I and II are true. Conclusions:
I. All compass being pencils is a possibility.
1. Statements : A > B  C < D, C = E > G II. At least some Cutters are pencils.
Conclusions : I. D > E II. B > E 9. Statements: Some wallets are bags.
Some bags are leather.
2. Statements : P  Q > M  N, Q = S
All purses are leather.
Conclusions : I. S > P II. N < S Conclusions: I. some purse are bags.
II. Some purse are wallet.
3. Statements : S > M = Z > T < Q > V
Conclusions : I. V = S II. Q > M 10. Statements: Some circles are triangles.
All triangles are squares.
4. Statements : T<U=VS>PQ No square is a rectangle.
Conclusions : I. S > T II. V  Q Conclusions:
I. No rectangle is a triangle.
5. Statements : M  N > R > W, E = J > L  W II. Some circles are not rectangles.
Conclusions : I. E > W II. M > L Directions (11-15): Study the following information to
answer the given questions
Directions (6-10): In each of the question-sets below are Eight students M, N, O, P , U, V, W and X are sitting around
three statements followed by two conclusions numbered I a square table in such a way four of them sit at four corners
and II. You have to take the given statements to be true while four sit in the middle of each of the four sides. The
even if they seem to be at variance with commonly known one who sit at the 4 corners face the centre and others
facing outside.
facts and then decide which of the given conclusions
M who faces the centre sits third to the left of V. U who faces
logically follows from the given statements, disregarding the centre is not an immediate neighbour of V. Only one
commonly known facts. Given answer. person sits between V and W. P sits second to right of N. N
(a) If only conclusions I follows. faces the centre. O is not an immediate neighbour of M.
(b) if only conclusion II follows. 11. Which one does not belong to that group out of five?
(c) if either conclusions I or conclusion II follows. (a)N (b)O (c)U
(d) if neither conclusion I nor conclusion II follows. (d)P (e)M
(e) if both conclusion I and conclusion II follow. 12. Which will come in the place of ?
NOU UXM MWP ?
6. Statements: All pencils are Cutter (a)PVN (b)PWM (c)POW
Some Cutter are Scale. (d)POV (e)None of these
No Scale is a campass. 13. What is the position of W with respect to O ?
Conclusions: (a)Third to the right (b)Second to the left
I. All pencils being Scale is a possibility. (c)Second to the right (d)Fourth to the right
II. No campass is a Cutter. (e)None of these

2 Adda247 Publications For any detail, mail us at


Publications@adda247.com
50+ Bank PO | Clerk Previous Year’s Papers 2016 – 2020
14. Who sits third to the left of N ? (c) None of the given options is true.
(a)X (b)M (c)W (d) Both R and P are immediate neighbors of W.
(d)V (e)None of these (e) V sits on the immediate right of W.
15. Which is true from the given arrangement ? 23. How many persons are seated between V and P?
(a)W faces the centre (b)N faces outside (a) None (b) One (c) Two
(c)X faces inside (d)M faces the centre (d) Four (e) Three
(e)None of these
24. In a certain code language SERIES is written as
Directions (16-18): Study the information carefully and QCGTGU. How is EXPERT written in that code
answer the question given below. language?
M is father of A and C. R is brother of C. A is Husband of T (a) VTGRZG (b) RPCRZG (c) GZRCPR
and S is daughter of T. V is grandmother of S. (d) RPCGZR (e) None of these
16. How is T related to M? 25. How many such pairs of letters are there in the word
(a) Son in law (b) Daughter COMPOSE each of which has many letters between
(c) Daughter in law (d) Can’t be determined them in the word as they have between them in the
(e) None of these English alphabetical series?
17. If R has only one sister C than what is the relation of A (a) None (b) One (c) Two
to S? (d) Three (e) None of these
(a) Mother (b) Father (c) Uncle Directions (26-30): Study the information carefully and
(d) Can’t be determined answer the question given below.
(e) None of these Nine persons P, Q, R, S, T, U, V, W and X. they live on a
18. How is M related to S? separate floor each of a 9-floor building but not necessarily
(a) Father (b) Father in law in the same order. The ground floor is numbered 1, the first
(c) Grandfather (d) Granddaughter floor is numbered 2 and so on until the topmost floor is
(e) None of these. numbered Nine.
Only two persons live below the floor on which V lives.
Directions (19-23): Study the following information Only one person lives between V and P.
carefully to answer the given questions: W lives on an odd-numbered floor but not on floor
Eight friends P, Q, R, S, T, U, V and W are seated in a straight no. 7.
line facing north, but not necessarily in the same order. Only two persons live between W and Q. X does not live on
• Q sits second to right of U. U sits at one of the extreme the topmost floor. P does not live on the lowermost floor. S
ends of the line. lives immediately below R but R does not sit on topmost
• Only three persons sit between Q and T. floor. Neither R nor T live on floor no 6. U lives immediately
• R sit third to the left of S. Only two persons sit between above P.
S and P.
• V is not an immediate neighbor of T. 26. How many persons live between the floors on which P
and S live?
19. Who among the following represents the person (a) Three (b) More than three
seated at the extreme right of the line? (c) None (d) Two (e) One
(a) V (b) W (c)U
(d) R (e) P 27. Who lives on the floor immediately below V?
(a) U (b) T (c) S
20. Who among the following sit exactly between S and P? (d) Q (e) X
(a) U, P (b) Q, U (c) U, V 28. On which of the following floor numbers does X live?
(d) T, W (e) Q, T (a) Four (b) One (c) Two
21. What is the position of V with respect to T? (d) Five (e) Seven
(a) Third to the left (b) Second to the right 29. Which of the following is true with respect to U as per
(c) Fourth to the right (d) Third to the right the given arrangement?
(e) Second to the left (a) Only three persons live between U and Q
22. Based on the given arrangement, which of the (b) Only three persons live above U.
following is true with respect to W? (c) Only one person sits between U and S.
(a) Only two persons sit between W and R. (d) U sits on odd numbered floor.
(b) Only two persons sit to the right of W. (e) None of these.

3 Adda247 Publications For any detail, mail us at


Publications@adda247.com
50+ Bank PO | Clerk Previous Year’s Papers 2016 – 2020
- There is one day gap between Chemistry class and
30. Who lives on the floor numbered 5?
Geography class. And biology class scheduled on
(a) U (b) Q (c) S
Sunday.
(d) P (e) None of these
34. How many days gap between Maths and Chemistry
Directions (31-33): Study the information carefully and
class?
answer the question given below.
(a) One (b) Two (c) Three
Mark starts from his house and moving in the south (d) Four (e) None
direction and after moving 25m, he took a right turn and
move 40 m to reach his uncle house. again Mark start 35. Hindi class is scheduled on which day?
moving southwards and after travelling 50m he took a left (a) Monday (b) Wednesday (c) Thursday
and travels 80 m to reach his aunt home. (d) Friday (e) None of these

31. In which direction his aunt house is located with 36. Which of the following is correct combination given
respect to his house? below?
(a) south west (b) south east (c) north east (a) Hindi= Monday
(d) north west (e) None of these (b) Physics= Tuesday
(c) Chemistry= Thursday
32. Uncle house in which direction with respect to aunt (d) Mathematics= Monday
house? (e) Biology= Friday
(a) North east (b) North west (c) South west
37. On which day of the week is Chemistry class is
(d) South east (e) None of these
schedule?
33. If Point A is 25m. to the north of uncle’s house then (a) Monday (b) Tuesday (c) Wednesday
what is the distance between A and Mark house? (d) Thursday (e) None of these.
(a) 40 m. (b) 30 m. (c) 20 m.
38. Four of the followings five are alike in a certain ways
(d) Can’t be determined
form a group which one does not belong to the group?
(e) None of these. (a) Tuesday=Hindi
Directions (34-38): Study the information carefully and (b) Monday=Chemistry
answer the question given below. (c) Friday=Physics
Gaurav Join classes from Monday to Sunday of the same (d) Wednesday=Hindi
week for different subject viz. Biology, Chemistry, Physics, (e) Thursday=English
Hindi, - Mathematics, English and Geography. 39. If Divyaraj finds that he is fourteenth from the left end
- Hindi class taken by him on Wednesday. of the row and 7th from the right end of the row, then
- There is one day gap between Hindi class and how many boys must be added to the row such that
Mathematics class. there are 30 boys in the row?
- And there is three day gap between mathematics class (a) 8 (b) 10 (c) 12
and English class. (d) 14 (e) None of these
- English class is scheduled immediately before Physics
40. Find odd one out from given series-
class but not in Monday.
AZD FUI HSK OLP SHV
- Chemistry is scheduled immediately after
(a) AZD (b) FUI (c) HSK
mathematics class.
(d) OLP (e) None of these

QUANTITATIVE APTITUDE
41. Two pipes can fill a tank in 10 hours and 16 hours
42. a, b, c and d are four consecutive even numbers, if the
respectively. A third pipe can empty the tank in 32
sum of ‘a’ and ‘c’ is 120, what is the product of ‘b’ and
hours. If all the three pipes are opened simultaneously
‘d’?
then in how much time the tank will be full? (in hours)
11 13 4 (a) 4030 (b) 3780 (c) 3900
(a) 7 21 (b) 7 21 (c) 8 21 (d) 3900 (e) 3840
5 9
(d) 6 14 (e) 8 14

4 Adda247 Publications For any detail, mail us at


Publications@adda247.com
50+ Bank PO | Clerk Previous Year’s Papers 2016 – 2020
43. Three numbers are given. The average of first and third (a) 12 days (b) 8 days (c) 14 days
numbers is 24 more than that of average of second and (d) 10 days (e) 9 days
third numbers. Find out the difference between the
first and second numbers. 45. A sum of money fetches Rs 240 as simple interest at the
(a) 36 (b) 40 (c) 42
(d) 48 (e) 46 rate of 5 p.c.p.a. after 6 years. What is the principal?
44. If 3 men or 9 boys can finish a piece of work in 21 days. (a) Rs 200 (b) Rs 400 (c) Rs 800
In how many days can 5 men and 6 boys can complete
the same piece of work? (d) Rs 1,200 (e) Rs 1,000

Directions (46– 50): Study the given table carefully and answer the questions.
Table shows the total population in six different cities and the ratio of literate to illiterate population and also the
percentage of graduate out of literate population in each city.
Cities Population (in thousand) Literate: Illiterate Percentage Graduate out of literate
A 22 5:6 20%
B 16 3:5 35%
C 96 2:1 32%
D 20 2:3 25%
1
E 24 5:3 33 3%
46. Graduate population of city B and D together is (a) 82% (b) 72% (c) 93%
approximately what percent more/less than graduate (d) 79% (e) 89%
population of city A and E together?
(a) 54% (b) 50% (c) 47% Directions (51-55): What will come in the place of the
(d) 42% (e) 37% question mark (?) in the following number series?
47. Population of city C who are literate but not graduate 51. 1, 11, 59, 239, 719, ?
is how much more than the average graduate (a) 1438 (b) 1439 (c) 1428
population of city D and E together? (d) 1429 (e) 1419
(a) 40020 (b) 4020 (c) 4200 52. 18, 8, 30, 20, 42, ?
(d) 4420 (e) 40040 (a) 38 (b) 36 (c) 28
48. If the ratio of illiterate male to female in city B is 3:5 (d) 32 (e) 30
and ratio of graduate male to female population in city
D is 2 : 3. Then find the ratio of total illiterate male in 53. 2880, 480, 96, ?, 8, 4
city B and graduate female in city D? (a) 16 (b) 24 (c) 20
(a) 23 : 7 (b) 8 : 25 (c) 75 : 16 (d) 28 (e) 32
(d) 21 : 8 (e) 25 : 8 54. 8, 10, 20, 50, ? , 248
49. Illiterate population of city D is what percent of the (a) 115 (b) 103 (c) 113
Illiterate population of city ‘C’? (d) 108 (e) 118
(a) 25% (b) 37.5% (c) 40%
(d) 50% (e) 62.5% 55. 8, 6, 8, 14, 30, ?
50. Literate population of cities A and B together is (a) 75 (b) 76 (c) 77
approximately what percentage of the population (d) 78 (e) 79
which are not graduate of city D?
Directions (56-60): A Company produces three different products namely food, drinks and cosmetic products. If total
production of the company was same for all years and % production of three products in particular years given below,then
answer the questions that follows:

5 Adda247 Publications For any detail, mail us at


Publications@adda247.com
50+ Bank PO | Clerk Previous Year’s Papers 2016 – 2020
56. In 2013, number of food products produced by the 62. I. 3𝑥 2 + 14𝑥 + 15 = 0 II. 3𝑦 2 − 1 3y + 14=0
company is what percent more/less than cosmetic
63. I. 12𝑥 2 − 17𝑥 + 6 = 0 II. 𝑦 2 − 16𝑦 + 63 = 0
products produced in year 2016?
1 2
(a) 33 3 % (b) 25% (c) 66 3 % 64. I. 𝑥 2 − 48𝑥 + 575 = 0 II. 46𝑦 2 − 35y-11=0
(d) 20% (e) 50% 65. I. 15𝑥 2 − 11𝑥 − 12 = 0 II. 20𝑦 2 −49y + 30=0
57. If total production in year 2017 was 1,20,000. Find the 66. Three friends Satish, Bhavya and Abhi complete the
difference between number of food products produced work in 10 days, 15 days & 12 days respectively. They
in 2017 and drink products produced in 2014? started to work together but Satish left the work after
(a) 12000 (b) 15000 (c) 12500 two days and Abhi left the work 1 day before the
(d) 10000 (e) 11500 completion of the work. In how many days the whole
58. Find the ratio b/w number of cosmetic products work will be completed?
8 7
produced in 2017 and number of food products (a) 5 9 days (b) 6 days (c) 78 days
produced in 2013. (d) 8 days (e) 9 days
(a) 1 : 4 (b) 1 : 2 (c) 2 : 1 2rd
(d) 3 : 4 (e) 4 : 1 67. 3
of first number is equal to the cube of the second
number. If the second number is equal to 12% of 100,
59. The difference b/w food products and drink products
what is sum of the first & 2nd number?
produced by the company in 2015 is 15000. Find the
(a) 2408 (b) 2640 (c) 2426
average of food and cosmetic products produced by
(d) 2604 (e) 2804
company in 2013?
68. A wholeseller sells an item to a retailer at 20%
(a) 30000 (b) 50000 (c) 40000
discount, but charges 10% on the discounted price for
(d) 45000 (e) 25000
packaging & delivery. The retailer sells it for 1023
60. Find the total production in 2018 if there is an increase more, thereby earning a profit of 25%. At what price
of 10% in total production in 2018 as compared to had the wholeseller marked the item ?
previous year given that number of drink products (a) Rs. 4620 (b) Rs. 4650 (c) Rs. 4850
produced in 2015 was 12000? (d) Rs. 5240 (e) Rs. 5445
(a) 55000 (b) 44000 (c) 66000 69. The present age of Bhagat and Abhi are in ratio of
(d) 33000 (e) None of these 9 : 8 respectively. After 10 years the ratio of their ages
Directions (61-65): In each of these questions, two will be 10 : 9. What is the difference in their present
equations I and II are given. You have to solve both the age?
equations and give answer (a) 8 years (b) 6 years (c) 12 years
(a) if 𝑥 > 𝑦 (b) if 𝑥 ≥ 𝑦 (d) 4 years (e) 10 years
(c) if 𝑥 < 𝑦 (d) if 𝑥 ≤ 𝑦 70. The circumference of two circles is 132 m and 176 m
(e) if 𝑥 = 𝑦 or no relation can be established between 𝑥 respectively. What is difference between the area of
and 𝑦 larger circle and smaller circle ? (in m²)
(a) 1052 (b) 1128 (c) 1258
61. I. 𝑥 2 − 264 = 361 II. 𝑦 3 − 878 = 453 (d) 1078 (e) 1528

Directions (71-75): Study the given line graph carefully and answer the questions.
Line graph shows the percentage of chair sold by six shopkeepers.
Total chair sold by all shopkeepers = 96 thousands.
35
Percentage of chair’s sold

30
25
20
15
10
5
0
A B C D E F
shopkeeper

6 Adda247 Publications For any detail, mail us at


Publications@adda247.com
50+ Bank PO | Clerk Previous Year’s Papers 2016 – 2020
71. Chairs sold by shopkeeper B and D together is how 75. What is the ratio of average of chairs sold by
much more than chairs sold by shopkeeper A and F shopkeeper B, C and D together to average of chairs
together ? sold by shopkeeper A and E together?
(a) 10420 (b) 11520 (c) 12480 (a) 25 : 33 (b) 21 : 11 (c) 26 : 33
(d) 11740 (e) 15220 (d) 11 : 24 (e) 11 : 26
72. Chairs sold by shopkeeper A and E together is how Directions (76-80): What should come in place of
much percentage more than chairs sold by shopkeeper question mark (?) in the following questions?
B and C together?
(a) 10% (b) 6% (c) 8% 76. 1528 + 525 ÷ 25 – 840 = 510 + ?
(d) 12% (e) 14% (a) 199 (b) 299 (c) 159
(d) 189 (e) 165
73. F sold only three types of chairs i.e. K, L and M in the
ratio 3 : 5 : 4 .Find the difference of chairs sold by F of 77. √1225 ÷ 7 + 18.5 × 16 – 18% of 10800 = ? – 1800
type K and M together and that of type L? (a) 259 (b) 169 (c) 157
(a) 320 (b) 840 (c) 740 (d) 129 (e) 141
(d) 420 (e) 640
78. 65% of 180 + ?% of 210 = 80% of 225
74. If there is another shopkeeper P who sells three types (a) 45 (b) 30 (c) 40
of chairs i.e. X, Y and Z. If chairs of type X sold is half of
(d) 50 (e) 25
the total chairs sold by shopkeeper F, Chairs of type Y
sold is 20% of the chairs sold by shopkeeper A and 79. √1500+? +17.5 × 8 – 5% 𝑜𝑓 20 = 42
2
chairs of type Z sold is 5 th of total chairs sold by (a) 145 (b) 115 (c) 120
shopkeeper B. Then find total number of chairs sold by (d) 135 (e) 125
Shopkeeper P? 13 8
(a) 12348 (b) 16368 (c) 12244 80. 17
of 156 of 153 = ?
(d) 10368 (e) 10428 (a) 8 (b) 12 (c) 7
(d) 6 (e) 4

Solutions
REASONING ABILITY
1. (a); 𝐷 > 𝐶 = 𝐸(𝑇𝑟𝑢𝑒)𝐵 ≥ 𝐶 = 𝐸(𝐹𝑎𝑙𝑠𝑒) 8. (e);

2. (b);𝑆 = 𝑄 ≥ 𝑃(𝐹𝑎𝑙𝑠𝑒)𝑆 = 𝑄 > 𝑀 ≥ 𝑁(𝑇𝑟𝑢𝑒)

3. (d);𝑉 = 𝑆(𝐹𝑎𝑙𝑠𝑒)𝑄 > 𝑀(𝐹𝑎𝑙𝑠𝑒)


9. (d);
4. (a); 𝑆 ≥ 𝑉 = 𝑈 > 𝑇(𝑇𝑟𝑢𝑒)𝑉 ≥ 𝑄(𝐹𝑎𝑙𝑠𝑒)

5. (a); 𝐸 = 𝐽 > 𝐿 ≥ 𝑊(𝑇𝑟𝑢𝑒)𝑀 ≥ 𝑁 > 𝑅 > 𝑊 ≤


𝐿(𝐹𝑎𝑙𝑠𝑒) 10. (e);

6. (a);

Direction (11-15);

7. (b);

7 Adda247 Publications For any detail, mail us at


Publications@adda247.com
50+ Bank PO | Clerk Previous Year’s Papers 2016 – 2020
11. (b); 12. (a); 13. (d); 6 U
14. (a); 15. (d); 5 P
Direction (16-18); 4 Q
3 V
2 X
1 W

26. (e); 27. (e); 28. (c);


16. (c); 17. (b); 18. (c); 29. (b); 30. (d);
Direction (19-23); Direction (31-33);

19. (e); 20. (d); 21. (a);


22. (b); 23. (e);

24. (b);

31. (b); 32. (b); 33. (a);


Direction (34-38);
Day Subjects
25. (d); Monday Mathematics
Tuesday Chemistry
Wednesday Hindi
Thursday Geography
Direction (26-30); Friday English
Saturday Physics
Floor Persons Sunday Biology
9 T 34. (e); 35. (b); 36. (d);
8 R 37. (b); 38. (d); 39. (b);
7 S 40. (d);

QUANTITATIVE APTITUDE

41. (b);Part of the tank filled in 1 hour 43. (d);Let the numbers be a, b, and c respectively.
1 1 1 16+10−5 21 𝑎+𝑐 𝑏+𝑐
= 10 + 16 − 32 = = 160 ∴ 2 − 2 = 24
160
160 13 ⇒ (a +c) – (b + c) = 24 × 2 = 48
∴ Tank will be filled in = 7 hours
21 21 ⇒ a –b = 48
42. (e); ∵ a, b, c and d are four consecutive numbers and a 44. (e); ∵ 3 men = 9 boys
+ c = 120 ∴ 1 man = 3 boys
∴ a +a+4 = 120 ∴ 5 men + 6 boys
⇒ 2𝑎 = 116 ⇒ 𝑎 = 58 = (5 × 3 + 6) boys = 21 boys
∴ M1D1= M2D2
∴ b = 60 and d= 64
= 9 × 21 = 21 × D2
∴ b × d = 60 × 64 = 3840 9×21
= D2= 21 = 9 days

8 Adda247 Publications For any detail, mail us at


Publications@adda247.com
50+ Bank PO | Clerk Previous Year’s Papers 2016 – 2020
𝑆𝐼×100
45. (c); Principal = 𝑇𝑖𝑚𝑒×𝑅𝑎𝑡𝑒 53. (b);
240×100
∴ = 𝑅𝑠 800
5×6

46. (d);Graduate population of city A and E together


5 20 5
= 22000 × 11 × 100 + 24000 × 8 × 3
1 54. (e);
= 2000 + 5000 = 7000
Graduate population of city B and D together
3 35 2 25
= 16000 × 8 × 100 + 20000 × 5 × 100
= 2100 + 2000 = 4100 55. (c);
7000−4100
Required percentage = × 100
7000
2900
= 7000 × 100 ≈ 42%

47. (a); Population who are literate but not graduate of city 56. (c); Let total production of the company be x
(.30x−0.10x)
C ∴ Required percent = × 100
2 68 0.30x
= 96000 × 3 × 100 2
= × 100 = 66 % less
2
3 3
= 43520
Average graduate population of city D & E together 57. (a); Required difference = 30% of 1,20,000 – 20% of
1 2 25 5 1
= 2 [20000 × 5 × 100 + 24000 × 8 × 3] 1,20,000 = 12000
1
= [2000 + 5000] = 3500 58. (e); Let total production be x
2
40% of x
∴ Required difference = 43520 – 3500 Required ratio = 10% of x = 4 ∶ 1
= 40020
59. (d);Let total production be x
48. (e); Illiterate male in city B
5 3 ATQ,
= 16000 × 8 × 8 = 3750
10% of x = 15000
Graduate female in city D x
2 25 3 10
= 15000
= 20000 × × ×
5 100 5 x = 1,50,000
= 1200
3750 Required average
Required ratio = 1200 = 25 ∶ 8 10% of 1,50,000+50% of 1,50,000
=
2
49. (b);Illiterate Population in City D 15000+75000
3 = = 45000
= 20,000 × = 12000 2
5
Illiterate Population in City C 60. (b); Let total production of each previous years
1
= 96,000 × = 32000 be x
3 30
12000
Required % = 32000 × 100 = 37.5% ∴ 100 x = 12000 ⇒ x = 40000
110
Total production in 2018 = 100 × 40000
50. (e); 𝑅𝑒𝑞𝑢𝑖𝑟𝑒𝑑 𝑝𝑒𝑟𝑐𝑒𝑛𝑡𝑎𝑔𝑒
5 3 = 44000.
22,000× +16,000×
= 11 8
× 100
3 2 75
20,000× +20,000× ×
5 5 100
61. (e);
10,000+6,000 1600 𝑰. 𝑥 2 − 264 = 361 𝑰𝑰. 𝑦 3 − 878 = 453
= 12,000+6,000 × 100 = 18
≈ 89% 2
𝑜𝑟, 𝑥 = 361 + 264| 𝑜𝑟, 𝑦 3 = 453 + 878
∴ 𝑥 2 = 625 𝑜𝑟, 𝑦 3 = 1331
51. (b); ∴ 𝑥 = √625 = ±25 3
∴ 𝑦 = √1331 = 11
Hence no relation can be established.
62. (c);
52. (d);

Hence 𝑥 < 𝑦
9 Adda247 Publications For any detail, mail us at
Publications@adda247.com
50+ Bank PO | Clerk Previous Year’s Papers 2016 – 2020
63. (c); After 10 years.
9𝑥+10 10
=
8𝑥+10 9
81x + 90 = 80x + 100
x = 10
∴ required difference = 10 years.
Hence 𝑥 < 𝑦
70. (d);Let radius of smaller & larger circles be r₁ & r₂
64. (a); respectively.
2π r₁ = 132
r₁ = 21 m
2π r₂ = 176 ⇒ r₂ = 28 m.
∴ Required difference
22
Hence 𝑥 > 𝑦 = 𝜋(𝑟22 – 𝑟12 ) = 7 × 49 × 7 = 1078 m²
65. (e); 71. (b);Required difference
= [(16 + 12)%– (12 + 4)%] × 96000
12
= 100 × 96000 = 11520
72. (a); Required percentage
(12+32) –(16+24)
= × 100
No relation (16+24)
4
66. (a); = × 10 = 10%
40
73. (e); Total chairs sold by shopkeeper F
4
= 100 × 96000 = 3840
Required difference
( 7 –5)
= 12 × 3840 = 640
(Satish+ Bhavya+ Abhi) 2 days work = 15 × 2 = 30 74. (d);Total chairs sold by Shopkeeper P
unit 1 1 2
= [ × 4 + × 12 + × 16] ×
96000
Bhavya 1 day work = 4 unit 2 5 5 100
∴ Whole work will be completed = 10368
26 8 16+24+12
= 2+ 9 +1 = 2+29 +1
8
75. (c); 𝑅𝑒𝑞𝑢𝑖𝑟𝑒𝑑 𝑟𝑎𝑡𝑖𝑜 = 3
12+32
= 5 𝑑𝑎𝑦𝑠 52×2
2
9
= 3×44 = 26 ∶ 33
100×12
67. (d);𝑆𝑒𝑐𝑜𝑛𝑑 𝑛𝑜. = = 12
100 76. (a); 1528 + 21 – 840 – 510 = ?
3 3 3
∴ 𝑓𝑖𝑟𝑠𝑡 𝑛𝑜. = 12 × 2 = 1728 × 2 ? = 1549 – 1350
= 2592 ? = 199
∴ Required sum = 12 + 2592 = 2604 35
68. (b);Let the price marked by whole seller be Rs. x 77. (c); 7
+ 296 – 1944 = ? – 1800
80 301 + 1800 – 1944 = ?
∴ 𝑆. 𝑃. 𝑜𝑓 𝑎𝑟𝑡𝑖𝑐𝑙𝑒 𝑓𝑜𝑟 𝑤ℎ𝑜𝑙𝑒 𝑠𝑒𝑙𝑙𝑒𝑟 = 𝑥 × ×
100
110 22𝑥 ? = 157.
= = 𝐶. 𝑃 𝑜𝑓 𝑎𝑟𝑡𝑖𝑐𝑙𝑒 𝑓𝑜𝑟 𝑟𝑒𝑡𝑎𝑖𝑙𝑒𝑟
100 25 65 ? 80
22𝑥 125 11𝑥 78. (b);100 × 180 + 100 × 210 = 100 × 225
𝑆. 𝑃. 𝑜𝑓 𝑎𝑟𝑡𝑖𝑐𝑙𝑒 𝑓𝑜𝑟 𝑟𝑒𝑡𝑎𝑖𝑙𝑒𝑟 = 25
× 100 = 10 ?
ATQ, × 210 = 180 – 117
100
11𝑥 22𝑥 63×100
10
– 25 = 1023 ?= = 30
210
55𝑥 –44𝑥
50
= 1023 79. (e); 1500 + 140 – 1 + ? = 1764
11x = 1023 × 50 ? = 1764 – 1639
⇒ x = Rs. 4650 ? = 125
69. (e); Let present age of Bhagat & Abhi be 9x and 8x 13 8
80. (d);17 × 156 × 153 = ? ⇒ ? = 6
respectively

10 Adda247 Publications For any detail, mail us at


Publications@adda247.com
50+ Bank PO | Clerk Previous Year’s Papers 2016 – 2020

1 Adda247 Publications For any detail, mail us at


Publications@adda247.com
50+ Bank PO | Clerk Previous Year’s Papers 2016 – 2020

Mock IBPS RRB PO Mains 2019


35
REASONING ABILITY

Directions (1-5): Study the following information to 5. The number of persons were born between Kamal
answer the questions given below: and Pranav is same as the number of persons were
Seven persons were born in seven different years. Their born between Swati and ___?
ages are calculated with respect to 2019. None of them was (a) Pooja (b) Dheeraj (c) Aditi
born before 1965. They belong to different professions (d) Nisha (e) None of these
such as Manager, HR, Artist, Doctor, Teacher, Engineer, and 6. Statement I- The Indian Army plans to recruit
Pilot but not necessarily in the same order. women in combat roles, especially with regard to the
Note: All the persons were born on the same date and same operations in Jammu & Kashmir, Army Chief General
month. Bipin Rawat said at the Passing Out Parade at the
Nisha was 38 years old. No one was born between Nisha Indian Military Academy (IMA).
and the one who is Pilot. Three persons were born between Statement II- Many a time jawans in Kashmir feel
the one who is Pilot and Aditi, who is the oldest among all
hesitant about dealing with women when they are in
and born before 1970 but in the even-numbered year. No
the front lines.
one was born between Aditi and the one who is a Manager.
Statement III- The situation in Kashmir has been
More than one person was born between the ones who are
Manager and Pilot. Dheeraj was born before Nisha but not volatile for decades, flaring up for worse since last
just before. There is 4 years difference between Aditi and summer. In the last four days alone, the army has
Dheeraj. The difference between the ages of Dheeraj and killed 13 suspected militants.
Kamal is the same as between Nisha and Swati. Pranav who Which of the following may be cause/effect among the
was born before Pooja but after Swati. Swati was 11 years above statements?
older than Pranav. Pranav was born in 1998. The (a) Statement II will be cause and Statement I and III
difference between the ages of Kamal and Pranav was not will be it’s effects.
more than 20 years. The one who is a Teacher was born (b) Statement II and III will be cause and I will be it’s
just before HR. Pooja does not an HR. There are 7 years of effect.
difference between the ones who are Doctor and Engineer. (c) Statement III is cause and Statement I is it’s effect
No one was born between Pilot and Doctor. and Statement II is effect of independent cause.
(d) Statement II is cause and Statement I is it’s effect
1. How many persons were born between the ones who
were Artist and Doctor? and Statement III is effect of independent cause.
(a) One (b) None (c) Two (e) Statement III is cause and Statement II is it’s
(d) Five (e) None of these effect and Statement I is effect of independent
cause.
2. In which year Kamal was born?
(a) 1976 (b) 1978 (c) 1981 7. Statement: No one in the Royal Family wants to be
(d) 1987 (e) None of these king or queen by their choice, Prince Harry has told a
US magazine, adding that "we will carry out our duties
3. Which of the following combination is true? at the right time". "Is there any one of the Royal
(a) Nisha-41-HR Family who wants to be king or queen? I don't think
(b) Dheeraj-41-Manager
so," he told Newsweek.
(c) Aditi-51-Doctor
Which of the following is not in line with the above
(d) Swati-32-Piolt
statement?
(e) None id true
(I) He said the royals were doing their duties "for
4. Four of the following five are alike in a certain way the greater good of the people".
and hence they form a group. Which one of the (II) Harry, who recently said he had received
following does not belong to that group? counselling to cope with the death of his mother
(a) Aditi-Artist in a car crash in Paris, said: "My mother had just
(b) Kamal-Manager died, and I had to walk a long way behind her
(c) Nisha-HR coffin, surrounded by thousands of people
(d) Swati-Pilot watching me while millions more did on
(e) Pooja-Engineer television.
2 Adda247 Publications For any detail, mail us at
Publications@adda247.com
50+ Bank PO | Clerk Previous Year’s Papers 2016 – 2020

(III) He also paid tribute to his "remarkable" Directions (11-15): Study the following information
grandmother for letting the younger royals do carefully and answer the questions given below:
things their own way. Eight persons are sitting in a straight line equidistant from
(a) Only (I) each other. Some of them are facing north and some of
(b) Only (II) them are facing south. Not more than two persons having
(c) Only (III) same direction are sitting adjacent to each other. Three
(d) Both (II) and (III) persons are sitting to the left of U. Only one person sits
(e) None of these between Z and W and both of them faces in the same
Direction (8-10): Study the following information direction. Z does not sit at the extreme end of the line. Y
carefully and answer the questions given below: who is not an immediate neighbor of S sits second to the
right of X who sits at one of the extreme ends of the line. V
In each of the questions below are given some statements sits fifth to the left of T who faces in the south direction. S
followed by some conclusions. You have to take the given sits immediate right of U and both faces in opposite
statements to be true even if they seem to be at variance directions to each other. Persons sitting at the extreme
with commonly known facts. Read all the conclusions and ends are facing in the same direction. Y faces north
then decide which of the given conclusions logically direction.
follows from the given statements disregarding commonly
11. How many persons are facing in the south direction?
known facts.
(a) Four (b) Five (c) Three
8. Statements: Only a few Pizza are Burger. Only a few (d) Two (e) Six
Burger are Maggie. All Maggie are Momo’s. No Pasta is 12. Who sits second to the left of S?
Maggie. (a) T (b) V (c) X
Conclusions: (d) Z (e) None of these
I. Some Maggie are not Burger.
II. Some Pizza are not Pasta. 13. Four of the following five are alike in a certain way
III. Only a few Pizza are Maggie. and hence form a group, which of the following does
(a) If only conclusion I and II follows not belong to the group?
(b) If only conclusion II follows (a) S (b) U (c) Z
(c) If either conclusion II or III and I follows (d) W (e) X
(d) None follows 14. How many persons sit between Z and Y?
(e) None of these (a) One (b) None (c) Three
(d) Two (e) More than three
9. Statements: Only a few Bank are Account. Few
Current are Saving. All Account are Saving. Some FD 15. Which of the following is true?
are not Account. (a) No one sits to the left of W
Conclusions: (b) U sits second to the right of T.
I. Some Account can be FD. (c) Z is facing in north direction
II. Some Saving are not FD. (d) Both (b) and (c) are true
III. All Bank being Account is a possibility. (e) All are true
(a) If only conclusion I and II follows Directions (16-20): Each of the questions below consists
(b) If only conclusion II and III follows of a question and two statements numbered I and II given
(c) If only conclusion I follows below it. You have to decide whether the data provided in
(d) All I, II and III follow the statements are sufficient to answer the question. Read
(e) None Follow both the statements and give answer.
10. Statements: Only Vertical is Circle. Some Vertical are (a) if the data in statement I alone are sufficient to answer
not Square. All Square are Triangle. the question, while the data in statement II alone are
Conclusions: not sufficient to answer the question.
I. Some Circle can be Triangle. (b) if the data in statement II alone are sufficient to
answer the question, while the data in statement I
II. All Square being Vertical is a possibility.
alone are not sufficient to answer the question.
III. Only a few Vertical is Triangle.
(c) if the data either in statement I alone or in statement
(a) If only conclusion I and II follows
II alone are sufficient to answer the question.
(b) Only II
(d) if the data given in both statements I and II together
(c) If only conclusion I follows
are not sufficient to answer the question.
(d) All I, II and III follow (e) if the data in both statements I and II together are
(e) None Follow necessary to answer the question.

3 Adda247 Publications For any detail, mail us at


Publications@adda247.com
50+ Bank PO | Clerk Previous Year’s Papers 2016 – 2020

16. Five members i.e. P, Q, R, S and T are in a family, then Directions (24-28): Study the information carefully and
who is the husband of R? answer the questions given below.
Statement I: R is sister in law of S who is the child of A certain number of persons are sitting in a row facing
P. north. M sits fourth to the left of O. Only two persons sit
Statement II: R is the daughter in law of Q who is the between M and Q. R sits second to the left of Q. S sits fourth
mother of S. to the right of O. Not more than five persons sit between P
17. What is the code for “cow”? and Q. P sits to the right of S. Only three persons sit to the
Statement I: “big cow rat” is coded as “mn vg dc” and left of R.
“rat dew grass” is coded as “vg bh sx” 24. Which of the following statement is true?
Statement II: “big same use” is coded as “mn we qs” (a) M sits at an extreme end
and “rat cow use” is coded as “we vg dc” (b) Only one person sits between M and R
(c) All are true
18. Six persons are of different heights. Who is 2nd
(d) Ten persons sits between M and S
shortest person?
(e) None is true
Statement I: Anshul is taller than Ankita and shorter
than Anamika. Only one person is taller than Akash. 25. How many persons are sitting between O and Q?
Statement II: Ankita is shorter than Akash and taller (a) One (b) Two (c) None
than Aditya. Ankit is not shortest person. (d) Three (e) More than three

19. Five persons i.e. F, P, R, S and T are living in different 26. If P sits at an extreme end of the row, then how many
floors of five floored building. Who is living in topmost persons are sitting in the row?
floor? (a) Ten (b) Twelve (c) Thirteen
Statement I: There are two floor gaps between P and (d) Eleven (e) More than thirteen
F who is immediate above R. S is in bottommost floor. 27. What is the position of S in the row?
Statement II: Only one person is above R who is just (a) Immediate right of P
above S. There are three floors gap between P and F (b) Sixth to the right of Q
who is not in topmost floor. (c) Eighth to the left of M
(d) None of these
20. Five persons i.e. A, B, C, D and G are sitting in a row
(e) Seventh to the right of R
facing North. Who sits in the middle of the row?
Statement I: D sits 3rd to the right of G who is at an 28. If L sits exactly between S and O, then how many
extreme end. Two persons sit between A and B who is persons sits to the left of L?
an immediate neighbor of D. (a) Ten
Statement II: B sits immediate right of D who sits 3rd (b) Nine
to the left of A. C sits immediate right of B. (c) Eight
(d) Cannot be determined
Directions (21-23): In each of the questions, relationships (e) More than ten
between some elements are shown in the statements.
29. India and the UAE have discussed measures to deepen
These statements are followed by conclusions numbered I
cooperation in a range of areas including defence,
and II. Read the statements and give the answer.
security, terrorism and trade as the two countries
(a) If only conclusion I follows.
held the second round of their strategic dialogue in
(b) If only conclusion II follows. Abu Dhabi.
(c) If either conclusion I or II follows. Which among the following is not-in-line with given
(d) If neither conclusion I nor II follows. statement?
(e) If both conclusions I and II follow. (I) The Indian delegation at the talks held yesterday
21. Statements: A≤B>D=C, C>E≤F was led by Minister of State for External Affairs
Conclusion: I. A>C II. E<B M J Akbar while the UAE side was headed by its
Minister of State for Foreign Affairs Anwar
22. Statements: P>Q>R=S, S≥T≤U≥V Gargash.
Conclusions: I. Q>T II. S≤V (II) India ships wheat to Afghanistan via Chabahar.
23. Statements: B>C>D<E=F, H≤E>G=N India on Sunday began shipment of wheat to
Afghanistan through the Iranian port of
Conclusions: I. F<N II. N>D
Chabahar.
4 Adda247 Publications For any detail, mail us at
Publications@adda247.com
50+ Bank PO | Clerk Previous Year’s Papers 2016 – 2020

(III) The UAE has raised its investment profile in Direction (34-38): Study the following information
India, and cooperation has increased on security carefully and answer the questions given below:
issues, the statement said. Eight boxes AC, BD, EG, FH, IK, JL, MO and NP have different
(a) Only I brands toffee i.e. Poppins, Melody, Kismi bar, Gems,
(b) Both II and III Hajmola, Boomer, Center fresh and Coffee bite places one
(c) Both I and III above another. All are not necessarily in same order.
(d) Only II Box FH is placed above the box which has Center fresh.
(e) None of these There are three boxes are between the boxes have Melody
and Hajmola. There are only three boxes are above the box
30. Modi government to reframe social welfare schemes MO. There is only one box is between MO and FH. Box AC
for OBCs. In the run-up to the high stakes elections in is immediate below the box which has Melody. Box BD
Gujarat and Himachal Pradesh, the Narendra Modi- places just above box which has Melody and just below the
led government has moved to reframe social welfare box has Gems. Box FH neither has Gems nor has Hajmola.
schemes for other backward classes (OBCs). Box EG places just above the box which has Poppins and
Which among the following can be hypothesized from just below the box which has Coffee bite. There is one box
the given statement? between box IK which has Boomer and box JL. Box MO
(I) The targeted schemes promise quality doesn’t have Center fresh.
residential accommodation and scholarships to 34. Which among the following box places just above the
OBC students. box which has Kismi bar?
(II) This reframe of social welfare schemes for OBCs (a) AC (b) BD (c) EG
by the Modi government is a political move to (d) FH (e) None of these
target high stakes elections in Gujarat and 35. Which among the following toffee is in box NP?
Himachal Pradesh. (a) Poppins (b) Melody (c) Kismi bar
(III) The ministry has laid down guidelines for (d) Gems (e) None of these
construction of hostels, which would be energy 36. How many boxes are placed between box AC and the
efficient, economical yet good quality. box which have Hajmola?
(a) Only (I) (a) One (b) Two (c) Three
(b) Both (III) and (I) (d) More than three (e) None
(c) Only (II) 37. Which among the following is not false?
(d) Both (II) and (III) (a) AC-Hajmola (b) BD-Gems (c) EG-Melody
(e) None of these (d) FH- Coffee bite (e) None is true
Directions (31-33): Study the following information 38. Four of the following five are alike in certain way
carefully and answer the questions given below. based from a group, find the one which does not
A@B (6)- A is 10m in north of B belong to that group?
A$B (10)- A is 14m in south of B (a) IK- Kismi bar
A%B (12)- A is 10m in east of B (b) EG- Coffee bite
A&B (15)- A is 13m in west of B (c) NP- Center fresh
(d) BD- Gems
P%Q(16), R&S(12), T&U(20), U$S(15), P@T(10),
(e) MO- Melody
W$R(17)
Directions (39-40): Study the following information
31. What is the shortest distance between point P and carefully and answer the questions given below:
point U?
(a) 2√130m (b) 520m (c) 130m In a certain code language
‘exams good for growth’ is coded as ‘jam, dam, mam, ram’
(d) 260m (e) None of these
‘bank exams are difficult’ is coded as ‘pam, jam, vam, bam’
32. If M is the mid point of the line segment TU, then ‘bank growth not easy’ is coded as ‘pam, ram, lam, tam’
determine the distance between the point T and point ‘easy for difficult preferred’ is coded as ’tam, mam, bam,
M? sam’
(a) 9m (b) 10m (c) 8m 39. What is the code for the word ‘exams’ in the given
(d) None of these (e) 7m code language?
(a) jam (b) tam (c) lam
33. What is the shortest distance between the point U and (d) ram (e) None of these
point W?
(a) 85m (b) 120m (c) None of these 40. ‘ram’ is the code for which of the following words?
(d) √104m (e) √130m (a) bank (b) exams (c) easy
(d) difficult (e) None of these

5 Adda247 Publications For any detail, mail us at


Publications@adda247.com
50+ Bank PO | Clerk Previous Year’s Papers 2016 – 2020

QUANTITATIVE APTITUDE
Directions (41-45): Following Line Graph given provides the details of total number of rides taken by 3 different drivers
in 5 different months and the Table DI given below provides the details of percentage of total female rides taken by all the
drivers in 5 different months and answer the questions accordingly.

700

650

600

550
A
500
B
450 C

400

350

300
JAN FEB MAR APR MAY

% of female ride % of female ride % of female ride


Month/ Driver taken by Driver taken by Driver taken by Driver
A B C
JAN 40% 25% 30%
FEB 30% 40% 44%
MAR 55% 50% 40%
APR 40% 45% 30%
MAY 60% 48% 60%
41. Total number of male rides taken by Driver B in (a) 14:19 (b) 17:23 (c) 18:23
January and march together is approximately what (d) 19:14 (e) 23:17
percentage more than the total female rides taken by 45. Find the total number of female rides taken by Driver
driver A in April and may together? B in all the five months together?
(a) 40% (b) 36% (c) 45% (a) 848 (b) 956 (c) 984
(d) 30% (e) 50% (d) 918 (e) 884
42. Find the total number of male rides taken by all the 46. A basket contains 8 Blue, 5 Red and 6 Green balls. 3
three drivers in March together? Balls are drawn from the basket, then find the
(a) 652 (b) 724 (c) 696 probability of getting all 3 balls drawn are of different
(d) 669 (e) 628 colors?
80 883 73
(a) 323 (b) 969 (c) 223
43. Find the difference between total no. of female ride 86 67
taken by all the 3 drivers in January to the total no. of (d) 969 (e) 173
male rides taken by all the 3 drivers in march?
47. Suresh was married 14 yrs ago and his present age is
(a) 327 (b) 294 (c) 268 3
(d) 214 (e) 237 times of the age at the time of his marriage. If his
2
1
44. Find the respective ratio of total no. of female rides son’s age is 3rd of his present age, then find the age of
taken by driver A in April and May together to the his son.
total no. of male ride taken by Driver B in January and (a) 16 yrs (b) 18 yrs (c) 14 yrs
march together? (d) 12 yrs (e) 20 yrs

6 Adda247 Publications For any detail, mail us at


Publications@adda247.com
50+ Bank PO | Clerk Previous Year’s Papers 2016 – 2020

48. A Square and an equilateral triangle have the same 54. Find the ratio of total dry waste produced on Monday,
area. If the perimeter of the square is 88 cm , then find Tuesday and Friday together to wet waste produced
the area of the equilateral triangle is? on Wednesday and Thursday together?
(a) 441 cm² (b) 400 cm² (c) 484 cm² (a) 194 :218 (b) 185 : 212 (c) 183 : 224
(d) 324 cm² (e) 576 cm² (d) 212:185 (e) 224 : 183
49. A, B and C together can complete a work in 8 days and 55. What is the amount of waste left by the end of the
A alone can complete the same work in 24 days. If A Wednesday?
and B started the work and after 2 days C also joined (a) 18 kg (b) 16 kg (c) 14 kg
them, then remaining work was completed by A, B (d) 20 kg (e) 12 kg
4
and C together in 65 days. Find in how many days B
56. Find the total quantity of dry waste on all the five days
alone can complete the whole work?
together?
(a) 28 days (b) 36 days (c) 24 days
(d) 32 days (e) 30 days (a) 381 kg (b) 413 kg (c) 361 kg
(d) 337 kg (e) 321 kg
50. In a mixture of milk and water, the proportion of milk
by weight is 60%. If from the 80 gm mixture, 20 gm of Directions (57-60): Given questions are based on a
mixture is taken out and 6 gm of pure water is added missing series pattern and following that pattern find the
to the mixture then find the ratio of milk and water in relation between P, Q and R.
the new mixture. 57. 300, 324, 384, 504, P, 1050
(a) 5 : 6 (b) 6 : 5 (c) 4 : 3 450, 474, Q, 654, 864, 1200
(d) 3 : 2 (e) 7 : 6 200, 224, 284, 404, R, 850
51. Three persons A, B and C started a business by (a) P>Q<R (b) P<Q<R (c) P=Q>R
2 1 3 (d) P<Q=R (e) P<Q>R
investing in the ratio of ∶ ∶ . After 5 months, B
3 2 4
2
increases his investment by 3rd of his initial amount. 58. 2700, 5400, P, 7200, 1440, 8640
If after 12 months, the difference of the profit shares 2100, Q, 1400, 5600, 1120, 6720
of A and C is Rs. 1,350 then find the profit share of B. 1500, 3000, 1000, R, 800, 4800
(a) Rs. 11,500 (b) Rs. 11,200 (c) Rs. 12,250 (a) P>Q>R (b) P<Q=R (c) P<Q>R
(d) Rs. 11,250 (e) Rs. 10,250 (d) P=Q>R (e) P<Q<R
Directions (52-56): The following Table DI shows the 59. 35, 70, 210, P, 4200, 25200
quantity of waste (Dry and Wet) picked by a truck on 5 140, 280, Q, 3360, 16800, 100800
different days. The capacity of the truck from Monday to 40, 80, 240, R, 4800, 28800
Wednesday is 180 kg and for rest two days is 150 kg. (a) P>Q<R (b) P<Q>R (c) P<Q<R
(d) P=Q<R (e) P>Q>R
Difference
Quantity Ratio of
between dry 60. 7, 15, 47, 191, Q, 5754
Day of wet wet to dry
and wet waste. 9, 19, 59, P, 1199, 7199
waste waste
(wet > dry) 11, 23, 71, 287, R, 8639
Monday - 5:4 20 kg (a) P>Q<R (b) P<Q>R (c) P<Q<R
Tuesday 110 kg - 22 kg (d) P=Q<R (e) P>Q>R
Wednesday 99 kg 9:7 -
Thursday 84 kg 7:y 24 kg Directions (61-64): In each of the following questions,
Friday - 12 : 7 40 kg two equations (I) and (II) are given. Solve the equations
and mark the correct option:
Note:- If the waste produce in a day is greater than the (a) if x>y
capacity of the truck, then the extra amount of waste will (b) if x≥y
be picked on next day. (c) if x<y
52. If on Wednesday the truck picks only 80% of the (d) if x ≤y
waste of his capacity, then find the difference between (e) if x = y or no relation can be established between x and
the quantity picked of wet waste and dry waste on the y.
same day? (priority given to wet waste)
61. I. 2x2 + 11x + 12 = 0 II. 8y2 -22y – 21 =0
(a) 64 kg (b) 48 kg (c) 62 kg
(d) 54 kg (e) 42 kg 62. I. x2 -17x -60=0 II. y2 + 42y +185 =0
53. What is the value of ‘y’? 63. I. x2 + 41x + 420 =0 II. 6y2 -11y -10 =0
(a) 4 (b) 5 (c) 6
(d) 3 (e) 8 64. I. x2 - 8x - 273 =0 II. y2 +6y -432 =0

7 Adda247 Publications For any detail, mail us at


Publications@adda247.com
50+ Bank PO | Clerk Previous Year’s Papers 2016 – 2020

Directions (65-69): Given pie chart provides the details of 69. Find the average of total fees received by both Junior
total number of patients visited to different doctors for Doctors and Senior Doctors together?
their problem and the table provides the details of fees of (a) Rs 1,28,500 (b) Rs 1,25,800 (c) Rs 1,30,250
different doctors. Read the instruction carefully and (d) Rs 1,22,350 (e) Rs 1,35,600
answer the question accordingly. Directions (70-74): Study the given passage carefully and
answer the following questions
Total Patients = 400 A train is travelling from station A to E. At station A, 80
person board in the ratio of male to female of 9 : 7.
At station B, 15 men got down and 5 women boarded the
train. At station C, half of the women got down and the
E A same number of men boarded the train
15% 25% At station D, x number of male got down and now the ratio
of male to female in train is 7 : 4
70. If 50% of male who were travelling from B to C do not
D B
have a valid ticket and 60% of the female travelling
28% 10%
C from B to C do not have a valid ticket, then find the
22% number of passengers who are travelling from B to C
with invalid ticket ?
(a) 43 (b) 39 (c) 47
(d) 49 (e) 51
Note:- 71. The number of females travelling from station B to C
A and B are Junior Doctor is approximately how much percent more than the
C and D are Senior Doctor number of males travelling from station D to E?
E is Dean (a) 10% (b) 21% (c) 18%
(d) 14% (e) 24%
Doctor’s Additional
Fees (in Rs) 72. Find the difference between the passengers travelling
Profile charge on fees
from starting point to destination point?
Junior Doctor 1000 15% (a) 25 (b) 30 (c) 34
Senior Doctor 1500 20% (d) 38 (e) none of these
Dean 2000 25% 73. Which of the following is true?
(A) The number of females travelling from station A
65. What is the difference between the total fees received to B is equal to the no. of males travelling from
by Doctor C and the total fees received by Doctor D? station D to E
(a) Rs 40800 (b) Rs 43200 (c) Rs 38400 (B) The total number of passengers travelling from
(d) Rs 36800 (e) Rs 34200 Station C to D is 45% of the no. of males who
boarded from the starting point
66. Find the respective ratio of total fees received by (C) The difference between the no. of male and
Doctor A from all its patient to the total fees received female travelling from station D to E is half of the
by Doctor E from all its patients difference between the no. of males and females
(a) 27 : 31 (b) 30 : 23 (c) 23 : 30 travelling from station C to D
(d) 31: 27 (e) 29 : 31 (a) Only A (b) Only C (c) Only A & C
(d) Only B & C (e) All A, B and C
67. Total fees received by junior doctor B is what
74. Find the ratio of total no. of passengers travelling
percentage more/less than the total fees received by
from station D to E and B to C?
Doctor E? (a) 17 : 13 (b) 11 : 14 (c) 13 : 17
1 2
(a) 69 3% (b) 60 3% (c) 72% (d) 14 : 11 (e) 17 : 19
2
(d) 68 % (e) 64% 75. Total distance between A and B is 792 km and Car P
3
starts from station A at 8 a.m with speed 64 km/hr
68. If the Doctor E’s fees is increased by 15% from the towards B and Car Q starts from station B at 11 a.m
current fees, then find the total fees received by with speed 86 km/hr towards A. Find the distance
Doctor E when the fees is increased? (in Rs) from station B when both cars will meet each other ?
(a) 1,72,500 (b) 1,78,500 (c) 2,12,000 (a) 430 km (b) 258 km (c) 344 km
(d) 1,92,500 (e) 2,04,000 (d) 312 km (e) 384 km

8 Adda247 Publications For any detail, mail us at


Publications@adda247.com
50+ Bank PO | Clerk Previous Year’s Papers 2016 – 2020

76. A, B and C invested Rs 5500, Rs 4500 and Rs 6000 for 78. A train travelling at 54 kmph crosses a platform in 25
three years in a partnership. After 1 year, A decreased seconds and a man standing on the same platform in
his investment by Rs 1000, B increased his 12 seconds
investment by Rs.500 and C’s investment remains the
same for the whole period of time. If the total profit at Quantity I: Length of the train.
the end of 3 years is Rs 9400, then find the profit share Quantity II: Length of the platform.
of C?
79. P works twice as fast as Q, whereas P and Q together
(a) Rs 4200 (b) Rs 3600 (c) Rs 3800
(d) Rs 4000 (e) Rs 3000 can work 3 times as fast as R. P,Q and R together can
15
Directions (77-80): Each of the following question is do the same work in 2
days
followed by two quantities I, and II. You have to determine Quantity I: Time taken by P and Q together to
the value of the quantities using the information provided complete the work.
and compare the quantities to answer as per the Quantity II: Time taken by Q and R together to
instruction set provided below.
(a) Quantity I>Quantity II complete the work.
(b) Quantity I≥Quantity II 80. The age of Manan is 50% more than that the age of
(c) Quantity I<Quantity II
Shikhar. The age of Rohit is 20% less than the age of
(d) Quantity I≤Quantity II
(e) Quantity I=Quantity II or no relation Manan. The age of Krunal is 10% less than the age of
Rohit.
77. Quantity I: By selling 15 apples, a seller gains the
selling price of 2 apples. Calculate his gain percentage. Quantity I: By what percent the age of Rohit is more
Quantity II: 25% profit is gained when an article is than the age of Shikhar.
sold for 625 rupees. Calculate the loss % when the Quantity II: By what percent the age of Krunal is
same article is sold for 435 rupees. more than the age of Shikhar.

ENGLISH LANGUAGE

Directions (81-85): In each of the questions given below, (I) apathetic (II) benevolent
a sentence is given with one blank. Below each sentence, (III) passive (IV) indifferent
FOUR words are given. Five options are provided with (a) Only (III)
various combinations of these words. You have to choose (b) Only (I), (III) and (IV)
the combination with the correct set of words which can (c) Both (I) and (III)
coherently fit into the given sentence. (d) Both (I) and (II)
81. When Margot lost all her money gambling, her family (e) All (I), (II), (III), (IV)
was forced to live in _____________. 84. Taking the yoga class has helped me release my stress
(I) penury (II) mansion so I can become more ____________________.
(III) grandeur (IV) poverty (I) abundant (II) pernicious
(a) Only (I) (III) placid (IV) injurious
(b) Only (I), (III) and (IV) (a) Only (III)
(c) Both (I) and (IV) (b) Only (I), (III) and (IV)
(d) Both (II) and (III) (c) Both (I) and (III)
(e) All (I), (II), (III), (IV) (d) Both (I) and (II)
82. Scared of heights all her life, nothing would ___________ (e) All (I), (II), (III), (IV)
Ruth's fear of flying. 85. The committee met for several hours, but
(I) alleviate (II) reduce ____________________ nothing because they argued the
(III) aggravate (IV) understand whole time.
(a) Only (III) (I) achieved (II) accomplished
(b) Only (I), (III) and (IV) (III) gained (IV) attained
(c) Both (I) and (III) (a) Only (III)
(d) Both (I) and (II) (b) Only (I), (III) and (IV)
(e) All (I), (II), (III), (IV) (c) Both (I) and (III)
83. Sometimes it seems as though we live in a/an (d) Both (I) and (II)
_____________ world filled with uncaring people. (e) All (I), (II), (III), (IV)

9 Adda247 Publications For any detail, mail us at


Publications@adda247.com
50+ Bank PO | Clerk Previous Year’s Papers 2016 – 2020

Directions (86-90): In the following questions two Column (2)


columns are given containing three sentences/phrases (D) in their late morning deals .
each. In first column, sentences/phrases are A, B and C and (E) Foreign Portfolio Investors (FPIs) sold shares .
in the second column the sentences/phrases are D, E and (F) with 22 of the 30 Sensex stocks finishing lower.
F. A sentence/phrase from the first column may or may not (a) C-F (b) B-F (c) A-D
connect with another sentence/phrase from the second (d) C-E (e) None of these
column to make a grammatically and contextually correct
90. Column (1)
sentence. Each question has five options, four of which
display the sequence(s) in which the sentences/phrases (A) A writer needs
can be joined to form a grammatically and contextually (B) the garden itself being
correct sentence. If none of the options given forms a (C) All this is fine
correct sentence after combination, mark (e), i.e. “None of Column (2)
these” as your answer. (D) houseboat on the silent river
(E) somehow find the necessary bubble
86. Column (1) (F) quiet and peaceful environment around him in
(A) The company along with its order to write.
(B) Including ongoing initiatives (a) C-F (b) B-F (c) A-F
(C) IL&FS has appointed (d) C-E (e) None of these
Column (2)
(D) Aligned with the broader objective Directions (91-100): Read the following passage carefully
(E) Advisers to prepare a resolution plan and answer the questions given below them. Certain
(F) Subsidiaries is facing a liquidity crisis words/phrases have been printed in bold to help you
(a) C-E and B-F locate them while answering some of the questions.
(b) A-F Indian roads are usually characterized by poor
(c) C-E infrastructure and congestion which affect travel time and
(d) Both (b) and (c) road safety. This is a big hindrance in economic
(e) None of these development and leads to inefficiency in the
transportation of goods and services across the country. To
87. Column (1) address this, the government has embarked upon a
(A) The increase in MSP for rabi crops comes massive overhaul of the country’s road network through
(B) There are notified crops Bharatmala Pariyojana—an umbrella highway
(C) For big companies, there are instances development programme involving 34,800 km of road
Column (2) network at an investment of Rs5.35 trillion, to be
(D) The government said in a release.
completed by 2022. The programme focuses on optimizing
(E) Just ahead of the RBI monetary policy
the efficiency of road traffic movement across the country
announcement.
by bridging critical infrastructure gaps through shorter
(F) Of even the infrastructure getting damaged.
routes. The end goal is to create economic corridors (ECs)
(a) C-F
along the path—new industries, more employment and
(b) C-E and B-F
new markets.
(c) A-E
The programme, however, will have a negative bearing on
(d) Both (a) and (c)
the existing road network because it will compete directly
(e) None of these
with some of the existing build-operate-transfer (BOT) toll
88. Column (1) road projects. Out of the 44 ECs, about 21 would partially
(A) India will be the third largest aviation or fully affect the existing alignments, while the remaining
(B) The biggest contribution in 23 that involve upgradation of existing alignment will not
(C) Firstly, we are seeing an result in any deviation. Among the 21 corridors affected,
Column (2) eight have a totally different route (which is shorter) while
(D) market globally a year sooner. the remaining 13 have some deviations from the existing
(E) as the world’s largest aviation alignment. Overall, there are 24 BOT projects and one
(F) restrictive protectionist measures operate-maintain-transfer (OMT) project whose traffic
(a) C-F (b) B-F (c) A-D could be affected by the proposed ECs. The Bharatmala
(d) C-E (e) None of these programme may result in traffic diversion from the
89. Column (1) existing road network to new roads, thereby affecting the
(A) Also, the broader NSE Nifty toll collection and, consequently, the debt servicing ability
(B) All the sectoral indices closed in the red of some of the BOT and OMT projects. This has raised the
(C) Maruti Suzuki, India’s largest carmaker risk of default on 25 national highway toll projects which

10 Adda247 Publications For any detail, mail us at


Publications@adda247.com
50+ Bank PO | Clerk Previous Year’s Papers 2016 – 2020

involve Rs19,435 crore of debt. The risk of such loan 93. The total debt at risk is
defaults will add to banks’ and financial lenders’ stressed (a) Rs10,456 crore
assets and non-performing assets. In terms of risk, 12% of (b) Rs3,483 crore
the projects have a high risk of leakage in traffic, if a (c) Rs19,435 crore
completely alternative route is available, 16% of the (d) Rs9,416 crore
projects have moderate risks, and 72%, low risks. To arrive (e) Rs18,544 crore
at the debt at risk, the debt outstanding for each of these
special purpose vehicles (SPVs), their repayment tenure, 94. The appropriate title of the passage is
concession end date, credit profile of the SPV and its (a) Renegotiating the concession agreement.
sponsor credit risk profile, are considered. Out of the total (b) Bharatmala and the rising debt issues
debt at risk for the 25 affected projects about Rs6,536 (c) Creating economic corridors
crore, which accounts for about 34% of the total debt at (d) the rising issue of stressed assets
risk, is high-risk. Projects with debt at a moderate risk have (e) The Kelkar Committee
an aggregate debt of Rs3,483 crore, while about Rs9,416 95. Remedial measure(s) available to BOT operators
crore of debt is considered to be low-risk. is/are
To ensure that the existing BOT projects that are at risk of (I) provide them the necessary resources for
default did not turn bad for the financial institutions, swift accomplishment of the project.
and adequate measures are needed. The Kelkar committee (II) realigning the terms and conditions of the model
had observed that since infrastructure projects span over concession agreement to ensure that banks do
20-30 years, a private developer may lose bargaining not end up accumulating NPAs.
power owing to abrupt changes in the economic or policy (III) Allowing renegotiation of the terms of
environment. It has thus recommended that the private concession agreement to private sector.
sector must be protected against such loss. This could be
(a) Only (I)
ensured by allowing renegotiation of the terms of the
(b) Both (II) and (III)
concession agreement.
(c) Only (III)
Financial institutions are already reluctant to finance the
(d) Both (I) and (II)
infrastructure sector, given the rise in non-performing
(e) All are correct
assets (NPAs). Add to this the probable difficulties that
would arise in the case of 25 BOT projects, which would 96. A phrase “did not turn” is given in the passage is
put additional stress on the road infrastructure exposure. highlighted which may or may not contain
The need of the hour is to realign the terms and conditions grammatical error. There are five alternatives given
of the model concession agreement to ensure that banks do below, one of which may replace the existing
not end up accumulating NPAs. Having an appropriate highlighted part to make the sentence grammatically
remedial mechanism for BOT operators will help retain correct and contextually meaningful.
interest for investments in new projects; for the lenders, it Choose the most appropriate alternative as your
will help curtail the number of stressed assets from the answer. If the phrase is grammatically correct, as
risk of default. given, and doesn’t require any correction, choose
91. According to the passage, Bharatmala Project aims to option (e) i.e., “No correction required” as your
(I) impact industrial development positively answer.
(II) create employment and develop new markets (a) does not turn (b) will not turn (c) do not turn
(III) improve the political status of the country. (d) could not turn (e) No correction required
(a) Only (I) Direction (97-98): Which of the following alternatives
(b) Only (II) and (III) among the five options provides the MOST SIMILAR
(c) Only (III)
meaning(s) of the word given in BOLD as used in the
(d) Only (I) and (II)
passage?
(e) All are correct
97. Curtail
92. According to the passage, what worries BOT
(I) reduce (II) curb
operators?
(III) retrench (IV) trunk
(a) The risk of loan default.
(a) Only (III)
(b) Disruptions in integrated Infrastructure network.
(c) Inability to maintain the overall structure of (b) Only (II) and (IV)
highway network in India. (c) Only (I), (II) and (III)
(d) Both (a) and (b) (d) Only (II), (III) and (IV)
(e) All are correct (e) All are correct

11 Adda247 Publications For any detail, mail us at


Publications@adda247.com
50+ Bank PO | Clerk Previous Year’s Papers 2016 – 2020

98. Embarked (B) Clearly, bankers were overconfident and probably did
(I) commence (II) initiate too little due diligence for some of these loans.
(III) rot (IV) undertake (C) The disbursement under Mudra loans alone is Rs.6.37
(a) Only (III) lakh crore, which is over 7% of the total outstanding
(b) Only (II) and (IV) bank credit.
(c) Only (I), (III) and (IV) (D) It is debatable whether banks have the resources and
(d) Only (I), (II) and (IV) manpower to do this when they are chasing the bigger
(e) All are correct borrowers for business and, increasingly these days,
recoveries.
Direction (99-100): Which of the following alternatives (E) In his note to Parliament’s Estimates Committee on
among the five options provides the MOST OPPOSITE bank non-performing assets (NPAs), Mr. Rajan has
meaning(s) of the word given in BOLD as used in the flagged three major sources of potential trouble.
passage? (F) These loans have been sanctioned under the Pradhan
Mantri Mudra Yojana, which aims to ‘fund the
99. Hindrance unfunded’, and is a signature scheme of the NDA
(I) contrite (II) impetus government.
(III) plausible (IV) Check (G) They are Mudra credit, which is basically small-ticket
(a) Only (II) loans granted to micro and small enterprises; lending
(b) Only (II) and (IV) to farmers through Kisan Credit Cards; and
(c) Only (I), (II) and (III) contingent liabilities under the Credit Guarantee
(d) Only (II), (III) and (IV) Scheme for MSMEs, run by the Small Industries
(e) All are correct Development Bank of India.
100. Reluctant (H) Given that these are small loans up to Rs.10 lakh each,
with the borrowers mostly from the informal sector,
(I) vaunt (II) willing
banks have to monitor them very closely.
(III) splendid (IV) eager
(a) Only (III) 106. Considering statement (A) “Former RBI Governor
(b) Only (II) and (IV) Raghuram Rajan’s note of caution on the next
(c) Only (I), (II) and (III) financial crisis that could be building up needs to
(d) Only (II), (III) and (IV) be taken in all seriousness.” as the first sentence of
(e) All are correct the rearranged paragraph, then which among the
following fails to become the part of the coherent
Directions (101-105): Read each sentence to find out paragraph?
whether there is any grammatical or idiomatic error in it. (a) C (b) F (c) B
The error, if any, will be in one part of the sentence. The (d) H (e) G
number of that part is the answer. If there is ‘No error’, the
107. Considering statement (A) “Former RBI Governor
answer is (e). (Ignore errors of punctuation, if any.)
Raghuram Rajan’s note of caution on the next
101. Why we do not (a)/ meet to discuss (b)/ this matter financial crisis that could be building up needs to
in detail (c)/ on next Friday? (d)/ No error. (e) be taken in all seriousness.” as the first sentence of
the rearranged paragraph, then which one among the
102. Had the opportunity (a)/ been given to him (b)/ he following can consecutively follow the last statement
will have (c)/ proved successful (d). / No Error. (e) after the rearrangement (excluding the incoherent
103. Jamuna takes delight (a)/with music and (b)/hopes to one)?
become (c)/a successful musician. (d)/No error. (e) (a) Not only could they play one lender off against
another by threatening to divert payments to the
104. In spite of toiling (a)/ very hardly he (b)/ realized that favored bank, they could also refuse to pay
he had (c)/ not earned anything substantial. (d)/ No unless the lender brought in more money,
error. (e) especially if the lender feared the loan becoming
105. You should inform (a)/ your superiors about (b)/ all an NPA.
(b) The risk is that these small-ticket loans will drop
those events which are (c)/ directly related to your
under the radar and build into a large credit
work. (d)/ No error. (e)
issue in course of time. The same logic holds true
Directions (106-110): Answer the following questions for crop loans made through Kisan Credit Cards.
after rearranging the following sentences into a coherent (c) Yet, unless we can determine the unaccounted
paragraph and identify the sentence that doesn’t fit into the wealth of bankers, I hesitate to say a significant
context of the paragraph. Statement (A) is the first element was corruption.
sentence of the paragraph after the rearrangement. (d) Frauds are different from normal NPAs in that
(A) Former RBI Governor Raghuram Rajan’s note of the loss is because of a patently illegal action, by
caution on the next financial crisis that could be either the borrower or the banker.
building up needs to be taken in all seriousness. (e) None of these

12 Adda247 Publications For any detail, mail us at


Publications@adda247.com
50+ Bank PO | Clerk Previous Year’s Papers 2016 – 2020

108. Among the following pairs which one of them is compost or production of methane for household use or
formed with two consecutive statements after the power generation. But it is a major opportunity __(115)___.
rearrangement? Organic waste that could help green cities and __(116)__
(a) A - G (b) B - E (c) A - F small and affordable household biogas plants is simply
(d) D - C (e) E – G being thrown away. It is also ironic that while some
countries such as Rwanda and Kenya have introduced stiff
109. Considering statement (A) “Former RBI Governor
penalties for the __(117)___ of flimsy plastic bags, India is
Raghuram Rajan’s note of caution on the next
doing little to __(118)__ them from drifting into suburban
financial crisis that could be building up needs to
garbage mountains, rivers, lakes and the sea, and being
be taken in all seriousness.” as the first sentence of
___(119)___ by cattle feeding on dumped ___(120)___.
the rearranged paragraph, identify the correct
sequence of the sentences to form a coherent 111. (a) demolition (b) revive (c) restrict
paragraph (excluding the incoherent one). (d) absence (e) permission
(a) AEGCFHD (b) ADECBHF (c) AGHBCDE
112. (a) settled (b) equipped (c) surrounded
(d) ABCDGHF (e) AEGFCBD
(d) confined (e) embedded
110. Considering statement (A) “Former RBI Governor
113. (a) supplies (b) forms (c) classifies
Raghuram Rajan’s note of caution on the next
(d) demands (e) extends
financial crisis that could be building up needs to
be taken in all seriousness.” as the first sentence of 114. (a) manufacture (b) accumulation(c) generation
the rearranged paragraph, which of the following (d) foundation (e) conception
statement should be FOURTH sentence after the
115. (a) awed (b) misled (c) lost
rearrangement? (Excluding the incoherent sentence)
(d) absorbed (e) rapt
(a) D (b) C (c) G
(d) H (e) B 116. (a) feed (b) deliver (c) nurtures
(d) provide (e) produce
Directions (111-120): In the following passage, some of
the words have been left out, each of which is indicated by 117. (a) interest (b) favor (c) gain
a number. Find the suitable word from the options given (d) selection (e) use
against each number and fill up the blanks with
118. (a) reduce (b) discourage (c) prevent
appropriate words to make the paragraph meaningfully
(d) facilitate (e) reject
complete.
119. (a) absorbed (b) grazed (c) ravaged
In the __(111)___ of stakeholders at the local body level,
(d) ingested (e) grasped
recoverable resources __(112)__ in discarded materials are
lost due to dumping. Organic refuse, which __(113)__ about 120. (a) limit (b) refuse (c) exclude
50% of all garbage, readily lends itself to the ___(114)__ of (d) evade (e) cease

Solutions

REASONING ABILITY
Solutions (1-5): 6. (d); In the above question it is clear that Statement II
will be the cause and Statement I will be its
Year Age Persons Professions effect. Because it is mentioned in statement II
1968 51 Aditi Artist that jawans have doubt how to deal with the
1972 47 Dheeraj Manager women so this will be cause and its effect will be,
1978 41 Kamal Teacher The Indian Army will recruit women in combat
1981 38 Nisha HR roles to tackle such kind of situation.
1987 32 Swati Pilot 7. (c); Statement (I) suggests that royals are not kings
1998 21 Pranav Doctor or queens by choice but because they have to, for
the welfare of the country which is in line with
2005 14 Pooja Engineer
the statement. (II) fortifies the statement by
1. (e); 2. (b); 3. (d); stating out an example in which he had to face
problems because of his royal background. (III)
4. (b); 5. (b); is irrelevant to the statement.

13 Adda247 Publications For any detail, mail us at


Publications@adda247.com
50+ Bank PO | Clerk Previous Year’s Papers 2016 – 2020

8. (d); By statement II, P is in topmost floor.


Floors Persons
5 P
4 R
3 S
2 T
1 F
9. (c);
20. (a); By statement I, C is at middle of the row.

21. (b); I. A > C (False) II. E < B (True)


22. (a); I. Q > T (True) II. S ≤ V (False)
23. (d); I. F < N (False) II. N > D (False)
10. (b); Solutions (24-28):

24. (e); 25. (c); 26. (b);


Solutions (11-15): 27. (e); 28. (c);
29. (d); For I- This is in-line with the given statement as
it states about the meeting in between Indian
and UAE delegations which is also mentioned in
the given statement that India and the UAE have
11. (c); 12. (b); 13. (a); discussed measures to deepen cooperation in a
range of areas.
14. (d); 15. (c);
For II- This is not- in-line with the given
16. (e); By combining Statement I and II, we get that T is statement as it states about shipment of wheat
the husband of R. from India to Afghanistan through the Iranian
port of Chabahar whereas the given statement
states that India and the UAE have discussed
measures to deepen cooperation in a range of
areas.
For III- This is in-line with the given statement
as it states that UAE has raised its investment
17. (e); By combining Statement I and II, we get that the profile in India which is also mentioned in the
code for “cow” is “dc”. given statement that India and the UAE have
Words Code discussed measures to deepen cooperation in a
big mn range of areas.
cow dc 30. (c); For I- This statement can’t be hypothesized from
rat vg the given statement as nothing is mentioned
dew/grass bh/sx about facilities given under social welfare
same qs schemes for OBCs in the given statement.
use we For II- This statement can be hypothesized from
the given statement as it is mentioned in the
18. (d); We can’t conclude that who is the 2nd shortest given statement that Modi-led government has
person by combining both statements. moved to reframe social welfare schemes for
19. (c); By statement I, F is in topmost floor. other backward classes (OBCs) in the run-up to
the high stakes elections in Gujarat and
Floors Persons
Himachal Pradesh.
5 F For III- This statement can’t be hypothesized
4 R from the given statement as nothing is
3 T mentioned about facilities given under social
2 P welfare schemes for OBCs in the given
1 S statement.

14 Adda247 Publications For any detail, mail us at


Publications@adda247.com
50+ Bank PO | Clerk Previous Year’s Papers 2016 – 2020

31. (a); Solutions (34-38):

Box Toffee
IK Boomer
FH Kismi bar
JL Hajmola
MO Coffee bite
EG Gems
BD Poppins
NP Melody
AC Center fresh
32. (a);
34. (e); 35. (b); 36. (d);
37. (e); 38. (e);
Solutions (39-40): The codes for the words are as follows:

Words Codes
exams jam
growth ram
for mam

33. (d); good Dam


bank Pam
difficult Bam
are Vam
easy tam
not Lam
preferred Sam

39. (a); 40. (e);

QUANTITATIVE APTITUDE

41. (b); Total number of male rides taken by Driver B in 45 50 60


= 540 × + 420 × + 360 ×
January and March together 100 100 100
75 50 = 243+210+216 = 669
= 480 × + 420 ×
100 100
= 360 + 210 = 570 43. (e); Total no. of female ride taken by all the 3 drivers
Total no. of female rides taken by driver A in in January
40 25 30
April and may together = 360 × 100 + 480 × 100 + 560 × 100
40 60
= 570 × 100 + 320 × 100 = 144 + 120 + 168 = 432
= 228 + 192 = 420 Total no. of male rides taken by all the 3 drivers
570−420
Required percentage = 420 × 100 in March
45 50 60
= 35.71% = 36% (approx.) = 540 × + 420 × + 360 ×
100 100 100
= 243 + 210 + 216 = 669
42. (d); Total number of male rides taken by all the three
Required difference = 669 – 432 =237
drivers in March together
15 Adda247 Publications For any detail, mail us at
Publications@adda247.com
50+ Bank PO | Clerk Previous Year’s Papers 2016 – 2020

44. (a); Total no. of female rides taken by driver A in 50. (b); Ratio of Milk and water in mixture = 60 : 40
April and May together =3:2
40 60 Quantity of Milk left in mixture after 20 gm of
= 570 × 100 + 320 × 100 3
= 228 + 192 = 420 mixture is taken out = 60 × 5 = 36 gm.
2
Total no. of male ride taken by Driver B in Quantity of water left = 60 × = 24 gm
5
January and march together 36
75 50 ∴ Required ratio = 24+6 = 6 : 5
= 480 × 100 + 420 × 100
= 360 + 210 = 570 51. (d); Ratio of amount invested by P, Q and R = 8 : 6 : 9
420 𝐀 𝐁 𝐂
Required ratio = 570 = 14 : 19 8 × 12 6 × 5 + 10 × 7 9 × 12
45. (b); Total number of female rides taken by Driver B 96 : 100 : 108
Ratio of their profit shares
in all the five months together A B C
25 40 50 45
= 480 × + 380 × + 420 × + 520 × 24 : 25 : 27
100 100 100 100 1350
+ 500 ×
48 Profit share of B = 27 − 24 × 25 = Rs. 11,250
100
= 120 + 152 + 210 + 234 + 240 = 956 Solutions (52–56):
46. (b); Total Balls = 8 +5 +6 =19 balls Ratio of Difference
Probability of getting all balls of same color Quantity
wet to between dry Dry
8c3 +5c Day of wet
=
3 +6c3 dry and wet waste. waste
waste
19c3 waste (wet>dry)
8×7×6 5×4 6×5×4
( + + ) 86 Mon 100 kg 5:4 20 kg 80 kg
= 3×2×1 2×1 3×2×1
19 × 18 × 17 =969
3×2×1
Tue 110 kg 5:4 22 kg 88 kg
Required probability = 1 -
86
=
883 Wed 99 kg 9:7 22 kg 77 kg
969 969
Thu 84 kg 7:5 24 kg 60 kg
47. (c); Let suresh’s age at the time of his marriage be x Fri 96 kg 12 : 7 40 kg 56 kg
yrs
3 The above table is formed by calculating all the missing
Therefore, Suresh’s present age = x yrs data as per instructions
2
Also, Suresh’s present age = (x +14) yr 52. (d); On Wednesday the truck picks only 80% of the
3
2
x = x +14 waste of his capacity
3
x – x =14 Therefore, waste picked on Wednesday
2 80
= 180 × 100 =144 kg
x = 28 years
1 3 x 28 Quantity of wet waste picked = 99 kg
His son’s age =3 of 2 x = 2 = 2 = 14 yrs
Quantity of dry waste picked = 144 – 99 = 45 kg
48. (c); Here, perimeter of the square = 88 cm (given) Required difference = 99 – 45 = 54 kg
88
Side of the square = 4 =22 cm 53. (b); As on Thursday, Quantity of wet waste is 84 kg
Area of the square= a2 =(22)2 = 484 cm2 and the difference between wet waste and dry
waste is 24 kg (Given)
As it is given that area of equilateral triangle is
Quantity of Dry waste= 84 -24 = 60 kg
equal to the area of the square
As in the question, the ratio of wet waste and dry
So, the area of the equilateral triangle is 484 cm2 waste is 7 : y (Given)
84 7
49. (e); Let the time take by A,B and C alone be A days, B ATQ, =
60 y
days and C days respectively. 60
1 1 34 1 1 1 Therefore, y =12 =5
ATQ, 2 (A + B) + 5
× (A + B
+C) =1
1 1 34 1 54. (e); Total dry waste produced on Monday, Tuesday
2 (24 + B) + 5
× 8
=1 and Friday together= 80 + 88 + 56 = 224 kg
1 2 3
+ = Total wet waste produced on Wednesday and
12 B 20
2 3 1 Thursday together = 99 +84 = 183 kg
B
= 20 – 12 Required ratio = 224:183
2 9−5
B
= 60 55. (c); Total amount of waste left by the end of the
2 4 Wednesday
B
= 60
= (180 – 180) + (198 -180) +(176 -180)
B = 30 days = (0 + 18 – 4) = 14 kg
.

16 Adda247 Publications For any detail, mail us at


Publications@adda247.com
50+ Bank PO | Clerk Previous Year’s Papers 2016 – 2020

56. (c); Total quantity of dry waste taken on all the 5 II. 8y2 -22y – 21 =0
days together = 80 + 88 + 77 + 60 + 56 = 361 kg 8y2+6y-28y-21 =0
2y(4y+3)-7(4y+3)=0
57. (a); Here, the pattern followed is
(2y-7)(4y+3)=0
300 + (33 - 3)= 324 7 3
324 + (43 - 4)= 384 y = ,-
2 4
384 + (53 - 5)= 504 So, x < y
504 + (63 - 6)= 714 =P
62. (a); I. x 2 − 17x − 60 = 0
714 + (73 - 7)= 1050
x2-20x+3x-60=0
The next two series given follows the same
x(x-20)+3(x-20)=0
pattern as above series
(x+3)(x-20)=0
And according to that pattern, the value of Q and
R are 534 and 614 respectively x= - 3, 20
Here, P > Q < R II. y2 +42y + 185 =0
y2+37y+5y+185 =0
58. (c); Here, the pattern followed is y(y+37)+5(y+37)=0
2700 × 2 =5400 (y+5)(y+37)=0
5400 ÷ 3 =1800 = P y = -5, - 37
1800 × 4 =7200 So, x > y
7200 ÷ 5 =1440
1440 × 6 =8640 63. (c); I. x2 + 41x + 420 =0
The given next two series follows the same x2+21x+20x+420=0
pattern as above series x(x+21)+20(x+21)=0
And according to that pattern, the value of Q and (x+20)(x+21)=0
R are 4200 and 4000 respectively x= -20, - 21
Here, P < Q > R II. 6y2 -11y – 10 =0
6y2-15y+4y-10 =0
59. (d); Here, the pattern followed is 3y(2y-5)+2(2y- 5)=0
35 × 2 =70 (2y-5)(3y+2)=0
70 × 3 =210 5 2
y = ,-
210 × 4 =840 =P 2 3
840 × 5 =4200 So, x < y
4200 × 6 =25200 64. (e); I. x 2 − 8x − 273 = 0
The next two series follows the same pattern as x2-21x+13x-273=0
above series x(x-21)+13(x-21)=0
And according to that pattern, the value of Q and (x+13)(x-21)=0
R are 840 and 960 respectively
x= -13, 21
Here, P = Q < R
II. y2 +6y – 432 =0
60. (c); Here, the pattern followed is y2+24y-18y-432 =0
(7 × 2) + 1=15 y(y+24)-18(y+24)=0
(15 × 3)+2= 47 (y-18)(y+24)=0
(47 × 4)+3= 191 y = 18, - 24
(191 × 5)+4= 959 =Q So, No relation
(959 × 6)+5= 5759
65. (b); Total fees of Doctor C per patient
The next two series follows the same pattern as 20
above series = 1500 + 1500× =Rs 1800
100
And according to that pattern, the value of P and Total fees received by Doctor C = 1800 × 88
R are 239 and 1439 respectively = Rs 158400
Here, P < Q < R Total fees of Doctor D per patient
20
61. (c); I. 2x2 + 11x + 12 = 0 = 1500 + 1500× = Rs 1800
100
2x2 + 8x + 3x + 12 = 0 Total fees received by Doctor D = 1800 × 112
2x (x + 4) + 3(x + 4) = 0 = Rs 201600
(x + 4) (2x + 3) = 0 Required difference = 201600 – 158400
3
x= -4, - = Rs 43200
2

17 Adda247 Publications For any detail, mail us at


Publications@adda247.com
50+ Bank PO | Clerk Previous Year’s Papers 2016 – 2020

66. (c); Total fees of Doctor A per patient 70. (b); Total males who does not have a valid ticket
15 50
= 1000 + 1000× =Rs 1150 from Station B to C = 30 × =15
100 100
Total fees received by Doctor A = 1150 × 100 Total females who does not have a valid ticket
=115000 Rs 60
from Station B to C = 40 × 100 =24
Total fees of Doctor E per patient
25 Total invalid ticket travellers from Station B to C
= 2000 + 2000× 100 =Rs 2500 = 15 +24 = 39
Total fees received by Doctor E = 2500 × 60
= 150000 Rs 71. (d); Total no. of females travelling from station B to C
115000 = 40
Required ratio =150000 = 23 : 30
Total no. of males travelling from station D to E
67. (a); Total fees of Doctor B per patient = 35
15 40−35
= 1000 + 1000× =Rs 1150 Required percentage = 35 × 100 =14.28%
100
Total fees received by Doctor B = 1150 × 40 =14%(approx.)
= 46000 Rs
Total fees of Doctor E per patient 72. (a); Total no. of passengers travelling at starting
25 point= 45+35=80
= 2000 + 2000× 100 =Rs 2500
Total no. of passengers travelling till destination
Total fees received by Doctor E = 2500 × 60 point =35 +20 =55
= 150000 Rs
150000−46000 Required difference =80 -55 =25
Required percentage = 150000 × 100
1 73. (c); In Statement A,
= 69 % Total no. of females travelling from station A to
3

68. (a); Current fees of Doctor E per patients =Rs 2500 B = 35


Increased fees of Doctor E per patients Total no. of males travelling from station D to E
15
= 2500 + 2500 × 100 =Rs 2875 = 35
Therefore, Statement A is true
Total fees received by Doctor E (after fees
increase) = 2875 × 60 =Rs 172500 In Statement B,
Total no. of passengers travelling from station C
69. (c); Total fees of Junior Doctor per patient to D =70
15
= 1000 + 1000× 100 = Rs 1150 Total no. of males who boarded from the starting
Total fees of Senior Doctor per patient point =45
20 Therefore, Statement B is false
= 1500 + 1500 × 100 = Rs 1800
Fees received by both junior doctor and senior In Statement C,
doctor together = (100+40) × 1150 + (112+88) Difference between no. of males and females
× 1800= 161000 + 360000 =521000 travelling from station D to E =35 -20 =15
521000 Difference between no. of males and females
Required average = 4 = Rs 130250
travelling from station C to D = 50 -20 =30
Solutions (70-74): As given, at station A ,80 person board Therefore, statement C is true
and the ratio of male to female is 9: 7, therefore there will Therefore Statement A and C is true
be 45 male and 35 female.
Then at station B,15 men got down and 5 women board the 74. (b); total no. of passengers travelling from station D
train, therefore total men at station B be 30 and total to E= 35 +20=55
female is 40. total no. of passengers travelling from station B
Then at station C, half of the women got down and same no. to C = 30+40=70
55
of men boarded the train, then total male will be 50 and Required ratio = 70=11: 14
total female will be 20.
Finally at station D, x no. of male got down and ratio of male 75. (c); Relative speed of car =64 + 86 = 150 km/hr
to female is 7 : 4, then total no. of male will be 35 and total Distance travelled by Car P in 3 hrs = 64 × 3
female will be 20 = 192 km
STATION Male Female Remaining distance =792 -192=600 km
A 45 35 Time taken to cover remaining distance where
600
B 30 40 they will meet each other =150=4 hrs
C 50 20 Distance travelled by Car Q from station B in 4
D 50 – x = 35 20 hrs = 86 × 4 = 344 km

18 Adda247 Publications For any detail, mail us at


Publications@adda247.com
50+ Bank PO | Clerk Previous Year’s Papers 2016 – 2020

76. (b); A : B : C Again, Time taken by train to cross the platform


5500 × 12 4500 × 12 6000 × 36 = 25 sec
+ + Length of the platform = 25 × 15 – 180
4500× 24 5000× 24 = 375 – 180 =195 m
= = = Hence, Quantity I < Quantity II
29 : 29 : 36 79. (c); P : Q = 2:1 (efficiency ratio)
36
Profit share of C = 9400 × 94 = Rs 3600 (P+Q) : R = 3 : 1 (efficiency ratio)
Thus, P : Q : R = 2: 1: 1 (efficiency ratio)
77. (a); Quantity I: Let SP of one apple be 1 rs 15
Now total work = × 4= 30 units
SP of 15 apple = 15 rs 2
And profit = 2 rs Quantity I: Time taken by P and Q together to
30
CP = 15 -2 =13 rs. complete the work = = 10 days
3
2 Quantity II: Time taken by Q and R together to
% Profit = 13 × 100 =15.38 %
30
Quantity II: 625 =125% complete the work = = 15 days
2
125
Therefore, 425 =625 × 435 = 87% Hence, Quantity I < Quantity II
Loss % = 100 – 87=13 % 80. (a); Shikhar Manan Rohit Krunal
80 90
Hence, Quantity I > Quantity II 100 150 150 × 100 =120 120 × 100=108
78. (c); Let length of the platform be x metres 50 75 60 54
5 Quantity I:
Speed of the train = 54 × =15 m/s 60−50
18 Required percentage = 50 × 100 =20 %
According to the question, train takes 12 sec to 54−50
cross a man on the platform Quantity II: 50 × 100= 8%
So, length of the train =12 × 15 =180 m Hence, Quantity I > Quantity II

ENGLISH LANGUAGE

81. (c); Among the given words, alternatives (I) and (IV) grammatically correct and contextually coherent
suitably fit in the blank and form a grammatically sentence. All the other words fail to adhere,
correct and contextually coherent sentence. All either the grammatical syntax or the context of
the other words fail to adhere, either the the sentence. Hence, option (b) is the most
grammatical syntax or the context of the suitable answer choice.
sentence. Hence, option (c) is the most suitable Benevolent means well-meaning and kindly.
answer choice. Apathetic means showing or feeling no interest,
Penury means the state of being very poor; enthusiasm, or concern.
extreme poverty. Passive means accepting or allowing what
Mansion means a large, impressive house. happens or what others do, without active
Grandeur means splendour and impressiveness, response or resistance.
especially of appearance or style. Indifferent means having no particular interest
Poverty means the state of being extremely poor. or sympathy; unconcerned.
82. (d); Among the given words, alternatives (I) and (II) 84. (a); Among the given words, only alternative (III)
suitably fit in the blank and form a grammatically suitably fits in the blank and form a
correct and contextually coherent sentence. All grammatically correct and contextually
the other words fail to adhere, either the coherent sentence. All the other words fail to
grammatical syntax or the context of the adhere, either the grammatical syntax or the
sentence. Hence, option (d) is the most suitable context of the sentence. Hence, option (a) is the
answer choice. most suitable answer choice.
Alleviate means make (suffering, deficiency, or a Abundant means existing or available in large
problem) less severe. quantities; plentiful.
Aggravate means make (a problem, injury, or Pernicious means having a harmful effect,
offence) worse or more serious. especially in a gradual or subtle way.
Placid means not easily upset or excited.
83. (b); Among the given words, alternatives (I), (III) and Injurious means causing or likely to cause
(IV) suitably fit the blank and form a damage or harm.

19 Adda247 Publications For any detail, mail us at


Publications@adda247.com
50+ Bank PO | Clerk Previous Year’s Papers 2016 – 2020

85. (e); All the given words suitably fit in the blank 92. (d); Both (a) and (b) options are correct in context of
forming a grammatically correct and the passage.
contextually coherent sentence. Hence, option Existing road network got affected with
(e) is the most suitable answer choice. implementation of Bharatmala Pariyojana. This
programme affect the toll collection and thereby
Achieved means successfully bring about or debt servicing ability. Hence option (d) is the
reach (a desired objective or result) by effort, correct choice. Sentence (c) is incorrect as it is
skill, or courage. not mentioned anywhere in the passage.
Accomplished means achieve or complete Refer the lines “The Bharatmala programme
successfully. may result in traffic diversion from the existing
Gained means obtain or secure (something road network to new roads, thereby affecting the
wanted or desirable). toll collection and, consequently, the debt
Attained means succeed in achieving (something servicing ability of some of the BOT and OMT
projects.”
that one has worked for). “Out of the 44 ECs, about 21 would partially or
86. (d); IL&FS has appointed advisers to prepare a fully affect the existing alignments, while the
resolution plan, (C) and (E) makes a perfect remaining 23 that involve upgradation of
match. existing alignment will not result in any
deviation.”
The company along with its subsidiaries is facing
a liquidity crisis, (A) and (F) makes a perfect 93. (c); The total debt at risk is Rs19,435 crore. It has
match. mentioned in the passage that debt servicing
ability has raised the risk of default on 25
87. (d); For big companies, there are instances of even national highway toll projects involving
the infrastructure getting damaged, Sentence (C) Rs19,435 crore of debt.
and (F) makes proper combination as a Hence option (c) is the most appropriate choice.
sentence. The increase in MSP for rabi crops Refer the line of second paragraph “This has
raised the risk of default on 25 national highway
comes just ahead of the RBI monetary policy
toll projects which involve Rs19,435 crore of
announcement, (A) and (E) makes the perfect debt.”
match as in sentence.
94. (b); “Bharatmala and the rising debt issues” is an
88. (c); Only sentence (A) and (D) makes a perfect match appropriate title of the passage.
as a sentence, India will be the third largest 95. (b); Refer the fourth paragraph of the passage in
aviation market globally a year sooner. which it is clearly mentioned that the need of the
89. (b); Only sentence (B) and (F) makes a perfect match hour is to realign the terms and conditions of the
as they both together states that “All the sectoral model concession agreement to ensure that
banks do not end up accumulating NPAs .Now in
indices closed in the red, with 22 of the 30 the third paragraph, refer the lines “This could
Sensex stocks finishing lower.” be ensured by allowing renegotiation of the
90. (c); Only sentence (A) and (F) makes a perfect match terms of the concession agreement.”. Hence
option (b) is the correct choice. Sentence (I) is
as a sentence, “A writer needs quiet and peaceful
irrelevant here.
environment around him in order to write.”
96. (c); ‘do not turn’ is the correct phrase to be replaced
91. (d); We can conclude from first paragraph of the thus making the sentence grammatically correct.
passage that sentences (I) and (II) are in The sentence is in present tense ‘are needed’,
relevance with the passage. As mentioned in the ‘are at risk’ Hence ‘could’ and ‘would’ will not be
paragraph that the goal is to create new used as they are past form of ‘can’ and ‘will’. The
industries, more employment and new markets. subject of the sentence ‘BOT projects’ is plural
Sentence (III) is totally irrelevant in context to hence, ‘does’ will not be used. Hence ‘do not turn’
is the correct choice making option (c) as
the passage.
correct.
Hence option (d) is the correct choice.
Refer the lines “The programme focuses on 97. (c); Curtail means reduce in extent or quantity;
optimizing the efficiency of road traffic impose a restriction on. Hence (I), (II) and (III)
are similar in meaning to Curtail.
movement across the country by bridging
critical infrastructure gaps through shorter 98. (d); Embarked means begin (a course of action).
routes. The end goal is to create economic Hence it has same meaning as commence,
corridors (ECs) along the path—new industries, initiate and undertake.
Commence means begin.
more employment and new markets.” Rot means decay or cause to decay.

20 Adda247 Publications For any detail, mail us at


Publications@adda247.com
50+ Bank PO | Clerk Previous Year’s Papers 2016 – 2020

99. (a); Hindrance means a thing that provides information on the note issued by him stating the
resistance, delay, or obstruction to something or three troubles that could be caused by increasing
someone. Hence it has opposite meaning as that non-performing assets. Next should be
to impetus which means something that makes statement (G), as it provides information on the
a process or activity happen or happen more troubles mentioned in statement (E). Since
quickly. statement (C) provides data on the outstanding
Plausible means in a way that seems reasonable loan it should be following statement (G).
or probable. Statement (F) provides information on the
Contrite means feeling or expressing remorse at mudra loans mentioned in statement (C).
recognition that one has done wrong. Statement (H), quotes the problem and the
solution to recover the small loans. Statement
100. (b); Reluctant means unwilling and hesitant;
(D) coherently connects with statement (H)
disinclined. Hence it has opposite meaning as
forming a meaningful paragraph. Among the
willing and eager.
given sentences, only statement (B) is describing
Splendid means magnificent, very impressive.
about the lack of professionalism on the part of
Vaunt means boast about or praise.
Banks while issuing loans. It is to be noted that
101. (a); ‘Why do we not’ will come in place of ‘why we do sentence (B) begins with “Clearly” which is
not’ as the sentence is in interrogative form. indicating that something similar in context
102. (c); In place of ‘he will have’, ‘he would have been’ should have been mentioned in the earlier
will be used. statements. Therefore, statement (B) is
incoherent. Thus the sequence formed is
With Had+ subject + been + V3, Subject + would
AEGCFHD. Since the last statement, statement
+ have + been + V3 is used.
Ex. Had he been taken to hospital, he would (D) is describing about the problems associated
have been saved. with the banks to recover the loans, the next
statement should option (b) as it is further
103. (b); Replace ‘with’ by ‘in’ as “take delight in describing about the risk if these small loans
something” is the correct usage. cumulatively form a huge amount. Sine all the
e.g. She takes delight in dancing. other statements are incoherent, option (b) is
104. (b); ‘hard’ will be used in place of ‘hardly’ as ‘hardly’ the most suitable answer choice.
is used to depict ‘rarely’ or ‘scarcely’ whereas 108. (e); Drawing a hint from statement (A), it can be
here ‘hard’ means ‘tough’. understood that the paragraph is describing
105. (e); The sentence is grammatically correct. about an alarming note issued by Former RBI
Governor Raghuram Rajan regarding the
106. (c); Drawing a hint from statement (A), it can be cumulative bad-debts which can lead to financial
understood that the paragraph is describing crisis. Statement (E) should logically follow
about an alarming note issued by Former RBI statement (A) as it is further providing
Governor Raghuram Rajan regarding the information on the note issued by him stating the
cumulative bad-debts which can lead to financial three troubles that could be caused by increasing
crisis. Among the given sentences, only non-performing assets. Next should be
statement (B) is describing about the lack of statement (G), as it mentions all the three
professionalism on the part of Banks while troubles mentioned in statement (E). Therefore,
issuing lo It is to be noted that sentence (B) statements (E) and (G) form a logical pair. Hence,
begins with “Clearly” which is indicating that option (e) is the most suitable answer choice.
something similar in context should have been 109. (a); Drawing a hint from statement (A), it can be
mentioned in the earlier statements. Since, none, understood that the paragraph is describing
of the statement can find relevance with about an alarming note issued by Former RBI
statement (B), option (c) becomes are most Governor Raghuram Rajan regarding the
suitable answer choice. cumulative bad-debts which can lead to financial
107. (b); Drawing a hint from statement (A), it can be crisis. Statement (E) should logically follow
understood that the paragraph is describing statement (A) as it is further providing
about an alarming note issued by Former RBI information on the note issued by him stating
Governor Raghuram Rajan regarding the the three troubles that could be caused by
cumulative bad-debts which can lead to financial increasing non-performing assets. Next should
crisis. Statement (E) should logically follow be statement (G), as it provides information on
statement (A) as it is further providing the troubles mentioned in statement (E). Since

21 Adda247 Publications For any detail, mail us at


Publications@adda247.com
50+ Bank PO | Clerk Previous Year’s Papers 2016 – 2020

statement (C) provides data on the outstanding 112. (e); ‘embedded’ is the correct word to be filled in the
loan it should be following statement (G). blank.
Statement (F) provides information on the According to the theme of the paragraph,
mudra loans mentioned in statement (C). ‘embedded’ is the correct word.
Statement (H), quotes the problem and the
solution to recover the small loans. Statement 113. (b); ‘forms’ is the correct word.
(D) coherently connects with statement (H) 114. (c); ‘generation’ is the correct word.
forming a meaningful paragraph. Among the
As production is used in the sentence, hence we
given sentences, only statement (B) is describing
can get the idea that generation will be the
about the lack of professionalism on the part of
Banks while issuing loans. It is to be noted that answer.
sentence (B) begins with “Clearly” which is 115. (c); ‘lost’ is the correct word.
indicating that something similar in context Awed means filled with awe or wonder.
should have been mentioned in the earlier Misled means cause (someone) to have a wrong
statements. Therefore, statement (B) is idea or impression.
incoherent. Thus the sequence formed is
Rapt means completely fascinated or absorbed
AEGCFHD. Hence, option (a) is the most suitable
answer choice. by what one is seeing or hearing.

110. (b); Drawing a hint from statement (A), it can be 116. (a); ‘feed’ is the correct word to be filled in the blank.
understood that the paragraph is describing Organic wastes are useful for the use of small
about an alarming note issued by Former RBI and affordable households.
Governor Raghuram Rajan regarding the
117. (e); ‘use’ is the correct word.
cumulative bad-debts which can lead to financial
crisis. The logical sequence of the sentences after 118. (c); “prevent’’ is the correct word.
rearrangement excluding the incoherent one is
AEGCFHD. Since statement (C) is the fourth 119. (d); ‘ingested’ is the correct word to be filled in the
statement in the sequence, option (b) is the most blank.
suitable answer choice. Ravaged means severely damaged.

111. (d); ‘absence’ is the correct word. 120. (b); ‘refuse’ is the correct word.
Desertion means abandonment, leaving. As refuse is also used elsewhere in the
Paucity means the presence of something in paragraph, we can get a hint.
only small or insufficient quantities or amounts. Refuse means worthless and useless part of
Sparsity means to scatter or spread. something.

22 Adda247 Publications For any detail, mail us at


Publications@adda247.com
50+ Bank PO | Clerk Previous Year’s Papers 2016 – 2020

1 Adda247 Publications For any detail, mail us at


Publications@adda247.com
50+ Bank PO | Clerk Previous Year’s Papers 2016 – 2020

Mock IBPS RRB PO Mains 2018


36
REASONING ABILITY

Directions (1-5): Study the information carefully and 6. Statement: The State Government has decided to
answer the questions given below. declare ‘Kala Azar’ as a notifiable disease under the
Epidemics Act. Family members or neighbours of the
Seven persons A, B, C, D, E, F and G are born in different
patient are liable to be punished in case they did not
years 1950, 1953, 1961, 1968, 1972, 1985, 1996 but not
inform the State authorities.
necessarily in the same order. They are born on same date
Courses of action:
of the same month in different years. (All the calculations
I. Efforts should be made to effectively implement
of ages are done by considering 2018 as the base year)
the Act.
They like different fruits i.e. grapes, litchi, fig, mango,
II. The cases of punishment should be propagated
guava, apple, pomegranate.
through mass media so that more people become
The sum of the present age of B and E is 90 years. The aware of the stern actions.
person who is 2nd eldest likes mango. C is younger to F. The (a) If only I follow
sum of the present age of G and the one who likes guava is (b) If only II follow
103 years. Neither C nor A likes grapes and fig. The (c) If either I or II follow
difference between the present ages of the one who likes (d) If neither I nor II follow
guava and the one who likes fig is 11 years. The number of (e) If both I and II follow
persons born before A is same as the number of persons
7. Statement- Two companies A and B signed a deal in
born after the one who likes apple. The youngest person
which both have agreed to not disclose any
likes pomegranate. The sum of the present age of D and A
confidential details of their company to the rivals.
is 115 years. A is younger to D. The one who likes litchi is
Which of the following could be the reason of the legal
older to the one who likes grapes. The number of persons
notice sent by company B to company A for breaching
born between the one who likes guava and grapes is more
the contract?
than the persons born between the ones who like grapes
I. Company A is unlawfully receiving more amount
and litchi. G does not like fig and B is younger to E.
of profit without indulging company B in it.
II. Company A is sharing the private data and
1. Who among the following likes fig?
details of company B for its own profit.
(a) G (b) F (c) C
(a) Only I is implicit
(d) B (e) none of these
(b) Only II is implicit
2. Which among the following pair of persons are the (c) Either I or II is implicit
oldest and youngest respectively? (d) Neither I nor II is implicit
(a) E, A (b) F, C (c) C, F (e) Both I and II are implicit
(d) B, D (e) none of these
Directions (8-12): Study the information carefully and
3. In which of the following year was E born? answer the questions given below.
(a) 1968 (b) 1950 (c) 1961
Ten people are seating on two parallel rows. P, Q, R, S and
(d) 1985 (e) none of these
T are seating in row 1 and facing towards south. A, B, C, D
4. Which of the following statement is true regarding F? and E are seating in row 2 and facing north. They like
(a) E was born before F different Colors- pink, blue, black, yellow and orange. Two
(b) F is the youngest persons like same color such that one of them is in row 1
(c) the difference between the present ages of G and and the other is in row 2(i.e. both the persons who like Pink
F is divisible by 3 colour sit in different rows)
(d) F likes guava
R sits second from one of the ends. The one who sits
(e) none of these
immediate right of R faces C. P sits right to T but not
5. How many persons were born between A and E? immediate right. The number of persons sitting to the left
(a) Two (b) One (c) Three of C is same as the number of persons sitting to the right of
(d) Four (e) None T. D faces the one who likes yellow. The one who likes pink

2 Adda247 Publications For any detail, mail us at


Publications@adda247.com
50+ Bank PO | Clerk Previous Year’s Papers 2016 – 2020

in row 1 faces the one who sits 2nd to the left of the one who 16. Which of the following is the nearest point to X, which
likes pink in row 2. Only one person sit between S and the is the mid-point of BC?
one who likes orange. The one who likes black sits 3rd to (a) E (b) F (c) C
the left of the one who likes pink in row 2. More than two (d) B (e) H
person sits between D and A. E does not like pink. C likes 17. If in the number 68942735, 1 is added to second,
yellow. R does not like orange and pink. S faces the one who fourth, sixth and eighth digit and 1 is subtracted from
likes blue. The one who likes yellow sits immediate left to the first, third, fifth and seventh digits, then how many
R. P does not like black. digits are repeated in the new number thus formed?
8. Who among the following likes blue in row 1? (a) 5 and 9 (b) 5 and 8 (c) 6, 5 and 8
(a) P (b) Q (c) R (d) 9, 1 and 3 (e) None of these
(d) S (e) none of these 18. Which alphabet is 3rd from the left in the meaningful
9. How many persons sit between E and the one who four letter word formed from the first, second,
likes black? seventh and eighth letter of the word PRODUCTION?
If more than one word is formed then mark answer as
(a) Two (b) One (c) Three
X and no meaningful word is formed then mark
(d) Four (e) None
answer as Z.
10. Which of the following statement is true regarding S? (a) T (b) X (c) P
(a) E faces S (d) Z (e)
(b) A likes same color as S
Directions (19-23): Study the following information
(c) D faces S
carefully and answer the given questions.
(d) B sits 3rd left to the one who faces S
(e) none of these Nine persons A, B, C, D, E, F, G, H, I like different flowers
tulip, lily, rose, lotus, sunflower, orchid, iris, Tithonian and
11. Which of the following color is liked by R? Cosmos also each of them live in three different cities
(a) blue (b) black (c) orange Patna, Surat and Mumbai. Not less than two and not more
(d) yellow (e) none of these than four live in same cities.
12. Four of the following belongs to a group based on G lives with only the one who likes tulip. B lives with H and
their seating arrangement find the one that does not the one who likes Lotus D and F lives with the one who likes
belong to that group? Sunflower. The one who likes Tithonian does not lives in
(a) T (b) C (c) R Surat. The one who likes Lily and the one who likes Orchid
(d) B (e) A live in the same city but not in Patna. Neither A nor H likes
Directions (13-16): Study the information carefully and Tithonian. The one who likes Iris lives with A and I. E lives
answer the questions given below. in Surat but does not like Lily. G does not live in Patna. H
does not like Iris. A does not like Rose and Lotus. D does
A person starts walking from A towards to reach B, which not like Orchid. The one who like Rose and the one who like
is 15m east to A. He then takes a right turn and walks 20 m Cosmos live in same city. Neither A nor I likes Sunflower.
to reach point C. From C, he takes left turn and walks15m
to point D, then again he takes a left turn and walks 10m to 19. Who among the following like Tulip?
point E. From E, he takes a left turn and walks 20m to reach (a) C (b) H (c) B
point F. He then takes a right turn and walks 5m to reach G (d) F (e) None of these
and finally takes a left turn to reach point H, which is 10 m 20. In Which of the following city D and E live
away from G. respectively?
13. If J is 5m north to G, then what is the distance between (a) Mumbai and Surat
A and J? (b) Both in Patna
(a) 5m (b) 10m (c) 15m (c) Patna and Surat
(d) Both in Surat
(d) 20m (e) none of these
(e) None of these
14. Four of the following five belongs to a group based on
21. A likes which of the following flower?
their directions find the one that does not belong to
(a) Lily (b) Tithonian (c) Cosmos
that group?
(d) Orchid (e) None of these
(a) A, G (b) F, C (c) B, D
(d) H, F (e) D, E 22. Four of the following are alike in a certain way so form
a group which of the following does not belong to that
15. How far is point A from point H? group?
(a) 5m (b) 10m (c) 15m (a) B (b) H (c) A
(d) 20m (e) none of these (d) F (e) I
3 Adda247 Publications For any detail, mail us at
Publications@adda247.com
50+ Bank PO | Clerk Previous Year’s Papers 2016 – 2020

23. C lives with which among the following? 28. Which of the following would be the difference of the
(a) B and H (b) D, F and E (c) A, I and E numbers which is 3rd from left end in step IV and 2nd
(d) Only G (e) None of these from right end in Step II?
Directions (24-25): In each question below is given a (a) 90 (b) 83 (c) 25
statement followed by two assumptions numbered I and II. (d) 10 (e) None of these
You have to consider the statement and the following
assumptions and decide which of the assumptions is 29. Which of the following element will be 6th to the left
implicit in the statement. of 3rd from the right end in step V?
Give answer- (a) 95 (b) adopt (c) 40
(a) If only assumption I is implicit. (d) scrap (e) None of these
(b) If only assumption II is implicit.
(c) If either I or II is implicit. 30. In which of the following step ’39 95 warm’ would be
(d) If neither I nor II is implicit. find in the same order?
(e) If both I and II are implicit. (a) Step V
24. Statement: The ‘M’ Cooperative Housing Society has (b) Step II and III
put up a notice at its gate that sales persons are not (c) Step II
allowed inside the society. (d) Step III and IV
Assumptions: (e) None of these
I. All the sales persons will stay away from the ‘M’
cooperative Housing Society. Directions (31-35): Study the following information
II. The security guard posted at the gate may be carefully and answer the given questions.
able to stop the sales persons entering the
Seven members of a family are sitting in a row facing north.
society.
G is third from one of the ends. G’s mother is second to the
25. Statement: “If it does not rain throughout this month, right of G. G’s wife is immediate left of G. M is an immediate
most farmers would be in trouble this year.” neighbour of G’s mother. B is third to the left of M. Only one
Assumptions:
person sit between D and M. D’s father sit second to the left
I. Timely rain is essential for farming.
II. Most farmers are generally dependent on rains. of D. Only one person sit between L’s daughter and M. B is
a male. B’s father is third to the left of L’s daughter. V is D’s
Directions (26-30): Study the following information to son and Q is daughter-in-law of L. Q sits to the left of L.
answer the given questions:
A word and number arrangement machine when given an 31. How many persons are sitting between Q and L?
input line of words and numbers rearranges them (a) One
following a particular rule. The following is an illustration (b) Three
of input and rearrangement. (c) More than three
Input: 78 former 44 quicker colder 65 taller outer 27 8 (d) Two
Step I: cold former 44 quicker 65 taller outer 27 8 79 (e) None
Step II: 9 cold former 44 quicker 65 outer 27 79 tall
Step III: form 9 cold 44 quicker outer 27 79 tall 66 32. How many persons sit to the left of L?
Step IV: 28 form 9 cold 44 quicker 79 tall 66 out (a) One
Step V: quick 28 form 9 cold 79 tall 66 out 45 (b) Three
Step V is the last step of the rearrangement. As per the rules (c) More than three
followed in the above steps, find out in each of the (d) Two
following questions the appropriate steps for the given
(e) None
input.
Input: warmer 76 94 scraper locker 11 greater 52 39 33. Who among the following is the grandson of M?
adopter (a) D and B (b) L (c) V and B
26. How many steps would be needed to complete the (d) Q and D (e) D
arrangement?
(a) IV (b) V (c) VII 34. How is D related to G?
(d) VI (e) None of these (a) Sister (b) Brother (c) Mother
(d) Father (e) Sister-in-law
27. What will the addition of the numbers which is third
from the right end in step II and 2nd from the right 35. Who among the following sit forth to the right of B’s
end in step IV? father?
(a) 112 (b) 210 (c) 151 (a) M (b) L (c) V
(d) 116 (e) None of these
(d) Q (e) D
4 Adda247 Publications For any detail, mail us at
Publications@adda247.com
50+ Bank PO | Clerk Previous Year’s Papers 2016 – 2020

Directions (36-38): Each of the questions below consists Direction (39-40): Statement- The CMC products are
of a question and two statements numbered I and II given quite popular and has been commonly used by the people
below it. You have to decide whether the data provided in of city X, as it became so popular among the people who are
the statements are sufficient to answer the question. Read
getting health benefits from it. But recently we have seen a
both the statements and;
(a) if the data in statement I alone are sufficient to answer sudden decline in the demand of the products.
the question, while the data in statement II alone are 39. Which of the following can be concluded from the
not sufficient to answer the question.
given statement?
(b) if the data in statement II alone are sufficient to
answer the question, while the data in statement I (a) The CMC products are not so popular in city Y.
alone are not sufficient to answer the question. (b) The CMC products with so many health benefits
(c) if the data either in statement I alone or in statement are very famous and in regular use by the people
II alone are sufficient to answer the question. of city X.
(d) if the data even in both statements I and II together (c) The quality of the products is not that good as it
are not sufficient to answer the question. was initially of the CMC products.
(e) if the data in both statements I and II together are
(d) A rival company which has come up with its
necessary to answer the question
product with lower price are becoming popular
36. In a family of six persons who is the father of E? among the people of city X.
I. C is the son-in-law of B. A is the only son of D. D
(e) The price of CMC products has been increased
is the grandmother of E.
II. E is the son of X, who is the only sister of A. B is significantly in a decade.
the father of X. 40. Which of the following can be inferred about the
37. Five persons are sitting around a circular table facing decrease in demand of CMC products?
the center. Who among the following sit immediate (I) The quality of products offered by CMC reduced
left of C? which decreases the demand of the products.
I. B sits second to the right of D. A second to the
(II) The prices are hiked of CMC products after
right of B. Two persons sit between D and E.
II. D sits second to the left of B. E is not an seeing its popularity which in turn reduces its
immediate neighbour of D. C is second to the left demand.
of E. (III) The people of city X do not need the healthy
38. Six persons P, Q, R, S, T, U were born in six different products now.
months January, March, May, July, August and October (IV) The people of city X are now much more
of a year. Only one person was born in one month. influenced with the foreign products, which
Who among the following born in a month affects the demand of CMC products.
immediately before T? (a) Only I and II
I. P was born after July. Only one person born
(b) Only III
between S and P. Q was born immediately before
P. R was born after P. T was born before S. (c) Only II and IV
II. S was born in May. P was born after S. Only two (d) All of the above
person were born between P and T. (e) Only I

QUANTITATIVE APTITUDE

41. In a bag, there are 8 black balls and 12 yellow and 42. A hemisphere is mounted on cylinder and the radius
green balls. If the probability of choosing a green ball of both is same i.e. 7 cm. Find the height of cylinder if
from the bag is 0.35 then, find the probability of 2
the total volume of hemisphere and cylinder is 3952
choosing two yellow balls from the bag if the ball 3
chosen is not replaced. cm³.
2 4 1 1
(a) 19 (b) 95 (c) 19 (a) 21 cm (b) 14 cm (c) 17 2 cm
6
(d) 95 (e) None of these (d) 28 cm (e) None of these

5 Adda247 Publications For any detail, mail us at


Publications@adda247.com
50+ Bank PO | Clerk Previous Year’s Papers 2016 – 2020

Directions (43-47): The given table shows the number of classes taken by four different guest tutors (A, B, C and D) on
different days in a week and honorarium paid to them for each class.
Tutors Number of classes Number of class Honorarium
taken each on taken each on paid for each
Monday, Tuesday and Thursday & class
Wednesday Friday (In Rs)
A 2 0 5000
B 3 — 8000
C 1 3 6000
D 2 2 4000
Note:
(i) Saturday and Sunday are Holiday.
(ii) ‘—’ is missing value
(iii) Each class is of one hour.
43. Find the ratio of number of classes taken by A to the 49. A shopkeeper allows a discount of 30% on marked
number of classes taken by D in a week? price of an article and customer also managed to get
(a) 4 : 5 (b) 3 : 5 (c) 2 : 3 additional 10% discount on discounted price and in
(d) 9 : 10 (e) None of the above this way get a total discount of Rs.925. If shopkeeper
1
44. Find the number of classes taken by C if he is paid an incurs a loss of 12 2% on selling the article then find
honorarium of Rs. 3.78 lakh for the completion of his its cost price for the shopkeeper?
course. (a) Rs. 1600 (b) Rs. 1800 (c) Rs. 1500
(a) 45 (b) 54 (c) 72 (d) Rs. 1925 (e) Rs. 2050
(d) 81 (e) 63 50. A and B started a business by investing Rs. 10,000 and
45. Find the difference between the honorarium paid to Rs. 9,000 respectively. But after 8 month, A and B both
C, if he teaches for 3 weeks and the honorarium paid withdrew their whole amount and C joined the
to D, if he teaches for 2 weeks? business with Rs. 12,000. If at the end of year,
(a) Rs. 82,000 (b) Rs. 74,000 (c) Rs. 88,000 difference between profit share of A and B is Rs. 576.
(d) Rs. 90,000 (e) Rs.80,000 Then, find total profit earned in a year ?
(a) Rs. 13,800 (b) Rs. 14,250 (c) Rs. 14,400
46. If honorarium paid to B for the completion of his (d) Rs. 14,950 (e) Rs. 15,500
course is Rs. 5.28 lakh and B takes 6 weeks for
completion of his course then honorarium paid to B Direction (51-55): Data given below shows total expense
and D together in 2 weeks is approximately what of a company in three different years.
percent of the honorarium paid to C in 4 weeks ? 2015 → Total salary expense of 80 employees is 10 lakhs
(a) 105% (b) 110% (c) 114% 2016 → Total expense in this year is same as previous year
(d) 120% (e) 125% while salary expense is increased by 26% as compared to
previous year and average salary expense is decreased by
47. If B takes no classes on Thursday and Friday in a Rs.500 as compared to previous year.
particular week then honorarium paid to A is what 2017 → Total expense is 60 lakhs which is 20% more than
percent of that of B in that week ? that in 2016. Others expense is 47.5 lakhs. Number of
(a) 38 ⅓% (b) 42 ⅓% (c) 41 ⅔% employees is decreased by 5 as compared to previous year.
(d) 45% (e) 46 ⅔% Total expense = Salary expense + Others expense
48. 40 men together complete a work in 12 days while 12 Salary expense = Number of employees × Average
women together complete that work in 48 days. All salary expense
men started working together and on every next day 51. Find the ratio between Others expense in 2015 to
10 men left the work and after four days, 16 women Salary expense in 2017?
joined to do the remaining work then find the time (a) 1 : 4
taken by 16 women to do the remaining work? (b) 4 : 1
1
(a) 25 days (b) 22 2 days (c) 30 days (c) None of the given options
1 1 (d) 5 : 16
(d) 28 2 days (e) 32 2 days
(e) 16 : 5

6 Adda247 Publications For any detail, mail us at


Publications@adda247.com
50+ Bank PO | Clerk Previous Year’s Papers 2016 – 2020

52. Avearage salary expense in 2015 is what percent less 7 6


60. I. 2 − x + x2 = 0
than that in 2017? 15 22
(a) None of the given options II. – +8=0
y2 y
(b) 10% (a) x ≤ y
(c) 15% (b) x > y
(d) 5% (c) x ≥ y
(e) 25% (d) no relation can be established between x and y
53. If average employees in 2015, 2017 and 2018 is 82. (e) x < y
Find number of employees in 2018? 1 1
(a) 86 61. Boat A travels 62 km in upstream in 2 hrs. The
2 2
(b) None of the given options speed of another boat B in still water is 80% of the
(c) 76 speed of boat A in downstream and the speed of
(d) 66 stream for both boat A and B is same i.e. 2.5 km/hr.
(e) 56 Find the distance travelled by boat B in upstream in 4
54. Female employees in 2016 is 45 more than male hours.
employees in 2016. Find the ratio between male to (a) 82 km (b) 86 km (c) 88 km
female employees in 2016? (d) 84 km (e) 94 km
(a) 11 : 9 (b) 5 : 2 (c) 2 : 5
(d) 9 : 11 (e) None of the given options 62. Ratio of A’s age 4 years later to B’s age 6 years later is
2 : 1. Another person C whose present age is twice the
55. Find the difference between Salary expense in 2017
sum of present age of A and B together and after 4
to Salary expense in 2016?
(a) 10,000 (b) 90,000 (c) 60,000 years ratio of C’s age to B’s age is 23 : 4 then find the
(d) 40,000 (e) None of the given options present age of B ?
Directions (56-60): In each question two equations (a) 16 years (b) 10 years (c) 15 years
numbered (I) and (II) are given. Student should solve both (d) 12 years (e) 18 years
the equations and mark appropriate answer. Direction (63-67): Pie charts given below shows number
56. I. x³ = – 512 of tourists applied for visa from five different cities. First
II. 2y² + 20y + 48 = 0 pie chart shows number of tourists applied for U.S.A +
(a) x ≥ y CANADA + ENGLAND while second pie chart shows
(b) no relation can be established between x and y.
(c) x > y number of tourists applied for U.S.A. Study the data
(d) x < y carefully and answer the following questions.
(e) x ≤ y Tourist (U.S.A + CANADA
57. I. x² + 47 = 72 + ENGLAND) = 5500
II. (y – 2)² = 3 (y – 2)
(a) x ≤ y A,
(b) x > y E, 18%
(c) x ≥ y 34%
(d) x < y B,
(e) no relation can be established between x and y. 20%
58. I. 2x² + 3x + 1 = 0 D, C,
1
II. 2 + + y = 0 16% 12%
y
(a) no relation can be established between x and y
(b) x ≥ y Tourist (U.S.A) = 2500
(c) x ≤ y
(d) x > y
(e) x < y E, A,
22% 22%
59. I. (x – 5)² = 16
II. y² + 8y = 84
(a) x ≥ y D,
B,
(b) x < y 18%
C, 24%
(c) no relation can be established between x and y
(d) x > y 14%
(e) x ≤ y

7 Adda247 Publications For any detail, mail us at


Publications@adda247.com
50+ Bank PO | Clerk Previous Year’s Papers 2016 – 2020

63. Ratio between number of tourists applied for Directions (70-74): The following questions are
CANADA to ENGLAND is 57 : 29 from city D. Find accompanied by two statements I and II. You have to
number of tourists applied for CANADA from city D is determine which statements (s) is / are sufficient /
approximately what percent of the total tourists necessary to answer the questions.
applied for visa from city D? 70. If there are only three members (Father, Mother and
(a) 28% (b) 32% (c) 54% Daughter) in the family then find the age of daughter?
(d) 38% (e) 44% I. If two years later the average age of all members
64. Find the central angle of total tourists applied from of the family will be 28 years and six years ago
city E? average age of Father and Mother was 30 years.
(a) 129.6° (b) 112.4° (c) 115.2° II. If two years ago ratio of age of Father to that of
(d) 100.8° (e) 122.4° Daughter is 9 : 1 and Father’s present age is four
years more than Mother’s present age and sum
65. Tourists applied for Canada from city A is 120 more of their ages is 72 years.
than tourists applied for England from city A. Find (a) Statement I alone is sufficient to answer the
total number of tourists applied for Canada from city question but statement II alone is not sufficient to
A? answer the question.
(a) 160 (b) Statement II alone is sufficient to answer the
(b) None of the given options question but statement I alone is not sufficient to
(c) 180 answer the question.
(d) 280 (c) Both the statements taken together are necessary
(e) 260 to answer the questions, but neither of the
statements alone is sufficient to answer the
66. Find the difference between number of tourists question.
applied for Canada and England from city D and (d) Either statement I or statement II by itself is
Number of tourists applied for Canada and England sufficient to answer the question.
from city E? (e) Statements I and II taken together are not
(a) 880 (b) 990 (c) 890 sufficient to answer the question.
(d) None of the given options (e) 790
71. Two persons Veer and Bhavya started a business by
67. Out of total tourist applied for U.S.A from city B and investing some amounts. Find investment of Veer.
city E, 24% and 72% respectively went to U.S.A. Find I. Veer and Bhavya together invested Rs 9,000 in
the ratio between number of tourists went to U.S.A the business and Veer left after one year.
from city B to city E? II. Bhavya invested Rs 2000 more next year and got
(a) 4 : 11 profit share of Rs 3400 out of total profit Rs
(b) None of the given options 5950.
(c) 5 : 22 (a) Statement I alone is sufficient to answer the
(d) 7 : 11 question but statement II alone is not sufficient
(e) 6 : 11 to answer the question.
(b) Statement II alone is sufficient to answer the
68. A person invested in scheme A at X% per annum at SI question but statement I alone is not sufficient to
for 5 years and in scheme B at 10% per annum for 2 answer the question.
years at CI. If the amount invested in both schemes are (c) Both the statements taken together are
in the ratio of 2 : 5 (A : B) and the total interest necessary to answer the questions, but neither of
received from both the schemes is 35% of the total the statements alone is sufficient to answer the
amount invested together in both the schemes then question.
find value of X ? (d) Either statement I or statement II by itself is
(a) 14% (b) 16% (c) 21% sufficient to answer the question.
(d) 28% (e) 35% (e) Statements I and II taken together are not
sufficient to answer the question.
69. Ruchi’s monthly salary is Rs. 60,000. She spends Rs.
12,000 on education of her child and Rs. X on her 72. A person travels from point A to B and back from
𝟏 𝟏 point B to A. Find average speed of the person during
house rent. Further she spends 𝟕 and 𝟔 of the whole journey if distance between A to B is 100km.
remaining amount on Food and shopping needs and I. Person takes 30 min more while coming back to
the remaining Rs. 29,000 invested in some scheme. the initial point.
Then find the value of X. 2
II. Speed of person from A to B is 6 3 km /hr more
(a) Rs. 7,000 (b) Rs. 13,000 (c) Rs. 16,000
than that of from B to A and ratio of time taken
(d) Rs. 6,000 (e) None of these
from A to B to that of B to A is 5 : 6.

8 Adda247 Publications For any detail, mail us at


Publications@adda247.com
50+ Bank PO | Clerk Previous Year’s Papers 2016 – 2020

(a) Statement I alone is sufficient to answer the (c) Both the statements taken together are
question but statement II alone is not necessary to answer the questions, but neither of
sufficient to answer the question. the statements alone is sufficient to answer the
(b) Statement II alone is sufficient to answer the question.
question but statement I alone is not (d) Either statement I or statement II by itself is
sufficient to answer the question. sufficient to answer the question.
(c) Both the statements taken together are (e) Statements I and II taken together are not
necessary to answer the questions, but sufficient to answer the question.
neither of the statements alone is sufficient to
74. Find time taken by A to cross B, if B has a head start of
answer the question.
30m and they run in same direction at same time?
(d) Either statement I or statement II by itself is
I . Speed of B is twice of relative speed of A and B.
sufficient to answer the question.
II . If B started 2 seconds earlier, then A took 16
(e) Statements I and II taken together are not
seconds to cross B.
sufficient to answer the question.
(a) Statement I alone is sufficient to answer the
73. Find the rate of interest p.c.p.a (take rate of interest question but statement II alone is not sufficient
same for Simple interest and Compound interest)? to answer the question.
I. When Rs. 25,000 invested for 2 years at that rate (b) Statement II alone is sufficient to answer the
of interest, then compound interest is Rs.250 question but statement I alone is not sufficient to
more than simple interest. answer the question.
II. After 10 years amount will be double if invested (c) Both the statements taken together are
at Simple interest. necessary to answer the questions, but neither of
(a) Statement I alone is sufficient to answer the the statements alone is sufficient to answer the
question but statement II alone is not sufficient question.
to answer the question. (d) Either statement I or statement II by itself is
(b) Statement II alone is sufficient to answer the sufficient to answer the question.
question but statement I alone is not sufficient to (e) Statements I and II taken together are not
answer the question. sufficient to answer the question.
Direction (75-79): Bar graph given below shows total students (Boys+Girls) who play four different games and number
of boys who plays these games respectively. Study the data carefully and answer the following questions.

120
100
80
60
40
20
0
Cricket Volleyball Football Tennis

(Boys+Girls) Boys

75. Find the ratio between total number of students 76. Total number of boys who play Cricket and Football is
(Boys+Girls) who play Cricket, Volleyball and Tennis how much less than total students (Boys+Girls) who
to total number girls who play Volleyball and play Football and Tennis?
Football? (a) 112
(a) None of the given options (b) 116
(b) 21 : 8
(c) 118
(c) 23 : 16
(d) None of the given options
(d) 23 : 8
(e) 114
(e) 21 : 16

9 Adda247 Publications For any detail, mail us at


Publications@adda247.com
50+ Bank PO | Clerk Previous Year’s Papers 2016 – 2020

77. Total number of boys who play Volleyball and Tennis (d) None of the given options
is what percent of the total girls who play same game? (e) 80%
1
(a) 333 % 79. Find the average number of girls who play Volleyball,
(b) None of the given options Football and Tennis?
2
(c) 66 % (a) 54
3
(d) 50% (b) 52
(e) 25% (c) None of the given options
(d) 56
78. Out of total boys and girls play Football, 50% and (e) 50
1
333 % respectively qualified for nationals. Find total
80. A container contains 120 lit of pure milk. If X lit of
students (Boys+Girls) who were disqualified for pure milk is replaced with Y lit of water then ratio of
nationals is what percent of total students milk to water becomes 20 : 1 and if 2X lit of pure milk
(Boys+Girls) who play this game? were replaced by Y lit of water, then ratio of milk to
(a) 40% water becomes 16 : 1. Find the value of ‘X+Y’?
(b) 60% (a) 20 (b) 25 (c) 24
(c) 50% (d) 30 (e) 16

ENGLISH LANGUAGE

Direction (81-90): Read the following passage carefully PMJAY, this is to be achieved using defined treatment
and answer the questions given below it. Certain words are packages for which rates are prescribed. Costs are a
given in bold to help you locate them while answering contested area between the care-providers and the Centre,
some of the questions. and many for-profit hospitals see the government’s
proposals as unviable. In the absence of adequate
Ayushman Bharat has been rolled out as a health
preparation, the Ayushman Bharat administration is
protection scheme that will provide guaranteed access to
talking of a rate review. More importantly, a lot of time has
treatment that is free at the point of delivery to about 40%
been lost in the NDA government’s tenure, when State
of the population selected on the basis of censured socio-
governments should have been persuaded to regulate the
economic indicators. It is the essential first step on the road
hospital sector under the Clinical Establishments
to universal health coverage, although it has been launched
(Registration and Regulation) Act, which dates back to
by the NDA government quite late in its term, possibly with
2010. The law broadly provides for standardization of
an eye on the 2019 general election. Since the Centre has
facilities and reasonable rates for procedures.
announced that 10.74 crore families identified through
Apprehensions of fraud have prompted Ayushman Bharat
Socio-Economic Caste Census data will be given an annual
administrators to announce that some key treatments
Rs.5 lakh cover under the Pradhan Mantri Jan Arogya
should be availed through public sector institutions. But
Yojana (the insurance component of the scheme), the
public facilities have been neglected for long. Going
question of eligibility appears settled. But the late start
forward, there are some clear imperatives. It is essential to
makes it virtually impossible for all those who are
reduce the pressure on secondary and tertiary hospitals
technically insured to avail the services that state agencies
for expensive treatments by investing in preventive and
must make available, within a reasonable time-frame. And,
primary care facilities. Here, the 150,000 health and
the allocation of just Rs.2,000 crore during the current year
wellness centres of the National Health Protection Mission
to the PMJAY cannot provide the promised cover to the
can play a valuable role. The first-order priority should be
large population sought to be included. Not all States and
to draw up a road map for universal health coverage,
Union Territories are in a position to raise their own share,
through continuous upgradation of the public sector
and a few have not even joined the scheme. The challenge
infrastructure.
of funding, therefore, remains. And without adequate
budgetary commitments, the implications of pooling the As Prime Minister Narendra Modi launched his
financial risk for such a large segment of the population government's ambitious healthcare scheme Ayushman
through insurers or state-run trusts or societies make the Bharat on Sunday, five states remained unconvinced.
outcomes uncertain. Telangana, Odisha, Delhi, Kerala and Punjab have said they
will not implement the programme till their concerns are
Guaranteeing health-care access using private or public
addressed since they have better health assurance
facilities presumes tight cost control. In the case of the

10 Adda247 Publications For any detail, mail us at


Publications@adda247.com
50+ Bank PO | Clerk Previous Year’s Papers 2016 – 2020

schemes."Ayushman Bharat doesn't only provide financial 84. What is/are the factor(s) that has influenced some of
support, but in the next 4 years it is envisaged that nearly the states to not implement the Ayushman Bharat
1.5 lakh next door wellness centres will be opened, which programme?
will provide free test facilities and treatment of simple (a) That it is a world’s largest government’s funded
ailments. Ayushman Bharat, dubbed as the world's largest healthcare scheme but withstands the financial
government-funded healthcare scheme, is expected to risks.
benefit 50 crore citizens. Thirty-one states and union (b) Because it has been launched by the NDA
territories have agreed to implement the programme that government quite late and that too not for the
will entitle nearly 10 crore economically backward betterment of health sector.
families to health coverage of Rs. 5 lakh each. (c) As the approach of this scheme is very diffused
The government is pursuing a holistic approach towards and fragmented.
the betterment of the health sector. While it focuses on (d) Since these states have better health insurance
affordable healthcare on one hand, emphasis is also laid on schemes.
preventive healthcare. The number of Ayushman Bharat (e) All of these.
beneficiaries is almost equal to the population of Canada,
85. Which of the statement is definitely false according to
Mexico and the United States put together. NITI Aayog
the passage?
member Dr Vinod Paul said the scheme -- to be funded on
a 60:40 ratio by the centre and state -- will come into effect (a) The outcomes of Ayushman Bharat Scheme are
from September 25, the birth anniversary of Rashtriya uncertain and there are implications of pooling
Swayamsevak Sangh ideologue Pandit Deendayal the financial risk.
Upadhyay. Over 8,735 hospitals, both public and private, (b) Thirty-one states and union territories have
have been empanelled for the scheme, and as many as 31 agreed to implement the programme that will
states and union territories have signed MoUs with the entitle nearly 10 crore economy.
Centre to implement the programme, a PTI report said. (c) The pressure on secondary and tertiary
hospitals for expensive treatments must be
81. What according to the author is possibly the reason
reduced by investing in preventive and primary
for the launching of Ayushman Bharat by NDA
government? care facilities.
(a) Continuous upgradation of private sector (d) The main focus of government under Ayushman
infrastructure. Bharat Scheme is only on affordable healthcare.
(b) Treatment to be availed through public sector (e) None of these.
institutions.
Directions (6-8): Choose the word which is most nearly
(c) To reduce the pressure on secondary and
tertiary hospitals. the SAME in meaning as the word printed in bold as used
(d) The general elections of 2019. in the passage.
(e) All of these 86. Censured
(a) Approved (b) Acclaimed (c) Endorsed
82. Why the outcomes of Ayushman Bharat scheme are
portrayed as uncertain? (d) Condemned (e) Sanctioned
(a) As it requires standardization of facilities and 87. Avail
reasonable rates for procedures. (a) Bruise (b) Afflict (c) Utilize
(b) Due to a lack of budgetary support. (d) Grieve (e) Mutilate
(c) Due to a viability in government’s proposals.
(d) Since the government is pursuing a holistic 88. Apprehensions
approach towards the betterment of health (a) Anxiety (b) Axiom (c) Conviction
sector. (d) Certitude (e) Verity
(e) All of these.
Directions (89-90) : Choose the word which is the
83. In the passage, author has cited the law that imparts
OPPOSITE in meaning as the word printed in bold as used
for normalization of facilities and reasonable rates for
procedures. Which law is it? in the passage.
(a) Pradhan Mantri Jan Arogya Yojana 89. Envisaged
(b) Consumer Protection Act and Medical Profession (a) Factual (b) Anticipate (c) Envision
(c) Clinical Establishments (Registration and
(d) Foresee (e) Contemplate
Regulation) Act
(d) Drugs and Cosmetics Act, 1954 90. Empanelled
(e) The Pre Natal Diagnostic Techniques Act and (a) Enlisted (b) Enrolled (c) Constituted
Rules (d) Allocated (e) Ousted
11 Adda247 Publications For any detail, mail us at
Publications@adda247.com
50+ Bank PO | Clerk Previous Year’s Papers 2016 – 2020

Directions (91-95): In each of the following questions, a Prohibitory orders under Section 144 were issued and
sentence is given with four words given in bold. The word 3,000 police personnel _____(99)_____ to ensure that the
may be mis-spelt or grammatically or contextually Kisan Kranti Yatra, which began from Tikait Ghat in
incorrect. You have to choose the incorrect word as your Haridwar on September 23, was _____(100)_____ from
answer. If all the words are correct, mark option (e). i.e. All entering Delhi.
are Correct as your answer.
Violence erupted as a section of the farmers tried to
91. The government is right to supersede the broad of _____(101)_____ through the barricades using tractor-
Infrastructure Leasing & Financial Services (IL&FS), trolleys. As clashes broke out, the police used tear-gas and
given the large size and exposure of insurance, water cannon. Seven policemen and several protesters
mutual funds and provident funds to the entity. were injured.
(a) supersede (b) broad (c) exposure
Seeking to _____(102)_____ the farmers to return, the
(d) entity (e) All are correct
government announced that a committee of Chief
92. In 1994, Allison, who was then at the University of Ministers would investigate their demands. Home Minister
California, Berkeley, reported that CTLA-4, a protein Rajnath Singh held discussions with Union Agriculture
present on T-cells, derailed the immune system’s Minister Radha Mohan Singh and other Ministers, after
assault on cancer cells by acting as a broke. which Minister of State for Agriculture Gajendra Singh
(a) reported (b) derailed (c) assault Shekhawat met the protesters to assure them that their
(d) broke (e) All are correct demands would be investigated. BKU chief Naresh Tikait,
however, said the farmers were “not satisfied” with the
93. Chile’s experience was miring by faulty data-
government’s assurance.
gathering, rules that hampered the trade in carbon
emission permits/coupons, and lackadaisical 96. (a) eroded (b) equivocated (c) fired
monitoring and enforcement of emission limits by (d) deviated (e) chortled
government authorities.
97. (a) flay (b) disperse (c) zap
(a) miring (b) hampered (c) lackadaisical
(d) gallop (e) amble
(d) enforcement (e) All are correct
98. (a) belated (b) constructive (c) restless
94. We anticipate that the rate hike will be
(d) erroneous (e) erudite
accompanied by a change in the stance of monetary
policy from neutral to withdrawal of accomodation. 99. (a) clambered (b) deployed (c) clenched
(a) anticipate (b) accompanied (c) neutral (d) bungled (e) belabor
(d) accommodation (e) All are correct
100. (a) arrested (b) duped (c) collated
95. Financial inclusion has the potential to become one (d) doused (e) cogitated
of the most robust force multipliers that can propel 101. (a) castigate (b) debase (c) dwindle
the next growth revolution in India in the 21st
(d) decoy (e) break
century, increasing GDP by close to a percentage
point, provided policymakers pull the right levers. 102. (a) admonish (b) berate (c) abolish
(a) inclusion (b) robust (c) propel (d) sway (e) camouflage
(d) increasing (e) All are correct Directions (103 – 110): Read each sentence to find out
Directions (96 -102): In the following passage there are whether there is any grammatical error in it. The error, if
blanks, each of which has been numbered. These numbers any, will be in one part of the sentence. The number of that
are printed below the passage and against each, five part is the answer. If there is no error, the answer is (e).
(ignore the errors of punctuation, if any.)
words/phrases are suggested, one of which fits the blank
appropriately. Find out the appropriate word/phrase in 103. Often in political campaigns, a point is reached (a)/ at
each case. which the candidates (b)/ take out their gloves (c)/
and start slugging with bare fists. (d)/ no error. (e)
Police _____(96)_____ water cannon and tear-gas shells to
104. People are worried (a)/ more because of the
_____(97)_____ thousands of farmers, affiliated to the
frequency (b)/ of occurrence of the crime rather (c)/
Bhartiya Kisan Union (Tikait group), who tried to break
than the magnitude of the crime. /(d). No error. (e)
through barricades to enter the national capital on the
Delhi-U.P. border at Ghazipur on Tuesday morning, even as 105. The number of students (a)/ competing in the event
_____(98)_____ negotiations between BKU leaders and (b)/ has been fallen (c)/ because of want of incentive.
Central Ministers failed to end the stand-off. (d)/ No error. (e)

12 Adda247 Publications For any detail, mail us at


Publications@adda247.com
50+ Bank PO | Clerk Previous Year’s Papers 2016 – 2020

106. When he used to walk (a)/ along the road, (b)/ a wild (a) improve, proper
and ferocious dog (c)/ knocked him down. (d)/ No (b) strengthen, durable
error. (e) (c) raise, adequate
(d) stimulate, effective
107. Even very small reservoirs (a)/ impounding water (e) provide, delicate
seasonally (b)/ can cause deformation in (c)/ the
neighbouring region.(d)/ No error (e) 115. Governments in India seem to delight in arbitrary
censorship under the ________ of _________ the public.
108. Sometimes they may also show you (a)/ a fake flight (a) garb, upholding
ticket they booked just to meet you (b)/ but had to (b) pretext, protecting
cancel because(c)/ of an urgent work or health (c) context, strengthening
issue(d)/. No error (e) (d) command, authorising
(e) guise, winning
109. The idea of (a)/ introducing genes to correct (b)/
heritable and other disorders (c)/ are revolutionary. Directions (116-120): In the following questions
(d)/ No error (e) rearrange the sentences given to form a meaningful
paragraph and choose the appropriate sequence of the
110. The students requested their (a)/ Principal to be coherent paragraph.
enough kind to (b)/ grant them five thousand rupees
for (c)/ the reconstruction of the tennis court. (d)/ No 116. (A) This is primarily because of restrictive export
error (e) policies (minimum export prices, export bans or
export duties) and domestic marketing policies
Directions (111-115): In each of the following sentences (due to the Essential Commodities Act, APMC,
there are two blank spaces. Below each sentence there are etc).
five pairs of words denoted by numbers (a), (b), (c) ,(d) and (B) The Producer Support Estimate (PSE) for India
(e). Find out which pair of words can be filled up in the works out to be minus (-) 14 per cent of the gross
blanks in the sentence in the same sequence to make the farm receipts for the period 2000-01 to 2016-17.
sentence grammatically correct and meaningfully (C) But a recent study, conducted jointly by the
complete. OECD and ICRIER, estimated that India’s trade
and marketing policies have inflicted a huge
111. Leisure must be spent carefully and __________ only, negative price burden upon the country’s
otherwise the devil will take the ________ of you. farmers.
(a) positively, care (D) Most countries support agriculture to ensure
(b) constructively, better food security and/or enhance farmers’ income.
(c) pro-actively, though India is no exception.
(d) objectively, energy (E) The main policy instruments to support farmers
(e) purposefully, measure in India include subsidised fertilisers, power,
agri-credit and crop insurance on the input side,
112. Nothing is impossible in the world of politics. States and minimum support prices for major crops on
which were _________ foes and had their deadly the output front.
missiles pointed at each other find themselves ________ (a) DEACB (b) BADEC (c) CDEAB
in military alliances. (d) ABCDE (e) DECBA
(a) implacable, partners
117. (A) Mountbatten parried him by telling that he
(b) incessant, capsized
would first go to meet the Maharaja, ‘who was an
(c) deadly, approaching
old acquaintance’. The visit took place during
(d) known, soliciting June 18- 23.
(e) clandestine, surreptitious (B) Mountbatten advised Hari Singh not to make the
113. Inferring attitudes from expressed opinion has independence declaration.
many_______. People may______ their attitude and (C) Gandhi met Mountbatten early in June, wanting
express socially acceptable opinions. to visit Kashmir instead of Nehru or ‘prepare the
(a) limitations, conceal way for Pandit Nehru’.
(b) advantages, show (D) However, Mountbatten’s mission was a failure.
(E) He conveyed Patel’s message that ‘the States
(c) drawbacks, support
Department were prepared to give an assurance
(d) benefits, avoid
that, if Kashmir went to Pakistan, this would not
(e) reasons, acknowledge be regarded as unfriendly by Government of
114. We should move towards a system where the banks India’.
can_______ capital in the market with _______ safeguard (a) DEACB (b) BADEC (c) CABED
so that they continue to be public sector banks. (d) ABCDE (e) DECBA

13 Adda247 Publications For any detail, mail us at


Publications@adda247.com
50+ Bank PO | Clerk Previous Year’s Papers 2016 – 2020

118. (A) While road dust and pollution from heavy vehicles (C) In comparison, the out-of-pocket expenditure is
are primarily responsible for the noxious pall that sets much lower in China, at 34%, and in the US it is
on Delhi and other urban centres, the burning of 11%.
paddy stubble by farmers to clear their fields for the (D) With the announcement of the National Health
next crop is considered to be responsible for 20% of Protection Scheme (NHPS), India has embarked
the smog. on the path of universal health coverage.
(B) There is a 50% subsidy to farmers, and a 75% (E) India, currently, has a highly inadequate social
waiver to cooperative societies, agencies that security structure, and the situation is especially
rent out equipment, farmers’ interest groups or
dire in healthcare.
gram panchayats to buy such machines.
(a) CAEBD (b) BAEDC (c) ECDAB
(C) To address this, the Centre is partnering with
(d) DAEBC (e) DABEC
Punjab, Haryana and Uttar Pradesh to provide
farmers with a range of mechanised implements 120. (A) In fact, what’s worrying is that some of these
to clear their fields of paddy crop residue to expensive plants may end up being liquidated
prepare for sowing wheat. because lenders would resist selling them at big
(D) This year will be a crucial test for a scheme haircuts, fearful of being questioned by the
piloted by the Union government to address the investigative agencies.
winter haze. (B) What is more important, however, is that it won’t
(E) States have got nearly Rs.650 crore to help
really solve the issues faced by these plants and
farmers buy subsidised equipment such as
will, at most, help postpone the day of reckoning.
Happy Seeder, paddy straw choppers and Zero
Till Drill. (C) Which is why union power minister RK Singh is
(F) The onset of the winter season has come to be trying to convince RBI it should provide lenders
associated with toxic atmospheric pollution in some forbearance for these assets.
north India. (D) Whether the central bank will make exceptions
(a) DEFACB (b) FDACBE (c) CABEDF for one sector is not clear.
(d) ABCFDE (e) DFECBA (E) With over 40,000 MW of power plants already
unviable for want of fuel or power purchase
119. (A) The aim of the NHPS is to cover 10 crore families
agreements (PPAs), and likely to be sold for a
with medical insurance of Rs 5 lakh per
song if they turn into non-performing assets on
household per year.
(B) The country’s average out-of-pocket banks’ books and end up in the NCLT, it is
expenditure on healthcare is one of the highest understandable the government is seriously
globally, at 68%, and this means that of every Rs concerned.
100 spent by the public on healthcare, Rs 68 (a) CAEBD (b) EACDB (c) AEBCD
comes from their pocket. (d) DAEBC (e) BCEDA

Solutions

REASONING ABILITY
Directions (1-5): 6. (b); Only II is implicit as it is given in the statement
Year Person Fruit that both the companies signed the contract for
1950 F Fig
not revealing the details of their company with
1953 D Mango
1961 E Guava the rivals but company A may have done this and
1968 A Apple breached the contract, so the company B sent the
1972 G Litchi legal notice to company A.
1985 B Grapes
1996 C Pomegranate 7. (e); The Act is aimed at eradication of the disease and
1. (b); 2. (b); 3. (c); so it needs to be proclaimed and promoted. So,
4. (e); 5. (e); both the courses follow.

14 Adda247 Publications For any detail, mail us at


Publications@adda247.com
50+ Bank PO | Clerk Previous Year’s Papers 2016 – 2020

Directions (8-12): emerged. Since both the assumptions follow


from the given statement, so both I and II are
implicit.
25. (e); It is mentioned that farmers will be in trouble
without rain. This means that timely rain is
essential. Also, it shows that farmers are
dependent on rain. So, both I and II are implicit.
Direction (26-30):
In the given input output, one word and one number is
arranged in each step. In odd numbered steps such as I, III
and V the word is getting arranged from the left end while
8. (a); 9. (e); 10. (c); the number is getting arranged from the right end. While
in even numbered steps such as II and IV the numbers are
11. (b); 12. (e); arranged from the left end while the words are arranged
Directions (13-16): from the right end. For step I- The word which is first
according to the alphabetical order is arranged from the
left end while the highest number is arranged from the
right end and so on in the odd steps. For step II- The
number which is lowest is arranged from the left end while
the word which is last according to the alphabetical order
is arranged from the right end and so on in even steps. Also
while arranging each of the number is added by 1 and ‘er’
is removed from each word while arranging.
Input: warmer 76 94 scraper locker 11 greater 52 39
13. (b); 14. (e); 15. (a); adopter
16. (b); Step I: adopt warmer 76 scarper locker 11 greater 52 39 95
Step II: 12 adopt 76 scraper locker greater 52 39 95 warm
17. (b); Step III: great 12 adopt scraper locker 52 39 95 warm 77
Step IV: 40 great 12 adopt locker 52 95 warm 77 scrap
Step V: lock 40 great 12 adopt 95 warm 77 scrap 53

18. (e); From the first, second, seventh and eighth letter- 26. (b); 27. (d); 28. (b);
P,R,T,I 29. (c); 30. (b);
The meaningful word is – TRIP.
Direction (31-35):
Direction (19-23):
Persons City Flower
A Patna Cosmos
B Patna Iris
C Mumbai Tulip
D Surat Lily
E Surat Sunflower
F Surat Orchid
G Mumbai Tithonian
H Patna Rose
I Patna Lotus 31. (d); 32. (c); 33. (c);
19. (a); 20. (d); 21. (c); 34. (a); 35. (c);
22. (d); 23. (d); Direction (36-38):
Directions (24-25): 36. (e); From I,
24. (e); As from the given statement we can assume that
the notice on the gate of ‘M’ housing society will
be followed by all the sales persons. Also it can
be assumed that the security guards are not be
able to stop so the requirement of notice is

15 Adda247 Publications For any detail, mail us at


Publications@adda247.com
50+ Bank PO | Clerk Previous Year’s Papers 2016 – 2020

From II, May S


July Q
August P
October R
From II,
Case-1 Case-2
Months Persons Persons
From I and II, January T
March T
May S S
July P
August P
October

37. (c); From I, From I and II,


Months Persons
January U
March T
May S
July Q
August P
October R
From II, 39. (b); Only B is concluded from the given statement as
it is given clearly in the statement that the CMC
products are popular in city X but nothing is
mentioned about city Y or the quality and prices
of CMC products, so these can’t be concluded.
And also we cannot conclude anything about the
rival companies.
40. (a); Both I and II can be inferred from the given
statement as both the quality and price are the
factors which can affect the demand so I and II
38. (e); From I,
can be inferred. But III and IV cannot be inferred
Months Persons
as nothing is mentioned about the liking of
January T/
people of city X for foreign products or their
March T/
need of healthy products.

QUANTITATIVE APTITUDE

6 6
41. (c); Let the number of green balls be x. 43. (b); Required ratio = 2×3+2×2= 10 = 3 : 5
Then, number of yellow balls = (12 – x)
9×3.78×1,00,000
ATQ, 44. (e); Required no. of classes = = 63
xC 7 54000
1
= 20 ⇒ x = 7
20C
1 45. (a); Required difference = 9 × 6000 × 3 – 10 × 4000 ×
Number of yellow balls = 12 – 7 = 5 2 = Rs. 82000
5C 4C 1
Required probability = 20C1 × 19C1 = 19 46. (d); Let number of classes taken by B on Thursday
1 1

2 2 and Friday each be x.


42. (a); 3952 3 = 3 πr3 + πr2 h (h = height of cylinder) ATQ,
11858 2 22 22
⇒ = × ×7×7×7+ ×7×7×h (3 × 3 + 2x) × 6× 8000 = 5,28,000
3 3 7 7
⇒ h = 21 cm. ⇒x=1

16 Adda247 Publications For any detail, mail us at


Publications@adda247.com
50+ Bank PO | Clerk Previous Year’s Papers 2016 – 2020
(22×8000+20×4000) On 5th day no men will be remaining and all the
Required % = 36×6000
× 100
remaining work will be completed by womens
approximately =120%
Remaining work = 2280 units.
2280
47. (c); Honorarium paid to A = Rs. 6 × 5000 = Rs. 30,000 Required time = 16×5 = 28½ days.
Honorarium paid to B (in that week) = Rs. 9 ×
8000 = Rs. 72,000 49. (b); Let the MP be Rs. 100x
30000 70 90
Required % = × 100 = 41 ⅔% SP = 100x × 100 × 100 = 63x
72000
Discount = 100x – 63x = Rs. 925
48. (d); Let the efficiency of 1 man be M unit/day and ⇒ x = 25
that of 1 woman be W unit/day SP = 63 × 25 = Rs. 1575
ATQ, 1575
Cost price for shopkeeper = 7 × 8 = Rs. 1800
40 × 12 × M = 12 × 48 × W.
M 6
⇒W=5 50. (c); Ratio of profit share of
Let total work = 40 × 12 × 6 = 2880 units. A B C
10000 × 8 9000 × 8 12000 × 4
In 4 days, work done by men = (40×6 + 30×6 +
10 : 9 : 6
20×6 + 10×6) = 600 units.
Total profit in a year = 25 × 576 = Rs. 14,400
Solution (51-55)
10,00,000
In 2015, Average salary expense = = 12,500
80
In 2016, Average salary expense = 12,500 − 500 = 12,000
Total expense in 2017 = 60 lakh
100
⇒ Total expense in 2016 and 2015 each = × 60,00,000 = 50,00,000 = 50 lakh
120
Total salary expense in 2017 = 60 − 47.5 = 12.5 lakh
126
Total salary expense in 2016 = × 10,00,000
100
= 12,60,000
12,60,000
Number of employees in 2016 = = 105
12,000
Number of employees in 2017 = 105 − 5 = 100
12,50,000
In 2017, Average salary expense = 100 = 12,500

Year Average salary Number of Salary Expense Others expense Total expense
expense employees
2015 12,500 80 10 lakhs 40 lakhs 50lakhs
2016 12,000 105 12.6 lakhs 37.4 lakhs 50 lakhs
2017 12,500 100 12.5 lakhs 47.5 lakhs 60 lakhs

40 16 56. (d); I. x³ = – 512


51. (e); Required ratio = 12.5 = 5
⇒ x = –8
52. (a); Required % =
12,500−12,500
× 100 = 0% II. 2y² + 20y + 48 = 0
12,500 ⇒ y² + 10y + 24 = 0
53. (d); Number of employees in 2018 = 82 × 3 − 100 − ⇒ y = –4 or –6
∴y>x
80 = 246 − 100 − 80 = 66
57. (e); I. x² = 25
54. (c); Let total male employees in 2016 = y x = ±5
⇒ Female employees in 2016 = y + 45 II. (y – 2)² = 3 (y – 2)
ATQ, ⇒ y² + 4 – 4y = 3y – 6
y + y + 45 = 105 ⇒ y = 30 ⇒ y² – 7y + 10 = 0
Total male employees in 2016 = 30 ⇒ y = 2 or 5.
Female employees in 2016 = 75 ∴ no relation can be established between x and y
30 2 58. (b); I. 2x² + 3x + 1 = 0
Required ratio = 75 = 5
2x² + 2x + x + 1 = 0
55. (a); Required difference = 12.6 lakh − 12.5 lakh (x + 1) (2x + 1) = 0
–1
= 0.1 lakh = 10,000 x = –1 or 2

17 Adda247 Publications For any detail, mail us at


Publications@adda247.com
50+ Bank PO | Clerk Previous Year’s Papers 2016 – 2020
1
II. 2 + y + y = 0 64. (e); Central angle of total tourists applied from city E
= 34 × 3.6 = 122.4°
⇒ y² + 2y + 1 = 0 ⇒ y = –1
∴x≥y 65. (d); Tourists applied for Canada and England from
18 22
city A = 5500 × − 2500 × = 990 − 550
59. (c); I. (x – 5)² = 16 100 100
⇒ (x – 5) = –4 or (x – 5) = 4 = 440
⇒ x = 1 or x=9 Let, Tourists applied for England from city A = y
II. y² + 8y = 84 Tourists applied for Canada from city A
⇒ y² + 8y – 84 = 0 = y + 120
⇒ y² + 14y – 6y – 84 = 0 ATQ,
⇒ y = – 14 or 6 y + y + 120 = 440
∴ no relation can be established between x and y 2y = 320
y = 160
7 6
60. (c); I. 2– + 2 = 0 Tourists applied for Canada from city A
x x
⇒ 2x² – 7x + 6 = 0 = 160 + 120 = 280
⇒ 2x² – 4x – 3x + 6 = 0 66. (c); Number of tourists applied for Canada and
⇒ 2x (x – 2) – 3 (x – 2) = 2 16
England from city D = 5500 × 100 − 2500 ×
⇒ (2x – 3) (x – 2) = 0 18
⇒ x = 3/2 or 2 = 880 − 450 = 430
100
15 22 Number of tourists applied for Canada and
II. 2 – + 8 = 0 34 22
y y
England from city E = 5500 × − 2500 × =
⇒ 8y² – 22y + 15 = 0 100 100
⇒ 8y² – 12y – 10y + 15 = 0 1870 − 550 = 1320
⇒ 4y (2y – 3) – 5 (2y – 3) = 0 Required difference = 1320 − 430 = 890
⇒ y = 5/4 or 3/2 24% of 24% of 2500 24×24×2500
67. (a); Required ratio = =
∴x≥y 4
72% of 22% of 2500 72×22×2500

62.5 = 11
61. (b); Speed of boat A in upstream = 2.5 = 25 km/hr
Speed of boat A in downstream = 25 + 2 × 2.5 68. (a); Let amount invested in scheme A be 2p and in
= 30 km/hr. scheme B be Rs. 5p.
80 Total interest received from both the schemes
Speed of boat B in still water = 30 × 100 together
= 24 km/hr 2p×X×5
= 100 + 5p × 100
21
Required distance = (24 – 2.5) × 4 = 86 km
(Interest at 10% at CI in 2 years = 10 + 10 +
62. (d); Let the present age of A be x years. 10×10
100
= 21%)
and that of B be y years.
ATQ,
ATQ, 35 2p×X×5 5p×21
x+4 2 7p × = + ⇒ X = 14%
=
y+6 1
100 100 100

⇒ x + 4 = 2y + 12 69. (d); Remaining amount = 60,000 – (12000 + X)


⇒ x – 2y = 8 … (i) = (48,000 – X)
C’s present age = 2 (x + y) years. ATQ,
1 1
2(x+y)+4 23
= (48000– X) × (1– – ) = 29000
y+4 4 7 6
29
⇒ 8x + 8y + 16 = 23y + 92 ⇒ (48000– X) × 42 = 29000
⇒ 8x – 15y = 76 … (ii) ⇒ X = Rs. 6000
From (i) and (ii)
70. (d); From I
y = 12 years.
Sum of present age of all the members = 28 × 3 −
63. (b); Number of tourists applied for Canada and 3 × 2 = 78 yr
16 Sum of age of Father and Mother
England from city D = 5500 × 100 − 2500 ×
18 = (30+6) × 2=72 years
100
= 880 − 450 = 430 Age of daughter= 78-72=6 years
Number of tourists applied for Canada from city From II
57 Age of daughter can be easily found
D = 430 × 86 = 285
285 Either statement I or statement II by itself is
Required % = 880 × 100 ≈ 32% sufficient to answer the question.

18 Adda247 Publications For any detail, mail us at


Publications@adda247.com
50+ Bank PO | Clerk Previous Year’s Papers 2016 – 2020

71. (c); From (i) y = 5 m/sec and x = 7.5 m/sec


30
Let investment of Bhavya be Rs x Time in which A cross B = = 12 seconds
2.5
Then investment of Veer= Rs (9000-x)
Both the statements taken together are
From (ii)
necessary to answer the questions, but neither of
Ratio of profit shares
the statements alone is sufficient to answer the
Veer : Bhavya
question.
(9000-x)× 12 x× 12+(x+2000) × 12
96+72+108 276 276
(9000-x) : (2x+2000) 75. (d); Required ratio = (72−24)+(80−32) = 48+48 = 96
ATQ 23
9000−x 2550 =
= 8
2x+2000 3400
Here we can find value of x 76. (a); Total number of boys who play Cricket and
So, both the statements taken together are Football = 44 + 32 = 76
necessary to answer the questions, but neither of Total students who play Football and Tennis
the statements alone is sufficient to answer the = 80 + 108 = 188
question. Required difference = 188 − 76 = 112
72. (b); From (i) 77. (c); Total number of boys who play Volleyball and
Let time taken by person from A to B is x hr Tennis = 24 + 48 = 72
1
Then from B to A is (x+2) hr Total number of girls who play Volleyball and
Tennis = 72 − 24 + 108 − 48 = 48 + 60 = 108
From (ii) 72 2
Let speed while coming from B to A be y km/hr Required % = 108 × 100 = 66 3 %
20
And while going from A to B = (y+ 3 ) km/hr 78. (b); Total number of boys who play Football = 32
ATQ Total number of boys who qualified for nationals
100 50
20
(y+ 3 ) 5
= 32 × = 16
100
=
100
y
6 Total number of girls who play Football = 80 −
1 32 = 48
Here y=33 km
3 Total number of girls who play qualified for
We can find the required average speed of that 100
nationals = 48 × = 16
person by statement II alone. 300
Total students who were disqualified for
73. (d); P = Principal, Rate % = X%, T = time nationals = 32 − 16 + 48 − 16 = 16 + 32 = 48
From I → 48
PX2
Required % = × 100 = 60%
80
= 250
1002
250×100×100 79. (b); Total number of girls who play Volleyball
⇒ X2 = 25000
= 100 ⇒ X = 10% = 72 − 24 = 48
From II → Total number of girls who play Football
P×X×T
= P ⇒ X = 10% = 80 − 32 = 48
100
Either statement I or statement II by itself is Total number of girls who play Tennis
sufficient to answer the question. = 108 − 48 = 60
1 156
Required average = (48 + 48 + 60) = = 52
74. (c); Let speed of A and B be ‘x’ and ‘y’ respectively. As 3 3
A crosses B so, x > y 80. (b); ATQ
From I → (120−X)
= 1
20
y = 2(x − y) ⇒ x = 1.5y Y
From II → 120 − X = 20Y … (i)
In 2 seconds, B will travel 2y more And
(120−2X) 16
30+2y
Hence 16 = x−y ⇒ 16x − 30 = 18y Y
= 1
120 − 2X = 16Y … (ii)
From I and II together →
From (i) and (ii)
⇒ 16x − 30 = 18y
X= 20 & Y=5
Becomes
24y − 30 = 18y X+Y= 25

19 Adda247 Publications For any detail, mail us at


Publications@adda247.com
50+ Bank PO | Clerk Previous Year’s Papers 2016 – 2020

ENGLISH LANGUAGE

81. (d); The appropriate answer here is option (d). For Approved-officially agree to or accept as
the answer, refer to paragraph 1 where it is satisfactory.
mentioned Ayushman Bharat is the essential Acclaimed-publicly praised; celebrated.
first step on the road to universal health Endorsed- declared one's public approval or
coverage, although it has been launched by the support of.
NDA government quite late in its term, possibly Sanctioned- authorized.
with an eye on the 2019 general election. Here
87. (c); Avail means use or take advantage of (an
with the term ‘possibly’, we are referring to the
author’s view. opportunity or available resource).
Utilize means the same.
82. (b); Option (b) is the most suitable answer. Refer to Bruise- injury
first paragraph where it is given without Afflict-Trouble , bother
adequate budgetary commitments, the Grieve-Lament, Mourn
implications of pooling the financial risk for such Mutilate-Disfigure
a large segment of the population through
insurers or state-run trusts or societies make the 88. (a); Apprehension means anxiety or fear that
outcomes uncertain. Here the uncertainty of something bad or unpleasant will happen.
outcomes is about the implication of Ayushman Anxiety means a feeling of worry, nervousness,
Bharat Scheme. Hence option (b) is the answer. or unease about something with an uncertain
outcome. Axiom means a statement or
83. (c); Option (c) is the law which is the most suitable proposition which is regarded as being
choice. For the answer refer to second paragraph established, accepted, or self-evidently true.
where it is given that State governments should Conviction means a firmly held belief or opinion.
have been persuaded to regulate the hospital Certitude means absolute certainty or conviction
sector under the Clinical Establishments that something is the case.
(Registration and Regulation) Act, which dates Verity means a true principle or belief, especially
back to 2010. The law broadly provides for one of fundamental importance.
standardization of facilities and reasonable rates
for procedures. Rest of all the options are not 89. (a); Envisaged means contemplate or conceive of as
given in the passage. a possibility or a desirable future event.
Factual is related to facts. Hence it is the opposite
84. (d); Option (d) is the only pertinent choice here. The of the given word.
answer can be deduced from the second last Anticipate-regard as probable; expect or predict.
paragraph where it is mentioned that as Prime Envision-imagine as a future possibility;
Minister Narendra Modi launched his visualize.
government's ambitious healthcare scheme Foresee-be aware of beforehand; predict.
Ayushman Bharat on Sunday, five states
Contemplate- look thoughtfully for a long time
remained unconvinced. Telangana, Odisha,
at.
Delhi, Kerala and Punjab have said they will not
implement the programme till their concerns are 90. (e); Empanelled means enlisted or enrolled.
addressed since they have better health Ousted means expelled from a position or place.
assurance schemes. Hence it is the only opposite of the given word.
Enlisted means engaged.
85. (d); Option (d) is the only statement which is
Enrolled means recruit to perform a service.
definitely false among all the given statements.
Allocated means distributed for a particular
This statement can be corrected as follows:“ The
purpose.
main focus of government under Ayushman
Bharat Scheme is not only on affordable 91. (b); The word “broad” in the given sentence should
healthcare but also on preventive healthcare.” be replaced by “board” to form a contextually
86. (d); Censured means express severe disapproval of meaningful sentence. “Board” means a group of
(someone or something), especially in a formal people constituted as the decision-making body
statement. of an organization; while “broad” means having
Condemned means expressed complete a distance larger than usual from side to side;
disapproval of; censured wide. All the other highlighted words are

20 Adda247 Publications For any detail, mail us at


Publications@adda247.com
50+ Bank PO | Clerk Previous Year’s Papers 2016 – 2020

grammatically correct and contextually words are correct, the option (e) becomes the
meaningful. Hence, the option (b) is the most most feasible answer choice.
suitable answer choice. Inclusion means the action or state of including
Supersede means take the place of (a person or or of being included within a group or structure.
thing previously in authority or use); supplant. Robust means strong and healthy; vigorous.
Exposure means the state of having no Propel means drive or push something forwards.
protection from something harmful.
96. (c); Equivocate [verb] means ‘to lie, attempt to
92. (d); The word “broke” should be replaced by “brake” conceal the truth’;
as the phrase “acting as…” indicates that the next Deviate [verb] means ‘turn away from (a
word should be a noun. However “broke” is a principle, norm); depart; diverge;
verb which should be replaced by the noun Chortle [verb] means ‘chuckle with delight’;
“brake” which means an interruption of Water cannon and tear-gas shells are something
continuity or uniformity. All the other that are fired or shot to scatter a violent mob.
highlighted words are grammatically correct Among the given options, the option (c) ‘fired’ is
and contextually meaningful. Hence, the option the most appropriate choice and is the correct
(d) is the most suitable answer choice. answer.
Derailed means obstruct (a process) by diverting ‘Police fired water cannon and…’
it from its intended course.
97. (b); Flay [verb] means ‘strip the skin off (a corpse or
Assault means make a physical attack on.
carcass);
93. (a); The verb “miring” should be replaced as it is in Zap [verb] and gallop [verb] mean ‘to get rid of
its present participle form while the sentence is or kill something or someone especially
in the past tense. Therefore the correct form of intentionally;
“miring” required here is “mired”. All the other Amble [verb] means ‘walk or move at a slow
highlighted words are grammatically correct pace, relaxed pace’.
and contextually meaningful. Hence, the option Water cannon and tear-gas shells are something
(a) is the most suitable answer choice. that are fired or shot to scatter a violent mob.
Mired means involve someone or something in The congregation of the farmers was violent, and
(a difficult situation). this is reflected from the sentence ‘who tried to
Hampered means hinder or impede the break through barricades to enter…’ So, it can be
movement or progress of. Lackadaisical means safely assumed that the objective of the Police to
lacking enthusiasm and determination; and use the water cannon and tear-gas shells was to
carelessly lazy. scatter or disperse the violent congregation of
Enforcement means the act of compelling the farmers.
observance of or compliance with a law, rule, or Among the given options, the option (b) is the
obligation. most appropriate choice and is the correct
answer.
94. (d); The word “accommodation” is misspelt in the
‘…and tear-gas shells to disperse thousands of
given sentence as “accommodation”. All the
farmers…’
other words have been spelt correctly and are in
appropriate grammatical and contextual syntax. 98. (c); Belated [adj.] means ‘delayed’;
Hence, the option (d) is the most viable answer Erudite [adj.] means ‘learned; scholarly;
choice. The blank will be filled by an adjective which
Anticipate means regard as probable; expect or would qualify the noun ‘negotiations’ which
predict. were failed and were held between BKU leaders
Accompanied means be present or occur at the and Central Ministers. What sort of adjective
same time as (something else). should qualify ‘negotiations’ under the given
Neutral means having no strongly marked or circumstances.
positive characteristics or features. Among the given options, ‘restless’, a synonym of
Accommodation means the process of adapting hectic, is the most appropriate word to fill the
or adjusting to someone or something. blank and is the correct answer.
Hence, the option (c) is the correct answer.
95. (e); All the highlighted words of the given sentence
‘, even as hectic/restless negotiations between
are grammatically correct, contextually
BKU leaders and Central Ministers failed’
meaningful and correctly spelt. Since all the

21 Adda247 Publications For any detail, mail us at


Publications@adda247.com
50+ Bank PO | Clerk Previous Year’s Papers 2016 – 2020

99. (b); Clamber [verb] means ‘climb by crawling’; Among the given options, the option (e) ‘break’
Bungle [verb] means ‘mismanage; blunder’; is the most relevant and appropriate answer
Clench [verb] means ‘close tightly; grasp’; choice.
Belabor [verb] means ‘explain or go over ‘Violence erupted as a section of the farmers
excessively or to a ridiculous degree; attack tried to break through the barricades…’
verbally. 102. (d); Admonish means reprimand firmly.
The blank seems to be filled by a verb. The Abolish [verb] means ‘cancel; put an end to’;
preceding clause, which is in passive voice, tells Sway (verb) means ‘to persuade/convince’;
us that Prohibitory orders were issued… The Berate [verb] means ‘scold strongly’;
preceding clause relates to given clause, where Camouflage [verb] means ‘disguise; conceal;’
the blank appears, through a conjunction ‘and’, The blank seems to be a part of the ‘to+infinitive’
meaning that the clause where the blank appears and would be filled by a first-form of verb. The
will also be in passive-voice. What verb can be government was seeking something in relation
filled in the blank and can follow the noun ‘Police to the farmers and in that respect, announced
personnel’? Under the given circumstances, that a Committee of the Chief Ministers would
police personnel can be deployed to ensure that investigate their demand. Under the given
the violent congregation of farmers didn’t enter circumstances, what was the Government
Delhi. seeking? The government would like to convince
Among the given options, the option (b) is the the farmers.
most appropriate answer. Among the given options, ‘sway’ is the most
‘3,000 police personnel deployed to ensure…’ relevant word.
100. (a); Dupe [verb] means ‘to fool someone or someone Hence, the option (d) is the correct answer.
easily fooled’; ‘Seeking to convince the farmers to return, …’
Douse [verb] means ‘plunge into water; drench; 103. (b); Replace ‘at which’ by ‘where’. Preposition ‘to’ or
extinguish’; ‘at’ is not used after verb ‘reach’ but ‘adverb of
Collate [verb] means ‘examine in order to verify Place’ is used.
authenticity; arrange in order’; Example. I reached Delhi around 9.
Cogitate [verb] means ‘think over’; She has reached where she wanted.
Arrested [verb] means ‘keep (something) from
happening’. 104. (c); The use of ‘rather’ is superfluous as only ‘than’ is
used after comparative degree (more).
The tone of the clause ‘Prohibitory orders under
Example. He speaks more clearly than you.
Section 144 were issued…’ suggests that the
motive behind deploying the police was to 105. (c); ‘Has fallen’ will be used in place of ‘has been
prevent the Kisan Kranti Yatra from entering fallen’ as there is no passive form of intransitive
Delhi. verb (fall).
Among the given options, the option (a) is the (Note: Verbs are of two types namely transitive
most relevant and appropriate answer choice. verbs and intransitive verbs. Transitive verbs
‘which began from Tikait Ghat in Haridwar on need an object while Intransitive verbs cannot
September 23, was prevented/arrested from have an object. Ex. “I bought a car” in which
entering Delhi…’ ‘bought’ is transitive. “The Baby smiled”, ‘smiled’
is an intransitive verb)
101. (e); Castigate [verb] means ‘criticize severely;
punish’; 106. (a); Replace ‘When he used to walk’ with ‘while he
Debase [verb] means ‘reduce in quality or value; was walking’. For the two incidents of past, if one
lower in esteem; degrade’; continues and the other one has happened, then
Decoy [verb] means ‘lure or bait’; past continuous tense is used for the former and
Dwindle [verb] means ‘shrink; reduce’; simple past for latter.
What could be the reason for the eruption of Example. While I was walking along the road, a
violence under the given circumstances which mad dog attacked me.
would relate to the phrase ‘through the 107. (e); There is no error in the given statement.
barricades…’?
It seems that a section of the farmers would have 108. (d); Here ‘work’ is used as an uncountable noun. The
tried to break through the barricades. Moreover, use of ‘an’ before ‘urgent’ is incorrect. Use ‘some’
break through is a phrasal verb which means in place of ‘an’ or we can use it like this “a piece
make or force a way through (a barrier). of urgent work”

22 Adda247 Publications For any detail, mail us at


Publications@adda247.com
50+ Bank PO | Clerk Previous Year’s Papers 2016 – 2020

109. (d); The error is in part (d) of the sentence. Use ‘is’ in Acknowledge means accept or admit the
place of ‘are’ as the subject of the sentence is ‘the existence or truth of.
idea’ which is singular.
114. (c); ‘raise, adequate’ fits the sentence appropriately
110. (b); ‘Enough’ will be used after ‘kind’ as when enough where adequate means satisfactory or
is used as adverb of quantity then enough is acceptable in quality or quantity.
always used after the adjective or adverb. Durable means able to withstand wear, pressure,
or damage; hard-wearing.
111. (b); The option ‘constructively, better’ fits the
Stimulate means raise levels of physiological or
sentence most appropriately where
nervous activity in (the body or any biological
‘constructively’ means promoting further
system).
development or advancement.
Adequate means satisfactory or acceptable in
Pro-actively means by taking action to control a
quality or quantity.
situation rather than just responding to it after it
has happened. 115. (b); ‘pretext, protecting’ fits the sentence
Objectively means in a way that is not influenced appropriately as ‘pretext’ means a reason given
by personal feelings or opinions. in justification of a course of action that is not the
real reason.
112. (a); Implacable [adj.] means ‘unable to be appeased
Garb means clothing, especially of a distinctive
or placated’;
or special kind.
Incessant [adj.] means ‘(of something regarded
Guise means an external form, appearance, or
as unpleasant) continuing without pause or
manner of presentation, typically concealing the
interruption’;
true nature of something.
Capsized [adj.] means ‘(of a boat) overturned in
Pretext means a reason given in justification of a
the water’;
course of action that is not the real reason.
Soliciting [participle] (solicit, verb) means ‘ask
for or try to obtain (something) from someone’; 116. (e); The logical sequence of the sentences to form a
Words like ‘soliciting’ and ‘approaching’ cannot coherent paragraph is DECBA. Hence, the option
fill the second blank because the preposition (e) is the most suitable answer choice.
phrase ‘in military alliances’ which follow the
117. (c); The logical sequence of the sentences to form a
blank doesn’t gel up well with these words.
coherent paragraph is CABED. Hence, the option
So, the options (c) and (d) are ruled-out.
(c) is the most suitable answer choice.
The options (b) and (e) are completely irrelevant
and out-of-context. So, these options are also 118. (b); The logical sequence of the sentences to form a
ruled-out. coherent paragraph is FDACBE. Hence, the
Hence, the correct answer is the option (a). option (b) is the most suitable answer choice.
113. (a); The option ‘limitations, conceal’ fits the sentence 119. (d); The correct sequence of the paragraph after the
appropriately where ‘conceal’ means prevent rearrangement of sentences is DAEBC.
(something) from being known.
Conceal means not allow to be seen; hide. 120. (b); The correct sequence of the sentence after the
Drawbacks means a feature that renders rearranging them to form a coherent paragraph
something less acceptable; a disadvantage or is EACDB.
problem.

23 Adda247 Publications For any detail, mail us at


Publications@adda247.com
50+ Bank PO | Clerk Previous Year’s Papers 2016 – 2020

1 Adda247 Publications For any detail, mail us at


Publications@adda247.com
50+ Bank PO | Clerk Previous Year’s Papers 2016 – 2020

Mock IBPS RRB PO Mains 2017


37
REASONING ABILITY

Directions (1-5): Read the following information carefully (a) Television is still most popular among kids.
to find given questions. (b) People love to play games at mobile phone instead
8 persons namely P, Q, R, S, W, X, Y and Z are sitting around of involving in outdoor activities.
a square table facing towards center. 4 of the persons are (c) Mobile has become the addiction for the younger
sitting in 4 corners while 4 of the persons are sitting in the generation.
middle of the sides. They have different ages. Those who sit (d) The older generation is not comfortable in using
in the corners having the ages multiple of 3.Those who sit mobile phone for their entertainment.
at the middle of the sides having the ages multiple of 2.
(e) A proper treatment is required by a doctor to get
P sits 2nd to the right of the one who has the age of 44. P is
an immediate neighbour of the one who is 39 years rid of the addiction of the mobile phone.
old. Two persons sit between Q and having the age of 39. 7. Mosquito bites can be more than just annoying and
Q has not the age of 44. W sits 2nd to the left of Q. S is itchy. They can make you really sick. Protect yourself
immediate right of the person having age of 15. Three and your family from mosquitoes during travelling.
persons sit between Z and the one having age of 15. The Because dangerous diseases like Zika, dengue, and
difference between the ages of Z and the one who is 2nd to chikungunya are spread by mosquitoes.
the left of Z is 6. S is older than Z. Immediate neighbours of
Which of the following can be assumed from the given
S are younger than S. Person having age 22 sits 2nd to the
statement?
right of the person having age 10. Age of P is not 10. Y sits
immediate left of the person having age of 22. Age of Y is (a) Mosquitoes are more dangerous than any other
51. Age of R is greater than 20. Age of R is 24 years more animals in the world.
than X. Age of X is 3 years more than age of P. (b) There is no treatment of diseases like Zika, dengue,
and chikungunya that are spread by mosquitoes.
1. Who among the following is of 9 years old? (c) Forty percent of the world’s population lives in an
(a) P (b) S (c) W area at risk for dengue spread by mosquito and an
(d) X (e) No one
estimated 390 million people per year are infected
2. Who sits 2nd right of the one who sits immediate left of with the viruses.
the one whose age is 51 years old? (d) There are more chances to get infected from
(a) R (b) S (c) Y mosquitoes while travelling rather than staying at
(d) Q (e) W home.
3. How many persons sit between the one who is of 15 (e) All travelers are suffering from diseases like Zika,
years old and the one who is of 12 years old, when dengue, and chikungunya spread by mosquitoes.
counted anticlockwise from the one who is of 15 years
Directions (8-11): Read the following information and
old?
(a) One (b) Two answer the questions that follow:
(c) Three (d) More than three ‘A’ walks 10 km north from point Q to reach point H. He
(e) None of these takes a left turn and walks 9 km to reach point S. On the
other side, ‘B’ walks 5 km north from point Y to reach point
4. ho among the following sits diagonally opposite to Z? J. Point Y is 9 km either east or west from point Q. Next ‘B’
(a) X (b) T (c) S turns to his right and walks 4km to reach point D. Also ‘A’
(d) P (e) W
turned left from point S and reached point M after walking
5. Who among the following sits immediate right of S? 5 km. M is in west direction from J.
(a) P (b) W (c) Y
(d) R (e) Z 8. What is the shortest distance between points M and J?
(a) 10 km (b) 26 km (c) 18 km
6. Television is no longer the medium of entertainment at (d) 12 km (e) Cannot be Determined
present. Mobile has taken the place of it among the
youngsters as they used to spend most of their time on 9. If ‘B’ walks 4 km east from point Y, then he is in which
it. direction with respect to point D?
Which of the following can be hypothesized from the (a) South (b) North-west (c) North-east
given statement? (d) North (e) South-west

2 Adda247 Publications For any detail, mail us at


Publications@adda247.com
50+ Bank PO | Clerk Previous Year’s Papers 2016 – 2020

10. What is the shortest distance between points Q and D? (b) If statement II alone is sufficient to answer the
(a) √194 km (b) √198 km (c) √197 km question, but statement I alone is not sufficient to
(d) √196 km (e) √195 km answer the question.
(c) If statement either I or II is sufficient to answer the
11. What is the direction of J with respect to Q?
question.
(a) South (b) North-west (c) North-east
(d) If both the statements I and II taken together are not
(d) North (e) South-west
sufficient to answer the question.
Directions (12-16): Read the following information (e) If both the statements I and II taken together are
carefully and answer the questions that follow: sufficient to answer the question.
Eight friends A, B, C, D, E, F, G and H born on 7th, 9th, 16th
17. What is the code for 'festival' in a code language?
and 19th in March and July. Each one of them likes either a
I. In that language `lo ko ni sa' means 'Celebrate
colour or a fruit. The persons who likes colours were born
festival light bright' and 'jo to ni fa' means 'festival
on that day which is a perfect square and the colour they
surprise candle shine '.
likes are:- Yellow, Green, Red and Blue. The persons who
II. In the same language 'bi ya la fa' means 'surprise
were born on the day which denotes a prime number like
high celebration delight' and ‘ya la fa ni’ means
fruits – Apple, Cherry, Mango and Banana.
‘Surprise festival high delight’.
The one who likes yellow colour was born on a day which
is a perfect square in the month of march. No person was 18. Six persons A, B, C, D, E and F are sitting around a
born between D and the one who likes yellow. D doesn’t circular table? Are they all facing the center?
like colours. Three persons were born between D and the I. A sits second to the left of F. D sits second to the
one who likes Mango. There is no person born between the right of F. Both E and C are immediate neighbour
one who likes Mango and Blue colour. The number of of A. F is facing inside.
people born after the one who likes blue colour is one less II. B is second to the left of E. Only D is between B and
than the number of persons born before A. The one who E. C is to immediate left of F. B sits opposite to A. C
likes Apple was born immediately before B. B do not like is third to the left of D. E is second to the right of B
Mango. The number of people born before B is same as the and second to the left of C.
number of persons born after G. Three persons were born
19. Six persons A, B, C, D, E, F have different heights. Who
between the one who likes Green and H. H and the one who
among the following is second tallest?
likes Cherry was born in the same month but not in March.
(I) F is taller than only one person. C is taller than A
C was born after the one who likes Blue colour. E doesn’t
but shorter than E.
like any colour. F doesn’t like Apple.
(II) A is taller than both F and B. E is not the tallest.
12. Who among the following likes green colour?
20. Who sits second to the left of Sparrow?
(a) B (b) A (c) G
(I) Five birds i.e. myna, parrot, bulbul, crow and
(d) H (e) C
sparrow are sitting in a row in north direction.
13. Who was born on 16th March? Myna sits at one of the extreme end. Parrot sits in
(a) D (b) A (c) G between Myna and Bulbul.
(d) F (e) E (II) Only one person sits in between Bulbul and
Sparrow. Sparrow does not sit to the left of Parrot.
14. Who among the following likes cherry?
(a) F (b) G (c) H 21. Five persons P, Q, R, S, T live on five different floors
(d) A (e) B such as bottom floor is numbered as 1 and top floor is
numbered as 5. Who among the following lives on
15. Who was born on 19th July?
fourth floor?
(a) C (b) A (c) D
(I) Three person lives between Q and T. R lives on
(d) B (e) E
third floor.
16. Who among the following likes red colour? (II) Only one person lives between P and S. T does not
(a) A (b) B (c) H live above R.
(d) C (e) D
Directions (22-26): Study the following information
Directions (17-21): Each of the questions given below carefully and answer the given questions.
consists of a question and two statements numbered I and Ten persons Q, R, S, T, U, V, W, X, Y and Z are going to four
II. You have to decide whether the data provided in the cities on four different dates i.e. 2nd, 4th, 7th and 9th of the
statements are sufficient to answer the question. month. Each person goes on different cities on different
(a) If statement I alone is sufficient to answer the question, dates. Four cities are namely Pune, Chandigarh, Kochi and
but statement II alone is not sufficient to answer the Patna, but not necessarily in the same order. At least one
question. person goes on each date.

3 Adda247 Publications For any detail, mail us at


Publications@adda247.com
50+ Bank PO | Clerk Previous Year’s Papers 2016 – 2020

No person goes to Pune on 4th of the month. R does not go (d) Traffic rules are stricter in day-time to avoid traffic
to any city on 2nd and 7th of the month. The persons, who congestion problem.
go on 7th, go to every city. Y goes to Kochi only with that (e) Traffic violation is not a punishable offence in
person who goes on 7th of the month. Maximum four India.
persons can go on the same date. Only one person goes to
Chandigarh. Z does not go on 7th of the month. S does not Directions (28-32): Study the following information
go to Pune but go with U, who goes on 9th of the month. One carefully to answer the given questions:
of the person, who goes to Kochi, goes on even number date A word and number arrangement machine when given an
of the month, which is more than 3. R goes to Pune with Q, input line of words and numbers rearranges them
but does not go with X. X goes on that city in which most of following a particular rule. The following is an illustration
the person are going. V does not go with S and Z but go with of input and rearrangement.
another person. Z goes to Pune. T does not go with U. W Input: 97 nosy 21 snow cold 32 asian 46 65 viral 83 high
does not go on 9th and 2nd of the month. X goes on an odd Step I: 211 97 nosy snow cold asian 46 65 viral 83 high 322
number date of the month and no any person go on that Step II: asian 211 97 nosy snow 46 65 viral 83 high 322 cold
date on which X goes in that city. One of the person, who Step III: 463 asian 211 97 nosy snow viral 83 high 322 cold
goes to Pune go on even number date of the month. S goes 654
on an odd number date of the month. Step IV: high 463 asian 211 97 snow viral 83 322 cold 654
22. Who among the following person goes to Chandigarh? nosy
(a) R (b) V (c) X Step V: 835 high 463 asian 211 snow viral 322 cold 654
(d) T (e) Q nosy 976
23. In which of the following city most of the persons are Step VI: snow 835 high 463 asian 211 322 cold 654 nosy
going? 976 viral
(a) Chandigarh (b) Pune (c) Kochi Step VI is the last step of the rearrangement. As per the
(d) Both Kochi and Pune (e) Patna rules followed in the above steps, find out in each of the
following questions the appropriate steps for the given
24 Four of the following five are alike in a certain way
input.
based on their positions in the above arrangement and
Input: peak 18 utility 76 emerge 27 beautiful 37 51 visible
so form a group. Who among the one that does not
belong to that group? 86 know
(a) S (b) R (c) Y 28. How many steps would be needed to complete the
(d) X (e) T arrangement?
25. Which of the following statement is true regarding U? (a) X (b) VIII (c) V
(a) Only U goes to Chandigarh. (d) VI (e) None of these
(b) U goes on 7th of the month
29. What will the addition of the numbers which is fifth
(c) None of the option is true
from the left end in step II and 5th from the right end in
(d) U goes to that city in which most of the person is
step IV?
going.
(e) U goes with Z (a) 312 (b) 210 (c) 162
(d) 165 (e) None of these
26. Who among the following person goes on 2nd of the
month? 30. Which of the following would be the difference of the
(a) Z (b) Q (c) W numbers which is 2nd from left end in step IV and 2nd
(d) Y (e) S from right end in Step II?
(a) 290 (b) 83 (c) 193
27. Releasing the 'Road Accidents in India - 2016' report,
(d) 101 (e) None of these
the Road Transport and Highways Minister said more
than 400 people lost their lives daily on roads in road 31. Which of the following element will be 6th from the left
accidents at night. of 3rd from the right end in step V?
Which of the following could be the appropriate cause (a) 181 (b) beautiful (c) 373
of the death in road accidents at night?
(a) Increase in population lead to more number of (d) know (e) None of these
accidents. 32. In Step IV, which of the following word/number would
(b) Many people prefer to travel in night rather than in
be on 4th position (from the left end)?
day.
(c) Lack of proper lightning arrangement and (a) visible (b) 181 (c) 97
undignified installation of traffic signals at many (d) utility (e) None of these
places.
4 Adda247 Publications For any detail, mail us at
Publications@adda247.com
50+ Bank PO | Clerk Previous Year’s Papers 2016 – 2020

33. Education has been a problem in our country and lack (a) E (b) H (c) D
of it has been blamed for all sorts of evil for hundreds (d) A (e) None of these
of years. Half the country does not even today have
38. How many cars are parked in between car B and car H?
access to proper education, and only a small fraction
(a) Four (b) Three (c) Five
can go to university.
(d) Two (e) None of these
Which of the following substantiates the given
statement? 39. Bihar Chief Minister Nitish Kumar on Monday
(a) Our education system today encourages excellence launched a State-wide campaign as one of the major
– in students, in teachers, throughout the system. step for women empowerment to abolish dowry,
(b) Our new education system creates entrepreneurs, stated it as major drawback of our society.
innovators, artists, scientists, thinkers and writers Is this decision sufficient to get rid from this custom,
who can establish the foundation of a knowledge which is running from long time in Bihar?
based economy. Which of the following will be best suited option to get
(c) India needs to embrace internet and technology if rid from dowry with their explanation?
it has to teach all of its huge population, the (a) Yes, as State govt. Is launching state wide
majority of which is located in remote villages. campaign, it will impact the whole society of Bihar.
(d) Our education system is geared towards teaching (b) No, It will not impact too much untill thinking of
and testing knowledge at every level as opposed to people will changed and dowry will be declared as
teaching skills. illegal and punishable offence in Bihar.
(e) We may have the most number of engineering (c) Yes, as many NGO run by women will also come
graduates in the world that has been translated against dowry.
into much technological innovation here. (d) No, It has been set in nerves of Bihar’s society as
Directions (34-38): Study the following information one of the necessities of the marriage.
carefully and answer the given questions: (e) Yes, As other states will also follow Mr. Kumar step
Seven cars A, B, C, D, E, I, H are parked in a linear row facing to abolish dowry from society.
north in such a way that no two cars parked with each
40. Statement-Whole world looking at India, says Modi.
other of according to alphabetical order (for ex- A is not
India’s growth presents a win-win partnership for both
parked with B, B is not parked with A and C and so on).
India and the U.S., Prime Minister Narendra Modi told
Some cars either of Petrol or some are Diesel variant. I is
while adressing the media gathering ahead of his first
third to the left of A. More than three cars are parked
meeting with President Donald Trump.
between the petrol cars. E is second to the right of B. H is a
(I) The talks between the Trump and Modi would be
diesel car and parked at one of the extreme end. C is a diesel
broad-ranging and seek to advance common
car and parked fourth to the right of D. All the cars are
priorities.
arranged in ascending order according to the distance
(II) Prime Minister Narendra Modi would only attend
covered by them from left to right. Car B covers 27km and
Car C cover 40km. H is to the right of E. I is of Diesel variant the UN ‘Vesak Day’ celebrations and no bilateral
car and no petrol variant car parked next to it. agreements will be signed during his visit to Sri
Lanka.
34. Which among the following are petrol cars? (III)US President Donald Trump and Prime
(a) D and E (b) A, I, B (c) A and D Minister Narendra Modi forging a strong personal
(d) A, I, D, B (e) None of these bond, and advancing a solid bilateral relationship.
35. What can be the distance covered by car I? (IV)The visit provides an opportunity to strengthen
(a) 45km (b) 63km (c) 17km the US-India strategic partnership, which the
(d) 31km (e) 25km President views as being critical in Asia-Pacific and
globally.
36. Which among the following cars are parked at extreme Which of the following is not in line with the given
ends? statement?
(a) D, E (b) H, B (c) H, D
(a) Only II (b)Only III and IV
(d) D, B (e) None of these
(c)Only II and III (d)Only IV
37. Which among the following car is parked immediate
left of car C? (e)Only I and III

5 Adda247 Publications For any detail, mail us at


Publications@adda247.com
50+ Bank PO | Clerk Previous Year’s Papers 2016 – 2020

QUANTITATIVE APTITUDE

Directions (41-46): Study the following table carefully to 46. The number of offline and online contestant together
answer the questions that follow. who completed the survey from village C are
The table shows the online and offline contestants taking approximately what percent of total participants on
part in a survey from four villages and total contestant who survey from village D?
(a) 52% (b) 62% (c) 48%
have not completed the survey (online and offline)
(d) 56% (e) 59%
Note-
1. Total contestants in a village= Online contestants + 47. A and B started business with Rs 600 and Rs 500
Offline contestants respectively. After 4 months, C replaces B with X% of
2. Total contestants in a village=Contestants who B’s capital. After 1 year C’s share of profit out of the
total profit 24000 is 5600. Find the value of X.
complete the survey + contestants who do not complete
(a) 60 (b) 70 (c) 75
survey (d) 66 (e) 65
48. A train is 216 m long and it crosses a platform in 19
seconds with speed 21 m/s. If some 21 m long boxes
are added in train and it crosses same platform, then it
takes 26 seconds to cross the platform with same
speed. How many boxes were added to the train?
41. In the village A, the number of online and offline (a) 7 (b) 10 (c) 12
contestants who didn’t complete the survey are equal, (d) 5 (e) 8
then online contestants from village A who completed
49. A can complete a work in 36 days. B is 33.33% more
the survey are what percent (approximate) more than
efficient than A. In how many days both complete the
offline contestants who completed the survey from the work if they work on alternate days starting with A?
same village? (a) 26 days (b) 30 days (c) 28 days
(a) 27% (b) 22% (c) 35% (d) 31 days (e) 36 days
(d) 31% (e) 37%
50. Rakesh added 12% of his salary in PPF,3/8th of the
42. Total number of contestants from village C who remaining amount is spent on clothes and the
completed the survey are how much more or less than difference between PPF and clothes expenses is Rs
total number of contestants who completed the survey 10500. Remaining amount is spent on house rent and
from village B? other expenses. If house rent expenses is Rs 1500 less
than other expenses, then what is the house rent
(a) 841 (b) 857 (c) 837
expenses?
(d) 851 (e) 860 (a) Rs. 12000 (b) Rs. 10000 (c) Rs. 13000
43. If the ratio between online to offline contestants who (d) Rs. 11000 (e) Rs. 15000
didn’t completed the survey from village ‘D’ is 8 : 11 Directions (51-55): In each of the following series, one
and 65% of contestants who completed the survey number does not follow a specific pattern. Find that
online are male and 60% of contestants who complete number.
the survey offline are female, then find the difference 51. 200, 196, 192, 180, 160, 130, 88
between females of contestants who completed the (a) 180 (b) 196 (c) 200
survey online and male contestants who completed the (d) 88 (e) 160
survey offline from village D ?
52. 9.2, 10.6, 7.6, 12.4, 6, 14, 4.4
(a) 102 (b) 88 (c) 104 (a) 10.6 (b) 14 (c) 4.4
(d) 108 (e) 106 (d) 7.6 (e) 12.4
44. Find the difference between the number of offline 53. 1, 730, 975, 1054, 1081, 1090, 1093
contestants of village C and that of village A. (a) 730 (b) 975 (c) 1090
(a) 45 (b) 40 (c) 38 (d) 1093 (e) 1054
(d) 35 (e) 30 54. 3, 4, 9, 28, 113, 565, 3397
45. Find sum the of difference between total number of (a) 9 (b) 3397 (c) 565
(d) 4 (e) 28
online and offline contestants who participated in the
survey from all four village. 55. 4, 6, 12, 30, 60, 315, 1260
(a) 950 (b) 980 (c) 960 (a) 12 (b) 1260 (c) 60
(d) 735 (e) 840 (d) 30 (e) 315

6 Adda247 Publications For any detail, mail us at


Publications@adda247.com
50+ Bank PO | Clerk Previous Year’s Papers 2016 – 2020

Directions (56-60): Study the following pie charts and answer the questions that follow:
Given below are two pie charts which shows the percentage distribution of cricket players and football players in 6 sports
club out of the total cricket and football players respectively in these six sports club.
Note:
1. If it is said that number of players playing “either” football or cricket then it means sum of players playing football and
cricket.
2. no players plays both games
Cricket = 2400 Football = 3200
F A F
11% 18% A
E 5%
25%
E 21%
24%
B
20%
B
D 16%
D 20%
C C
12%
15% 13%

56. What is the difference between the total number of 61. In bag A there are 5 red balls, X green balls and 7 yellow
players playing either Football or Cricket from club D balls. Probability of drawing one green ball from bag A
and those playing Football from club C and E together? is 2/5. In bag B there are (X-3) red balls, (X-4) green
(a) 152 (b) 160 (c) 165 balls and 6 yellow balls. 2 balls are drawn from bag B.
(d) 172 (e) 150 Find the probability that both the balls are red colour?
57. The ratio between male to female players who play (a) 2/23 (b) 1/21 (c) 4/21
either Cricket or Football from club E is 9 : 7, then Male 5
(d) 2/21 (e) 21
players from club E playing either Cricket or Football
are what percent of players playing Football from club 62. Sum ofresent ages of A and B is 41, Age of A two year
F? hence is equal to age of C one year ago and Age of A four
(a) 408.75% (b) 437.75% (c) 438.75% year hence is equal to age of B one year ago. If ratio of
(d) 416.75% (e) 428.75% present age of A and D is 3 : 4. Find the difference
5 th between age of C and D.
58. of players playing Cricket from club E are male and
12
7 th (a) 3 years (b) 5 years (c) 6 years
of players playing football from club C are male.
13 (d) 4 years (e) 8 years
Find the sum of total number of female players from
club E playing Cricket and total females from club C 63. Radius of a cylinder is equal to the side of an equilateral
playing Football . triangle having area 16 3 cm² and height of the
(a) 528 (b) 532 (c) 548 cylinder is equal to the perimeter of the triangle. Then
(d) 520 (e) 522
find the volume of cylinder.
59. Players who playing Football from club D and B (a) 1536 π sq. cm (b) 1518 π sq. cm
together are what percent (approximate) more or less (c) 1620 π sq. cm (d) 1460 π sq. cm
than players playing Cricket from club D and F (e) 1548 π sq. cm
together?
(a) 112% (b) 115% (c) 105% 64. A man invested Rs. 8000 in a scheme giving 20% p.a.
(d) 109% (e) 95% compound interest for two year if the interest received
from this scheme is 400% more than the interest on
60. Find the difference between the number of players
playing Cricket from club A and C together and the some other amount from another scheme giving 8%
number of players playing Football from club B, D and S.I. for 4 year. Find the total amount invested in both
F together. schemes.
(a) 615 (b) 520 (c) 525 (a) Rs. 11200 (b) Rs. 10200 (c) Rs. 12200
(d) 620 (e) 515 (d) Rs. 10400 (e) Rs. 10600

7 Adda247 Publications For any detail, mail us at


Publications@adda247.com
50+ Bank PO | Clerk Previous Year’s Papers 2016 – 2020

65. In one litre of mixture of alcohol and water, 30% is I. The area of the base of hall is 24 square metres.
water. The amount of alcohol that must be added to the II. The breadth, length and the height of the hall are
mixture, so that the part of water in the mixture in the ratio of 4 : 6 : 5.
becomes 15%, is: III. Area of one wall is 30 square metres.
(a) 1000 ml (b) 700 ml (c) 300 ml (a) Only I (b) Only II (c) Only III
(d) 900 ml (e) 1200ml (d) Either I or III (e) Data inadequate
66. The surface area of a spherical part of a hemispherical 69. 8 men and 6 women can complete a piece of work in 21
bowl with a flat circular detachable cover, excluding days. How many days will it take for 12 men and 9
the cover, is 616 sq cm. The area of the cover is 38.5 sq women to complete the same work?
cm. What is the volume of the bowl? A. 6 men can complete the work in 42 days.
(a) 1339 cm3 (b) 1430 cm3 B. 7 women can complete the work in 63 days.
C. The amount of work done by a woman is four-
(c) 1570 cm3 (d) 2096.69 cm3
sevenths of the work done by a man in one day.
(e) 2032. 69 cm3 (a) Any two of them
Directions (67-70): The following questions are (b) Any of them
accompanied by three statements (A) or (I), (B) or (II), and (c) Only C
(C) or (III). You have to determine which statement(s) (d) Either A or B only
is/are sufficient/necessary to answer the questions. (e) No need of any information
67. The ratio of the ages of Javed and Akhtar is 6 :11. Find 70. A train crosses another train in 10 sec. Find out the
out the ratio of their ages 5 years ago. lengths of the trains.
A. The difference of their ages is 25 years. A. Ratio between the lengths the of second and first
B. The difference of their ages after 5 years will be 25 train is 4 : 5.
years. B. Ratio between the speed of first and second trains
C. The sum of their ages is 85 years. is 1 : 2.
(a) Only A and C together are sufficient C. The speed of first train is 36 km/hr.
(b) Anyone of A, B and C is sufficient (a) Only A and B together
(c) Only A and B together are sufficient (b) Only B and C together
(d) Any two of A, B and C are sufficient (c) Only A and C together
(e) All together are necessary (d) Questions can’t be answered even after using all
68. What is the cost of painting the two adjacent walls of a the information
rectangular hall which has no windows or doors? (e) None of these

Directions (71-75): Given below the bar graph shows the percentage of valid voters in 5 villages in two years 2001 and
2005. Study the bar graph carefully to answer the based questions.
NOTE- Total voters in any year = Valid voters + Invalid voters

2001 2005
90
80
70
% of Valid voters

60
50
40
30
20
10
0
A B C D E
Villages

8 Adda247 Publications For any detail, mail us at


Publications@adda247.com
50+ Bank PO | Clerk Previous Year’s Papers 2016 – 2020

71. What is the difference between invalid voter from September and October. The ratio of chair, tables and
village C in the both given years, if valid voters in 2005 wardrobes sold by A in August is 42 : 36 : 23 while ratio of
11 chairs sold by A in August, September and October is 14 :
from village C are 4000 which is 31 19 % more than
23 : 27. Wardrobes sold by A in August is 230 less than
valid voters of same village in year 2001.
chairs sold in September by A. In September 665 chairs,
(a) 1190 (b) 1250 (c) 1290
400 tables and 210 wardrobes were sold by two
(d) 1350 (e) 1365
companies together. B sold same number of chairs in Aug
72. If in the year 2005 from village A, 2500 voters were and September. Number of tables sold by company B in
declared invalid voters 10% of valid voters opted September was equal to number of chairs sold by A in
NOTA and the winner got 200 more votes than loosing August while number of wardrobes sold by A in August and
candidate, then find the total vote hat loosing B in September were equal. Company B sold total 1025
candidate got in 2005 from village A. chairs in these three months together which was 480 more
(a) 4400 (b) 4600 (c) 5400 than total number of tables sold by A. Ratio of tables sold
(d) 5200 (e) 4800 by A and B in August is 12 : 11 and in October is 35 : 38
1 respectively. Total number of items sold in August was
73. In village B, if the total voters in 2001 were 13 23 % 1075. Total number of wardrobes sold by A in October was
more than total voters in 2005, then find the ratio of 35 less than wardrobe sold by B in October , while sum of
invalid voters in 2001 to the invalid voters in 2005 in wardrobe sold by A and B in October is 205.
same village.
(a) 131 : 160 (b) 130 : 161 (c) 127 : 141 76. Total number of chairs sold by B in September and
(d) 18 : 35 (e) 161: 130 October is
(a) 750 (b) 725 (c) 705
74. If there are 1600 males in valid voters from village E in (d) 715 (e) None of these
2001 and the females in valid voters of same village
and same year contributed is 36% of total valid voters. 77. By what percent tables sold by A in October are more
Find the percentage of invalid male voters in total than wardrobes sold by B in October ?
5 5 5
voters, if total males voters from village E in 2001 were (a) 456% (b) 406 % (c) 38 6 %
2000. 5
(a) 16.2% (b) 12.4% (c) 14.6% (d) 36 6 % (e) None of these
(d) 12.8% (e) 14.8%
78. Find the difference in number of chairs sold by A and B
75. If the ratio between valid voters from village B in 2001 in August.
and invalid voters from village D in 2005 was 16 : 3, (a) 101 (b) 120 (c) 105
then total voters from village D in 2005 were what (d) 110 (e) 112
percent more or less than total voters from village B in
2001 ? 79. What is the ratio of number of tables sold by A in
2
(a) 45 7 %
4
(b) 53 5 %
1
(c) 53 8 % August to that of B in September?
1 2 (a) 7 : 6 (b) 6 : 7 (c) 12 : 13
(d) 52 8 % (e) 50 5 %
(d) 11 : 12 (e) 6 :11
Directions (76-80): Study the following data to answer the
80. Find the number of wardrobes sold by B in October.
questions that follow.
There are two companies namely A and B, which sell (a) 80 (b) 120 (c) 115
chairs, tables and wardrobes in 3 months August, (d) 95 (e) 125

ENGLISH LANGUAGE

Directions (81-85): Read the following passage carefully ITeS fell from 20.8 per cent in 2012-13 to 14.9 per cent in
and answer the questions given below it. Certain words are 2014-15 and to a low of 7.3 per cent in 2015-16. Global
given in bold to help you locate them while answering circumstances combined with the specific nature of India’s
some of the questions. IT prowess seem to be responsible for this fall. India’s IT
There is a sense of gloom pervading India’s $150-billion industry recorded remarkable growth over a long period
information technology (IT) industry, which earned India because it exploited an outsourcing opportunity by
$88 billion in foreign exchange in 2015-16 through the perfecting a global delivery model for software and ITeS
exports of software and IT-enabled services (ITeS). based on cheap skilled labour. That opportunity was
According to data from the Reserve Bank of India (RBI), the offered by the cost-cutting imperatives facing the
rate of growth of the combined exports of software and corporate sector in the United States and elsewhere.

9 Adda247 Publications For any detail, mail us at


Publications@adda247.com
50+ Bank PO | Clerk Previous Year’s Papers 2016 – 2020

As a result, IT industry revenues grew in double digits, with (II) There is an abrupt decline in the rate of growth of
export earnings accounting for a large share of those the combined exports of software and ITs.
revenues. Employment grew rapidly, albeit from a low (III) Because of over-exploitation of
base and at a lower pace than revenues. And a service outsourcing opportunities.
industry to support the IT sector’s growth grew around the (a) Only (I) is correct
principal centres of its activity, suggesting that despite the (b) Only (II) is correct
absence of physical production the sector had backward (c) Both (I) and (III) are correct
linkages through which it delivered some economy-wide (d) Both (II) and (III) are correct
benefits. This combination of the characteristics of India’s (e) All are correct
IT success not only gave the industry a position of privilege
in the economy but made it the symbol of India’s ostensible 82. How the outsourcing opportunity helped India’s IT
post-globalisation success. industry in achieving exceptional growth?
However, there were a number of features of that model (I) It bettered India’s global delivery model for
that made it vulnerable to changes in circumstances. To software and ITeS based on cheap skilled labour.
start with, it had a high degree of dependence on exports (II) There was a significant growth in generation of
for growth, with the U.S. accounting for a very large share revenues of IT industry.
of those exports, followed by the European Union (E.U.) at (III) It helped in rapid growth of
a distant second. At the turn of the century, the U.S. market employment.
accounted for close to two-thirds of India’s IT exports and (a) Only (I) is correct
the E.U. for about a quarter, and even in 2015-16, the U.S. (b) Only (III) is correct
was first with 62 per cent and Europe second with 24 per (c) Both (I) and (II) are correct
cent. Little had changed for the industry. (d) Both (II) and (III) are correct
Secondly, software services (or code writing and (e) All are correct
customisation of different levels of sophistication) and 83. What does the author mean by the sentence, “Little had
ITeS, rather than IT products, accounted for an changed for the industry” as used in the passage?
overwhelming share of revenues. To garner those (a) Despite the subsequent growth in other sectors,
revenues, a workforce with essential IT skills and Indian IT industry failed to add value to the
familiarity with English, communication infrastructure, economy.
and the requisite organisation were the necessary (b) IT Industry still had a high degree of dependence
ingredients. But sustaining those revenues required
on exports for growth.
constant attention to cost competitiveness, which
(c) Software services and ITeS were responsible for
encouraged automation of the routine activities that
overwhelming share of revenues, rather than IT
constitute an important part of the industry’s operations.
products.
Thirdly, this output composition required combining
(d) Both (b) and (c)
offshore delivery with local services provision to
(e) All of the above
understand client requirements and customize services
and even run operations. So, on-site work remained an 84. Which of the following statements is/are true in
important component of the industry’s activity. In 2002- context of the passage?
03, 48 per cent of India’s exports of IT services was through (I) In 2002-03, 18.9 per cent of India’s exports of IT
the medium of a commercial presence on foreign soil and services was through the medium of a commercial
another 13.5 per cent through the presence of natural presence on foreign soil and another 16.1 per cent
persons. By 2015-16 those figures had come down to 18.9 through the presence of natural persons.
per cent and 16.1 per cent respectively. But the local (II) India’s IT success not only gave the industry a
presence, which ensured provision of 35 per cent of the position of privilege in the economy but made it
value services that had risen in value from Rs.31,100 crore the symbol of India’s ostensible post-globalisation
to Rs.5,76,310 crore between 2002-03 and 2015-16, was success.
undoubtedly large and crucial to the industry’s (III) A workforce with essential IT skills and
performance. familiarity with English, communication
Finally, a few firms (such as Tata Consultancy Services infrastructure, and the requisite organisation
(TCS), Infosys and Wipro) account for a very large share of were the necessary ingredients for drawing
the industry’s revenues, drawing attention to their attention of the outsourcing opportunity.
operations and brand as happened in the U.S. recently. (a) Only (I) is true
81. Why according to the passage India’s IT industry is (b) Only (II) is true
facing a sense of despondency? (c) Both (I) and (II) are true
(I) Because of unavailability of cheap skilled labour (d) Both (II) and (III) are true
force in the country. (e) All are true

10 Adda247 Publications For any detail, mail us at


Publications@adda247.com
50+ Bank PO | Clerk Previous Year’s Papers 2016 – 2020

85. Why according to the passage the growth rate of Agriculture in India has been facing many issues —
combined exports of software and ITeS slumped to a fragmented land holding, depleting water table levels,
new low? deteriorating soil quality, rising input costs, low
(a) Global circumstances combined with the specific productivity. Add to this vagaries of the monsoon. Output
nature of India’s IT prowess seem to be prices may not be remunerative. Farmers are often forced
responsible for this fall. to borrow to manage expenses. Also, many small farmers
(b) India’s over dependence on the exports of software not eligible for bank credit borrow at exorbitant interest
and ITeS for growth. rates from private sources. When nature rides roughshod
(c) Due to absence of physical production. over debt-ridden farmers in the form of erratic monsoon
(d) Lack of workforce with essential IT skills and and crop failures, they face grim options. Indebtedness is a
familiarity with English, and communication key reason for the many farmer suicides in the country.
infrastructure. Loan waivers provide some relief to farmers in such
(e) All of the above. situations, but there are debates about the long-term
effectiveness of the measure. Critics demand making
Directions (86-88): Choose the word/group of words
agriculture sustainable by reducing inefficiencies,
which is most similar in meaning to the word/group of
increasing income, reducing costs and providing
words printed in bold as used in passage.
protection through insurance schemes. They point out that
86. Prowess farm loan waivers are at best a temporary solution and
(a) Dexterity (b) Method (c) Composure entail a moral hazard — even those who can afford to pay
(d) Liberty (e) Serenity may not, in the expectation of a waiver. Such measures can
erode credit discipline and may make banks wary of
87. Garner
lending to farmers in the future. It also makes a sharp dent
(a)Pick up (b)Setup (c)Preserve
in the finances of the government that finances the write-
(d)Buy up (e)Amass
off. A blanket waiver scheme is detrimental to the
88. Imperative development of credit markets. Repeated debt-waiver
(a)Subtle (b)Piercing (c)Vital programmes distort households’ incentive structures,
(d)Supple (e)Mercurial away from productive investments and towards
Directions (89-90): Choose the word/group of words unproductive consumption and wilful defaults. These
which is most opposite in meaning to the word/group of wilful defaults, in turn, are likely to disrupt the functioning
words printed in bold as used in passage. of the entire credit system.
The real crisis for Indian farmer is that he or she is not in
89. Ostensible control of the produce, unlike other businesses, and is
(a)Avowed (b)Genuine (c)Alleged dependent on cartel of traders to fetch a decent price. The
(d)Illusory (e)Feigned cartel makes money in case of good or bad crop season as
90. Customize their margins remain intact. In fact, in case of a crop failure
(a)Sustenance (b)Standard (c)Maintenance the trader profit margin rises whereas the farmer is in
(d)Reshape (e)Comply distress without remunerative price. The governments –
Centre and states – have repeatedly failed to break the
Directions (91-95): Read the following passage carefully cartelisation and their effort to create farm infrastructure
and answer the questions given below it. through cold stores has helped the corporate sector more
Recent events — the UP government’s waiver of farmer than the farmers. Except some farmers in Maharashtra and
loans, dramatic protests by Tamil Nadu farmers in Delhi Punjab, most of the cold stores built with help of the
and a warning from the RBI Governor against loan waivers government subsidy are owned by corporates. So, now
— have once again brought farm loan write-offs under these corporates are buying produce in farms at cheap
public glare. rates, keep them in cold stores, repackage them and sell
Farm loans may be crop loans or investment loans taken to them in malls in cities at thrice the purchase price. Neither
buy equipment. Both farmers and banks reap a good the farmer gains nor the consumer.
harvest when all is well. But when there is a poor monsoon To be sure, the agriculture sector needs government
or natural calamity, farmers may be unable to repay loans. support but loan waivers are not the solution. On the
The rural distress in such situations often prompts States contrary, expenditure on loan waivers will eventually
or the Centre to offer relief — reduction or complete leave less fiscal space for public expenditure in agriculture.
waiver of loans. Essentially, the Centre or States take over India needs massive investment in areas such as irrigation,
the liability of farmers and repay the banks. Waivers are water conservation, better storage facilities, market
usually selective — only certain loan types, categories of connectivity and agricultural research. The problems in
farmers or loan sources may qualify. Indian agriculture are structural. They need long-term

11 Adda247 Publications For any detail, mail us at


Publications@adda247.com
50+ Bank PO | Clerk Previous Year’s Papers 2016 – 2020

solutions. Loan waivers will only end up complicating the (d) Loan Waivers will end up simplifying the problem
problem. The Indian economy has suffered a lot due to of farmers and economy.
competitive populism in the past. It’s time parties and (e) All are correct.
governments address the real issues.
Direction (96-98): The following sentences consist of three
91. According to the passage, why there is a need of blanks. You are provided with three words that fits
waiving the loans of the farmers? coherently in the sentence. Identify and mark the
(a) So that agricultural sector and economy do not get alternative catering the correct sequence of the words in
affected. which they must be filled to make the sentence
(b) Incapability of farmers to repay the loans due to contextually meaningful and grammatically correct.
natural disasters.
(c) As Private firms pressurize to repay the loans. 96. With the ___________________ of the Reserve Bank of
(d) To meet the long term solutions of farmers. India’s new bad loan ___________________framework that
(e) All of the above. seeks greater ___________________ from banks, Kotak
warned small and medium enterprise entities could
92. What are the disadvantages related to the loan waiver also be hit and banks will have to improve their
scheme? underwriting standards.
(a) It can abrade credit regulation and may make [I] Implementation
banks wary of lending to farmers in the future. [II] Disclosures
(b) It leads to less involvement of farmers for credit [III] Resolution
benefit from banks in the future. (a) (I), (II), (III) (b) (II), (I), (III)
(c) The loan waiver scheme perverts the households’ (c) (I), (III), (II) (d) (III), (II), (I)
incentive structure and leads to unproductive
(e) (III), (I), (II)
consumption.
(d) Both (a) and (c) are correct. 97. The department-related parliamentary
(e) All are true. ___________________, after reviewing the ___________________,
said the bridge course should not be made a
93. According to the passage, what needs to be done in
mandatory ___________________ in the present Bill.
order to resolve the issue?
[I] Proposal
(i) Banks should lend the appropriate amount of
[II] Provision
credit to farmers.
(ii) There should be Long term solutions of structural [III] Committee
problems of agriculture. (a) (I), (II), (III) (b) (II), (I), (III) (c) (I), (III), (II)
(iii) Investment in areas like irrigation, water (d) (III), (II), (I) (e) (III), (I), (II)
conservation, better storage facilities, etc. 98. Over the last year, the record of implementation of the
(a) Only (i) is correct steps in the peace accord has been ___________________,
(b) Only (ii) is correct though major strides were made in the form of
(c) Both (i) and (ii) are correct demilitarization and disbanding of the FARC and its
(d) Both (ii) and (iii) are correct ___________________into a ___________________ political force.
(e) All are correct [I] Legitimate
94. Which is the most appropriate title of the passage? [II] Conversion
(a) Need for farm loan write-offs. [III] Patchy
(b) The hazards of farm loan waivers. (a) (I), (II), (III) (b) (II), (I), (III)
(c) The real crises of Indian farmers. (c) (I), (III), (II) (d) (III), (II), (I)
(d) A Blanket waiver scheme. (e) (III), (I), (II)
(e) A Dramatic protests by Tamil Nadu farmers. Direction (99-100): Given below are sentences with a blank
95. Which of the following statements is false in context of that should be filled from the given alternatives. Identify
the passage? the most suitable expression that fits in the most
(a) For the rural distress, The Centre or States take appropriate manner in the sentence making it
over the culpability of farmers and repay the grammatically and contextually correct.
banks.
99. Investment bank Goldman Sachs downgraded its
(b) To regulate the expenses, farmers are forced to
forecasts for India’s economy on Tuesday in the wake
borrow.
of a more than $2 billion fraud at Punjab National Bank,
(c) A blanket waiver scheme is detrimental to the
________________________________ regulation of the banking
development of credit markets.
sector that would constrain credit growth.

12 Adda247 Publications For any detail, mail us at


Publications@adda247.com
50+ Bank PO | Clerk Previous Year’s Papers 2016 – 2020

(a) to warn a tighter spark 104. Despite giving the fact that almost half the work force
(b) warning it could spark tighter in the country is still engaged in farming, farmers’ woes
(c) warned them could spark tighter are naturally an important electoral plank.
(d) has warned it could sparked tighter (a) There is a fact that almost half
(e) warnings were it could spark tighter (b) Giving the fact that despite halving
(c) Given the fact that almost half of
100.________________________________ no fewer than 16 (d) The fact that states almost half of
accidents/derailments were caused by inept track (e) No correction required
management across five railway zones, and loss-
creating speed restrictions had been imposed on as 105. Tagore’s views on nationalism as expressed in his
many as 294 sections - on a permanent basis - because speeches in the US in 1916 and in China in 1924 as well
of deficient track-quality. were severely criticizing by both these countries,
(a) Considering that between 2014 and 2017 although they were appreciated by Albert Einstein and
(b) Since 2014 and 2017 expressed Romain Rolland.
(c) It questioned that while in 2014 and 2017 (a) had been criticized severely by both the countries
(d) Among 2014 and 2017 the note was (b) were severely criticized in both these countries as
(e) It noted that between 2014 and 2017 well
(c) was severely criticizing in both these countries as
Directions (101-105): Which of the phrases (a), (b), (c) and well
(d) given below each sentence should replace the phrase (d) were criticizing severely at both the countries as
printed in bold letters to make the sentence grammatically well
correct? If the sentence is correct as it is, mark (e) i.e., "No (e) No correction required
correction required" as the answer.
Directions (106-110): Following questions have words
101. The two Koreas have been divided along the world's given in bold each having five alternatives. Choose the word
most heavily fortified border since 1950-53 ending among the given options which has either most similar or
Korean War with an armistice, not a peace treaty. most opposite meaning to the bold word.
(a) since the 1950-53 Korean War ended with an
106. MEEK
armistice
(a)Pejorative (b)Assertive (c)Behest
(b) as the Korean War has ended with an armistice (d)Avid (e)Ubiquity
since 1950-53
(c) after Korean War ended during 1950-53 with an 107. PETRIFY
armistice (a)Appall (b)Unravel (c)Reek
(d) because the Korean War ended to an armistice (d)Invoke (e)Confiscate
during 1950-53 108. FIGMENT
(e) No correction required. (a)Filth (b)Fervor (c)Altercation
102. The setting up of territorial nation-states in South Asia (d)Incursion (e)Delusion
on the basis of religious or ethnic identities cannot 109. INNOCUOUS
bring any improvements in the lives of people. (a)Pronounce (b)Importunate (c)Obnoxious
(a) did not bring up any improvement (d)Juxtapose (e)Affluent
(b) have not brought about any improvement
(c) has not brought in the improvement 110. PROSCRIBE
(d) could not bring any of the improvement (a)Primitive (b)Condemn (c)Requite
(d)Circumvent (e)Precept
(e) No correction required
Directions (111-115): In the following passage there are
103. The annual election to choose five new non-permanent
blanks, each of which has been numbered. These numbers
members of the UN Security Council produced both the
are printed below the passage and against each five words
expected and the near preposterous.
are suggested, one of which fits the blank appropriately.
(a) has produced the unexpected and the near
Find out the appropriate word in each case.
preposterous
On October 2, 1983 the Grameen Bank Project (111) the
(b) has been producing both the expected and Grameen Bank. We invited the Finance Minister to be the
preposterous event Chief Guest at our (112) ceremony. But when the Ministry
(c) is both the expected and near preposterous came to (113) that the ceremony would take place in a
(d) produces both the expectation and near remote district, they said it would not be an (114) place to
preposterous launch a Bank and that the ceremony should be (115) in
(e) No correction required Dhaka so that all the top Government Officials could attend.

13 Adda247 Publications For any detail, mail us at


Publications@adda247.com
50+ Bank PO | Clerk Previous Year’s Papers 2016 – 2020

111. (a) reorganised (b) merged (c) named (d) to keep India under British control would require
(d) converted (e) became a huge number of European troops.
(e) No replacement required
112. (a) opening (b) closing (c) dedicated
(d) inaugurate (e) induction 118. (a) which was not possible.
(b) who ate from the same kitchens.
113. (a) reveal (b) know (c) aware
(c) who could not be taken for granted.
(d) inform (e) acquaint
(d) who held India only for trade and commercial
114. (a) excellent (b) available (c) inauspicious interests.
(d) appropriate (e) obvious (e) No replacement required
115. (a) invited (b) assembled (c) done 119. (a) were on the losing side in World War II.
(d) shifted (e) held (b) from the country in the event of an Indian
rebellion.
Directions (116-120): In the following questions, certain
(c) who delivered the final nail in the coffin of the
parts of the paragraph are highlighted, which suggest that
imperial masters.
they may or may not belong to the paragraph. There are five
(d) fought valiantly on India’s northeast borders but
sentences given as options including the highlighted part
against each number. Choose the option which suits best lost the battle.
(e) No replacement required
into the theme of the paragraph as well as to the contextual
meaning of the sentence. If the highlighted portion does not 120. (a) it is hard to appreciate that this is palpably false
require any replacement, choose option (e) i.e. “No and obfuscates the truth
replacement required” as your answer. (b) the Indian freedom struggle was written at the
The British ruled India only on the strength of the British behest of the first rulers of Independent India
Indian army (116) is likely to be reflected in the quality of (c) sections of the armed forces became restive
work too. Without this army, the British could never hold (d) the INA activities of Bose had weakened the
together, ‘the jewel in the crown’ and exploit its resources. foundation of the British Empire in India
Smartly, the British had raised the army on communal lines (e) No replacement required
with regiments based on castes and (117) communities to
perpetuate feudal feelings and prevent the development of a
national consciousness. Netaji Subhas Bose realizing this
raised an Indian National Army (INA) which had men of all
castes and communities in mixed regiments and (118) who
made them leave India in a hurry. The poorly equipped INA
men (recruited in South-East Asia from Indian prisoners of
war) (119) were the real heroes whom led India to freedom.
26,000 of the 60,000 INA soldiers perished on the
battlefront. Being with the Axis Powers, they were on the
losing side in World War II. Netaji disappeared after the
conclusion of the World War and (120) what happened to
him thereafter remains a matter of conjecture — albeit it is
now proven that no air crash in which he is supposed to
have died ever took place.
116. (a) among Indian soldiers who fought for the British.
(b) whose soldiers were Indians but officers were
British.
(c) had weakened the foundation of the British
Empire in India.
(d) which results in massive inequality of opportunity.
(e) No replacement required
117. (a) contained an intelligence assessment of the loyalty
of Indian army officers.
(b) began fearing an encore of the 1857 rebellion.
(c) they realized that Indian forces could no longer be
trusted to prop up the British.

14 Adda247 Publications For any detail, mail us at


Publications@adda247.com
50+ Bank PO | Clerk Previous Year’s Papers 2016 – 2020

Solutions

REASONING ABILITY

Directions (1-5): during traveling , it does not mean all travelers are
suffering from mentioned diseases.
Directions (8-11):

1. (e); 2. (d); 3. (b);


8. (c);
4. (a); 5. (d);
9. (a);
6. (c); For (a)- This can’t be hypothesized from the given
statement because preference in the sense of
popularity has been given to mobile instead of T.V.
For (b)- This can’t be hypothesized from the given
statement because there is direct comparison
between T.V and mobile phone in statement. There is
nothing mentioned about outdoor activities in the
statement. 10. (a); 11. (c);
For (c)- This can be hypothesized from the given
statement as it is mentioned in the given statement Directions (12-16):
that youngsters spend most of their time on mobile Month Date Persons Colour/Fruit
phones. March 7 E APPLE
For (d)- This can’t be hypothesized from the given March 9 B GREEN
statement because nothing is mentioned about that March 16 A YELLOW
older generation is comfortable in using mobile March 19 D BANANA
phones or not. July 7 F CHERRY
For (e)- This can’t be hypothesized from the given July 9 H RED
statement because it is a course of action, it cannot be July 16 G BLUE
an assumption. July 19 C MANGO
7. (d); For (a)-This can’t be assumed from the given
12. (a); 13. (b); 14. (a);
statement because there is no any comparison
between mosquitoes and other animals in the 15. (a); 16. (c);
statement.
For (b)- This can’t be assumed from the given Directions (17-21):
statement because Zika, Dengue and Chikungunya 17. (c); Either from Statement I or Statement II we get our
are considered as dangerous diseases but is not said final answer. The code for ‘Festival’ is ‘ni’.
anything about treatment of these diseases.
For (c)- This can’t be assumed from the given 18. (d); From-I From, II
statement there is no discussion about any stats in
the statement.
For (d)- This can be assumed from the given
statement as it is mentioned in the given statement
that Protect yourself and your family from
mosquitoes during travelling. so it can be assumed
that there are more chances to get infected from
mosquitoes while travelling.
For (e)- This can’t be assumed from the given 19. (e); From Statement I and II together we get our final
statement because there is only a suggestion that answer. E is the second tallest.
protect yourself and your family from mosquitoes D > E > C > A > F > B.

15 Adda247 Publications For any detail, mail us at


Publications@adda247.com
50+ Bank PO | Clerk Previous Year’s Papers 2016 – 2020

20. (e);From statement I and II, we get our final answer i.e. Step I: 181 peak utility 76 emerge beautiful 37 51 visible 86
Bulbul sits second to the left of sparrow. know 272
Step II: beautiful 181 peak utility 76 37 51 visible 86 know 272
emerge
Step III: 373 beautiful 181 peak utility 76 visible 86 know 272
21. (d); From statement I and II together we can’t get our emerge 514
final answer as either P or S sits on the fourth floor. Step IV: know 373 beautiful181 utility 76 visible 86 272
emerge 514 peak
Step V: 765 know 373 beautiful181 utility visible 272 emerge
514 peak 866
Step VI: utility 765 know 373 beautiful181 272 emerge 514
peak 866 visible

Directions (22-26): 28. (d); 29. (c); 30. (d);


Person Date City 31. (b); 32. (b);
Q 7th Pune
33. (c); For (a)- This statement does not substantiates the
R 9th Pune
given statement as it mentioned that Our education
S 9th Patna
system today encourages excellence whereas given
T 7th Chandigarh
statement states that Education has been a problem
U 9th Patna
in our country.
V 7th Kochi
For (b)- This statement does not substantiates the
W 4th Patna
given statement because it mentioned that Our new
X 7th Patna education system creates entrepreneurs and
Y 4th Kochi foundation of a knowledge based economy.
Z 2nd Pune For (c)- This statement substantiates the given
22. (d); 23. (e); 24. (c); statement as it mentioned that India needs internet
and technology if it has to teach all population in
25. (d); 26. (a); remote villages which is also mentioned in the given
27. (c); For (a)- This can’t be the reason behind more number statement that Half the country does not even today
of accidents at night as population increase could not have access to proper education, and only a small
lead to more number of accidents during night. fraction can go to university.
For (b)- This could not be the appropriate reason as For (d)- This statement does not substantiates the
preference does not matter too much for the reason given statement because it mentioned that Our
of death during night time. education system is geared towards teaching and
For (c)- This could be the major cause behind more testing knowledge whereas given statement states
number of accidents at night as improper installation that Education has been a problem in our country.
of traffic signal and lightning arrangement is the For (e)- This statement does not substantiates the
major reason behind it. given statement because it mentioned that We may
For (d)- This could not be the major cause of the have the most number of engineering graduates in
given statement as the traffic rules are same in both the world whereas given statement states that Half
day and night time also. the country does not even today have access to
For (e)- This could not be the major cause behind proper education, and only a small fraction can go to
more number of accidents at night as Traffic violation university.
is a punishable offence in India.
Directions (34-38):
Directions (28-32):
Logic: - There are six numbers and six words in the input. In
the first step the numbers are arranged in ascending
order from both the ends with a natural number
starting from 1 at unit place in left end number and 34. (c); 35. (d); 36. (c);
with a natural number 2 at unit place in right end
37. (a); 38. (a);
number. After that in second step the words are
arranged in alphabetical order from both the ends. 39. (b); For (a) It may be the possible approach but can’t be
And then again number are arranged in third step the best suited option as dowry is like stubborn
and words are arranged in forth step and so on. custom which cannot be eradicate easily.
Input: peak 18 utility 76 emerge 27 beautiful 37 51 visible 86 For (b) It will be the best suited option as it is
know necessary to change the thinking of people but most

16 Adda247 Publications For any detail, mail us at


Publications@adda247.com
50+ Bank PO | Clerk Previous Year’s Papers 2016 – 2020

important thing is to make dowry system as illegal it describes that the meeting between Trump and
and make it as punishable offence. Modi will strengthen the ties between India and US.
For (c) It will not make any major changes in the For II-This statement is not in line with the given
thinking of persons. statement as it states about Narendra Modi’s visit to
For (d) It is right that it has been set in the nerves of Sri lanka whereas the given statement describes
people as one of the necessities but it has not Modi’s first meeting with Trump.
suggested any action to get out this stubborn custom. For III-This statement is also in line with the given
For (e) It will make some impact all over India, as statement as it describes that the meeting of Modi
others states will follow Mr. Kumar but this step will and Trump will further make strong bilateral relation
not uproot it. between both the countries.
For IV-This is also in line with the given statement
40. (a); In the above question we have to find which because this statement states that this meeting will
statements do not support the given statement. give an opportunity to make strong partnership
For I-This statement is in line with the above between India and US.
statement because it supports the given statement as

QUANTITATIVE APTITUDE

41. (c); Offline contestant in village A = 350  44 = 275 350


=  44 = 275
56 56
contestant who complete the survey online  Required difference = 35
= 350 – 61 = 289
contestant who complete the survey offline 45. (a); Required sum
= 275 – 61 = 214 = 350  12 + 560  30 + 465  20 + 480  20
289 – 214 56 35 60 40
∴ Required % =  100 ≈ 35% = 75 + 480 + 155 + 240 = 950
214
42. (a); Total no. of contestant from village C who complete 46. (d); Number of contestants, who complete the survey
the survey online and offline from village C
465
= 465 + 
465  100– 108 = 667
 40 
=
  – 108 ≈ 667 60
 60 
and number of contentstants participants from
Total no. of contestant from village B who complete
village D
the survey 480
100 = 1200
= 560 +  560  65 
 – 92 = 1508 40
  35 
 Required percentage = 667
∴ Required number = 1508 – 667 = 841  100 = 56%
1200
43. (c); contestant who didn’t completed the survey
online = 8  190 = 80 47. (b); (A's profit) : (B's profit) : (c's profit)
19
contestant who didn’t completed the survey = 600  12 : 500  4 : 5x  8
offline = 11  190 = 110 = 180 : 50 : x
19 x
 Males contestant who completed the survey online ∴ C’s profit = × 24000
230 + x
65
=  (480 − 80) = 260 x
100  × 24000 = 5600
and 230 + x
females contestants who completed the survey  30x = 1610 + 7x
offline  x = 70%
60  480 
=   60 – 110 = 366 48. (a); Length of platform = 21 × 19 – 216 = 183 m
100  40  Let n boxes are added
 Required difference = (720 –110–366) 216 + 183 + 21n = 21 × 26
– ( 480 – 80 – 260 ) = 244 – 140 =104  21n = 147
44 (d); Offline contestants of village C n=7
= 465  40 = 310 49. (d); B will complete the work alone in
60
Offline contestants of village A 3
= × 36 = 27 days
4

17 Adda247 Publications For any detail, mail us at


Publications@adda247.com
50+ Bank PO | Clerk Previous Year’s Papers 2016 – 2020
A — 36 3 55. (c);
108
B — 27 4
Let total units of work = 108
No. of units done by A in 1 day = 3
No. of units done by B in 1 day = 4
Total work done in 2 days = 7 units 56. (b); Players from Club ‘D’ who play either of Cricket or
Work done in 30 days = 7 × 15 = 105 units Football
12 20
Remaining work will be done by A in 108 − 105 = 2400 × + 3200 ×
100 100
3
= 288 + 640 = 928
= 1 day.
Players of football from both club ‘C’ and ‘E’
∴ Total days taken = 31 days (13+21)
= 3200 × = 1088
100
50. (c); Let’s total salary of Rakesh = 100 %
∴ Required diff = 1088 – 928 = 160
12% of the salary is added as PPF.
Remaining Part = (100 – 12) = 88% 57. (c); Male players from club ‘E’ who play either of Cricket
3 or Football
Amount spent on clothes = of 88% = 33% 24 21 9
8 = [2400 × + 3200 × ] ×
100 100 16
9 9
Difference between PPF and cloth expanses = 33 - 12 = [576 + 672] × = 1248 × = 702
16 16
= 21% of salary = 10500 Players from club ‘F’ who playing football
Total salary = 50000 5
= 3200 × = 160
Other expanses = House Rent expanses + 1500 100
House Rent expanses + Other expanses = (100 - 33 - Required percentage= 702 100 = 438 3 %
12)% of salary 160 4
= 55% of salary = 27500 58. (a); Female players from club ‘E’ playing cricket
House Rent expanses + House Rent expanses + 1500 24 7
= 2400 × × = 336
= 27500 100 12
Female players from club ‘C’ playing football
2 × House Rent expanses = 27500 - 1500 = 26000 13 6
= 3200 × × = 192
House Rent expanses = 13000 100 13
Required sum = 336 + 192 = 528
51. (b);
59. (d); Players of club D and B who play football
36
= 3200 × = 1152
100
Players of club D and F who playing Cricket
23
= 2400 × = 552
100
(1152 –552)
∴ Required %= ×100≈108.7% =109%
552

60. (b); Players who play Cricket from both club A and C
33
= 2400 × = 792
100
52. (a);
Players who playing Football from Club B, D and F
together
41
= 3200 × = 1312
100
∴ Required difference = 1312 – 792 = 520
61. (d); Probability of drawing one green ball
53. (b); x 2
= = ⇒x = 8
12+x 5
2 2
C 5×4 2
∴ Required probability = 15 2
= =
15×14 21
C
62. (a); A + B = 41 …(i)
54. (c); C -1 = A + 2
C=A+3
And
A+4=B-1
⇒ B = A + 5 …(ii)

18 Adda247 Publications For any detail, mail us at


Publications@adda247.com
50+ Bank PO | Clerk Previous Year’s Papers 2016 – 2020

From (i) + (ii) 71. (c); Invalid voter of village C in 2005


36
A = 18 years = 4000 × = 2250
64
B = 18 + 5 = 23 years
Valid voter of village C in 2001
C = 18 + 3 = 21 years 19
A 3 4 = 4000 × = 3040
= D = × 18 = 24 years 25
D 4 3 Now, invalid voter of village C in 2001
∴ Required difference = 24 – 21 = 3 years 24
3040 × = 960
76
63. (a); Radius of cylinder = side of equilateral ∆ ∴ Required diff. = 2250 – 960 = 1290
3 2
 a = 16 3 , where a = sides of ∆ 72. (a); Total valid votes of village A in 2005
4 = 2500 ×
100
×
80
 a² = 64 20 100

⇒ a = 8 cm = 10,000
And, height of cylinder = 3 × 8 = 24 cm Total valid votes excluding Nota in village A in 2005
90
 Volume of cylinder = πr²h = 10,000 × = 9,000
100
= π × 8² × 24 = 1536 π cm³ ∴ According to que.
x + (x + 200) = 9000
64. (b); Compound interest earned in 2 years
20 2 ⇒ x = 4400
= 8000 [(1 + ) ] − 8000 = 3520 Required no. of votes of loosing candidates = 4400
100
Let amount invested in another scheme is Rs. P. 73. (b); Let the total voters in 2005 are x.
P×8×4
3520 = 500% of Total voters in 2001
100
3520×100 3 26
⇒P= = (1 + )x = x
32×5 23 26 25
23
x× × 130
⇒ P = 2200
23 100
∴ Required ratio = 35 =
x× 161
100

 Total investment = 8000 + 2200 = 10,200 74. (d); Total invalid male of village E in 2001 = 2000 – 1600
65. (a); In 1000 ml of mixture, = 400
Alcohol = 700 ml Total valid voters in 2001
100
Water = 300 ml = 1600 × = 2500
64
Let x ml of alcohol is mixed. 100
Total voters in 2001 = 2500 × = 3125
According to question 80
300 15 400
= ∴ Required % = × 100 = 12.8%
1000+x 100 3125
6000 = 3000 + 3x 75. (c); Let total voters of village B in 2001 = x
X= 1000 ml & Total voters of village D in 2005 = y
75
66. (e); 2πr 2 = 616 ∴

100
=
16
⇒ =
x 32
616 7 30
y× 3 y 15
r2 = × 100
2 22 17 1
r 2 = 98 ∴ Required % = × 100 = 53 %
32 8
2
∴ Volume = πr 3 Solutions (76-80): Let number of chairs, tables and wardrobes
3
2 22
= × × 98 × 7√2 = 2032.69 cm3 sold by A in August be 42x, 36x and 23x. Also, let chairs sold
3 7
by A in August, September and October be 14y, 23y and 27y
67. (b); J : A = 6 : 11 respectively.
A)11x − 6x = 25 ∴ 42x = 14y ⇒ y = 3x
So we can find out ratio of their age 5 year ago. and, 23x = 23y – 230
B) (11x + 5) − (6x + 5) = 25 ⇒ x = 5 and y = 15
C) 11x + 6x = 85 Now,
Chairs sold by B in September = 665 – 345 = 320
68. (e); Cost price per unit is not given. Chairs sold by B in August = 320
1 Tables sold by B in September
69. (e); 8M + 6 W =
21 = Chairs sold by A in Aug = 210
1
1.5 (8W + 6W) = 1.5 × ∴ Table sold by A in September = 400 – 210 = 190
21
12M + 9W =
1 Wardrobes sold by B in September = wardrobes sold by A in
14 Aug = 115
Work will be completed in 14 days. No information is ∴ Wardrobes sold by A in September = 210 – 115 = 95
required. Chairs sold by B in October = 1025 – 320 – 320 = 385
70. (d); Question can’t be answered because direction of Tables sold by A in October
movement of the trains are not given. = (1025 – 480) – (180 + 190) = 175

19 Adda247 Publications For any detail, mail us at


Publications@adda247.com
50+ Bank PO | Clerk Previous Year’s Papers 2016 – 2020
11
Tables sold by B in August = × 180 = 165 76. (c); Total chairs sold by B in September and October =
12
38
Tables sold by B in October = × 175 = 190 320 + 385 = 705
35
Wardrobes sold by B in August = 1075 – (210 + 320 + 180 + 77. (a); Required percentage
165 + 115) = 85 175−120 5
Let wardrobes sold by A in October be a and that by B be b in = × 100 = 45 %
120 6
October
∴ a = b – 35 and a + b = 205 78. (d); Asked difference = 320 – 210 = 110
⇒ a = 85 and b = 120
Chair Table Wardrobe 180 6
Months 79. (b); Required ratio = =
A B A B A B 210 7
Aug 210 320 180 165 115 85
Sep 345 320 190 210 95 115 80. (b); Wardrobes sold by B in Oct = 120
Oct 405 385 175 190 85 120

ENGLISH LANGUAGE

81. (b); Refer the first paragraph, “According to data from the symbol of India’s ostensible post-globalisation
Reserve Bank of India (RBI), the rate of growth of the success.” and “To garner those revenues, a workforce
combined exports of software and ITs fell from 20.8 with essential IT skills and familiarity with English,
per cent in 2012-13 to 14.9 per cent in 2014-15 and communication infrastructure, and the requisite
to a low of 7.3 per cent in 2015-16.” Hence only organisation were the necessary ingredients.” Hence
statement (II) is the correct reason. only statement (II) is true in context of the passage.
82. (e); Refer the last few lines of first paragraph, “India’s IT 85. (a); Refer the first paragraph, “According to data from the
industry recorded remarkable growth over a long Reserve Bank of India (RBI), the rate of growth of the
period because it exploited an outsourcing combined exports of software and ITeS fell from 20.8
opportunity by perfecting a global delivery model for
per cent in 2012-13 to 14.9 per cent in 2014-15 and
software and ITeS based on cheap skilled labour.”
to a low of 7.3 per cent in 2015-16. Global
and the first few lines of second paragraph, “As a
circumstances combined with the specific nature of
result, IT industry revenues grew in double digits,
with export earnings accounting for a large share of India’s IT prowess seem to be responsible for this
those revenues. Employment grew rapidly, albeit fall.” Hence (a) is the correct option in context of the
from a low base and at a lower pace than revenues.” passage.
Hence all three statements are correct in context of 86. (a); Prowess means skill or expertise in a particular
the passage. activity or field. Dexterity means skill in performing
83. (d); Refer the first few sentences of the third paragraph, tasks, especially with the hands. Hence both are
“However, there were a number of features of that similar in meanings.
model that made it vulnerable to changes in Serenity means the state of being calm, peaceful, and
circumstances. To start with, it had a high degree of untroubled.
dependence on exports for growth, with the U.S.
accounting for a very large share of those exports…” 87. (e); Garner means gather or collect (something,
and the first sentence of the fourth paragraph, especially information or approval). Hence “amass” is
“Secondly, software services (or code writing and the word most similar in meaning to it.
customisation of different levels of sophistication) 88. (c); Imperative means a factor or influence making
and ITeS, rather than IT products, accounted for an something necessary. Hence ‘vital’ is the word most
overwhelming share of revenues.” Hence both the similar in meaning to it.
statements (b) and (c) are correct in context of the
Supple means bending and moving easily and
passage.
gracefully; flexible.
84. (b); Refer the statements, “In 2002-03, 48 per cent of Mercurial means subject to sudden or unpredictable
India’s exports of IT services was through the changes of mood or mind.
medium of a commercial presence on foreign soil and
89. (b); Ostensible means stated or appearing to be true, but
another 13.5 per cent through the presence of natural
not necessarily so. Hence ‘genuine’ is the word most
persons.”, “This combination of the characteristics of
opposite in meaning to it.
India’s IT success not only gave the industry a
Avowed means that has been asserted, admitted, or
position of privilege in the economy but made it the
stated publicly.

20 Adda247 Publications For any detail, mail us at


Publications@adda247.com
50+ Bank PO | Clerk Previous Year’s Papers 2016 – 2020

90. (b); Customize means modify (something) to suit a a mandatory provision in the present Bill.” Therefore,
particular individual or task. Hence ‘standard’ is the option (e) becomes the most viable choice.
word most opposite in meaning to it. Proposal means a plan or suggestion, especially a
Sustenance means the maintaining of someone or formal or written one, put forward for consideration
something in life or existence. by others.
91. (b); Refer to the fourth sentence of second paragraph, Provision means the action of providing or supplying
“The rural distress in such situations often prompts something for use.
States or the Centre to offer relief — reduction or Committee means a group of people appointed for a
complete waiver of loans.” Hence (b) is the correct specific function by a larger group and typically
option in context of the passage. consisting of members of that group.
92. (d); Refer the fourth paragraph, “Repeated debt-waiver 98. (d); The correct sequence of the words to be filled in the
programmes distort households’ incentive sentence is (III), (II), (I) to gain a grammatically
structures, away from productive investments and correct and contextually meaningful sentence. Thus,
towards unproductive consumption and wilful the sentence formed is “Over the last year, the record
defaults.” and “Such measures can erode credit of implementation of the steps in the peace accord
discipline and may make banks wary of lending to has been patchy, though major strides were made in
farmers in the future.” Hence both (a) and (c) are true the form of demilitarization and disbanding of the
in context of the passage. FARC and its conversion into a legitimate political
93. (d); Refer to the sixth paragraph, “India needs massive force.” Therefore, Option (d) becomes the most
investment in areas such as irrigation, water suitable answer choice.
conservation, better storage facilities,” and “The Legitimate means conforming to the law or to rules.
problems in Indian agriculture are structural. They Conversion means the process of changing or causing
need long-term solutions.” Hence both the options something to change from one form to another.
(ii) and (iii) are correct. Patchy means existing or happening in small, isolated
94. (b); The author in the passage emphasized on the areas.
disadvantages of loan waiving scheme to the 99. (b); The expression “warning it could spark tighter” aptly
economy and also he has mentioned the steps that fits in the blank making the sentence coherent and
need to be implemented. Hence the title “The hazards logical. All the other expressions are either
of farm loan waivers” is the most appropriate one.
grammatically incorrect or contextually irrelevant.
95. (d); Refer to the last paragraph, “Loan waivers will only Thus, forming the sentence as “Investment bank
end up complicating the problem”. Hence statement Goldman Sachs downgraded its forecasts for India’s
(d) is false in context of the passage. economy on Tuesday in the wake of a more than $2
96. (..); The correct sequence of the words to be filled in the billion fraud at Punjab National Bank, warning it
sentence is (I), (III), (II) to gain a grammatically could spark tighter regulation of the banking sector
correct and contextually meaningful sentence. Thus, that would constrain credit growth.” Hence, option
the sentence formed is “With the implementation of (b) becomes the most viable answer choice.
the Reserve Bank of India’s new bad loan resolution 100. (e); The expression that fits appropriately in the blank is
framework that seeks greater disclosures from “It noted that between 2014 and 2017”. It justifies the
banks, Kotak warned small and medium enterprise
contextual meaning of the sentence and provides
entities could also be hit and banks will have to
coherence to it. All the other alternatives fail to
improve their underwriting standards.” Hence,
comprehend the grammatical syntax of the sentence.
option (c) is the most suitable choice.
Thus, the sentence formed is “It noted that between
Implementation means the process of putting a
decision or plan into effect; execution. 2014 and 2017 no fewer than 16
Disclosures means the action of making new or secret accidents/derailments were caused by inept track
information known. management across five railway zones, and loss-
Resolution means a firm decision to do or not to do creating speed restrictions had been imposed on as
something. many as 294 sections - on a permanent basis -
because of deficient track-quality.” Hence, option (e)
97. (e); The correct sequence of the words to be filled in the
is the most suitable answer choice.
sentence is (III), (I), (II) to gain a grammatically
correct and contextually meaningful sentence. Thus, 101. (a); “since the 1950-53 Korean War ended with an
the sentence formed is “The department-related armistice” is the correct phrase to replace the bold
parliamentary committee, after reviewing the part as ‘since’ in this case is used to describe the
proposal, said the bridge course should not be made reason. Hence (a) is the correct option.

21 Adda247 Publications For any detail, mail us at


Publications@adda247.com
50+ Bank PO | Clerk Previous Year’s Papers 2016 – 2020
102. (b); “have not brought about any improvement” is the 110. (b); The word “proscribe” means denounce or condemn.
correct phrase replacement as the phrase “bring Hence the word is similar in meaning to the word
about” means to make something happen, especially “condemn”. Hence option (b) is the correct choice.
to cause changes in a situation. Hence (b) is the Primitive means relating to, denoting, or preserving
correct option. the character of an early stage in the evolutionary or
historical development of something.
103. (e); The given sentence is grammatically correct. Requite means make appropriate return for (a
104. (c); “Given the fact that almost half of” is the correct favour, service, or wrongdoing).
phrase replacement as it makes the sentence Circumvent means overcome (a problem or
grammatically correct. Other options are difficulty) in a clever and surreptitious way.
grammatically incorrect. So (c) is the correct choice. Precept means a general rule intended to regulate
behaviour or thought.
105. (b); As the sentence is in passive form, “were severely
criticized in both these countries as well” is the 111. (e); 112. (a); 113. (b);
correct phrase to make the sentence grammatically 114. (d); 115. (e);
correct.
116. (b); The given highlighted portion in the paragraph does
106. (b); The word “meek” means quiet, gentle, and easily not make the sentence meaningful and it gives an
imposed on; submissive. The word “assertive” means incomplete sense to the paragraph. Among the given
having or showing a confident and forceful options, only statement (b) finds a logical attachment
personality. Hence both are opposite in meanings to with the theme of the paragraph which is about the
each other. Indian soldiers who fought for and against British
Pejorative means expressing contempt or army. It is to be noted that the first sentence
disapproval. generates a clear idea about the British Indian army
Behest means a person's orders or command. whose soldiers were Indians but officers were British
Avid means having or showing a keen interest in or i.e. the highlighted portion should be replaced by that
enthusiasm for something. statement which further explains the meaning of the
Ubiquity means the fact of appearing everywhere or term. Other choices given as options are irrelevant in
of being very common. the context of their usage in the sentence. Hence
option (b) is the correct choice.
107. (a); The word “petrify” means make (someone) so
frightened that they are unable to move. The word 117. (e); The given statement does not require any
replacement as it correctly follows the theme of the
“appall” means greatly dismay or horrify. Hence both
paragraph. It complements the prior part of the
these words are similar in meanings to each other.
sentence contextually and thus gives a complete
Unravel means investigate and solve or explain
meaning to the sentence. All other options are out of
(something complicated or puzzling). the context. Hence option (e) is the correct choice.
Reek means smell strongly and unpleasantly; stink.
Invoke means give rise to; evoke. 118. (b); Read the given sentence carefully, there is a mention
Confiscate means take or seize (someone's property) of different sections of people who formed the part of
with authority. Indian National Army (INA). The highlighted portion
should complement its prior part which is not
108. (e); The word “figment” means a thing that someone happening in this case, it should be replaced by the
believes to be real but that exists only in their expression which follows the former part. Among the
imagination. The word “delusion” means the action of given options, only expression (b) gives the exact
deluding or the state of being deluded. Hence both meaning to the sentence. Other options are
are similar in meanings to each other. unsuitable and irrelevant. Hence option (b) is the
Filth means disgusting dirt. correct choice.
Fervor means intense and passionate feeling.
119. (d); The highlighted portion in the sentence is completely
Altercation means a noisy argument or
incorrect as it gives a contrast meaning to the
disagreement, especially in public.
sentence. It doesn’t follow its previous part which
Incursion means an invasion or attack, especially a
states that INA men were poorly equipped. Among
sudden or brief one. the given options, only expression (d) can be
109. (c); The word “innocuous” means not harmful or matched with the idea behind the sentence. The
offensive. The word “obnoxious” means extremely expression gives a logical meaning to the sentence
unpleasant. Hence both are opposite in meanings. and thus allows it to follow the paragraph coherently.
Pronounce means declare or announce in a formal or Other options can be ignored owing to their variable
solemn way. meanings to the sentence. Hence option (d) is the
Importunate means persistent, especially to the point correct choice.
of annoyance. 120. (e); Read the sentence carefully, it is quite clear that the
Juxtapose means place or deal with close together for sentence is complete and meaningful. It doesn’t
contrasting effect. require any replacement as it correctly follows the
Affluent means (especially of a group or area) having theme of the paragraph. Hence option (e) is the
a great deal of money; wealthy. correct choice.

22 Adda247 Publications For any detail, mail us at


Publications@adda247.com
50+ Bank PO | Clerk Previous Year’s Papers 2016 – 2020

1 Adda247 Publications For any detail, mail us at


Publications@adda247.com
50+ Bank PO | Clerk Previous Year’s Papers 2016 – 2020

Mock IBPS RRB PO Mains 2016


38
REASONING ABILITY

Directions (1-5): Study the following information Directions (7): This question consists of four statements
carefully to answer the given question. followed by five conclusions. Consider the given
Twelve people are sitting in two parallel rows containing statements to be true even if they seem to be at variance
six people each, in such a way that there is an equal with commonly known facts. Read all the conclusions and
distance between adjacent persons. In row-1 A, B, C, D, E then decide which of the given conclusions does not
and F are seated and all of them are facing south. In row-2 logically follow from the given statements using all
P, Q, R, S, T and U are seated and all of them are facing statements together.
north. Therefore, in the given seating arrangement each 7. Statements: All salary are funds.
member seated in a row faces another member of the other Some funds are trajory.
row. U sits third to right of S. S faces F and F does not sit at All trajory are white.
any of the extreme ends of the line. D sits third to right of All white are paisa.
C. R faces C. The one facing E sits third to right of P. B and P Conclusions: (a)Some paisa are fund.
do not sit at the extreme ends of the line. T is not an (b) All white being salary is a possibility.
immediate neighbour of U and A is not an immediate (c) At least some paisa are trajory.
neighbor of C. (d) All white can be Paisa.
1. Who amongst the following represent the people (e) Some funds are white.
sitting at extreme ends of the rows?
(a) RF (b) TA (c) DR 8. If A is brother of K and K is married to J. F is the mother
(d) CQ (e) SA of J and Y is the only child of K. Then how is A related
to Y?
2. Four of the following five are alike in a certain way and (a) Father (b) Brother-in-law
thus form a group. Which is the one that does not (c) Uncle (d) Can’t be determined
belong to that group? (e) Brother
(a) B–T (b) A–Q (c) C–S
9. Kapil has twelve years of typing experience behind
(d) F–P (e) D–R
him; therefore, if you are looking for an efficient typist
3. Who amongst the following faces D? to enter your data into the new system, you need look
(a) T (b) P (c) Q no further.
(d) R (e) None of these The speaker assumes that:
(a) Twelve years of practice ensures typing efficiency.
4. Four of the following five are alike in a certain way and
(b) The type of typing required for the new system is
thus form a group, which is the one that does not
identical to what Kapil has been doing.
belong to that group? (c) Kapil’s job profile is the best that the new
(a) D (b) S (c) U employer is going to get.
(d) T (e) A (d) Kapil is an outstandingly fast and accurate typist.
5. How many persons are sitting between R and T? (e) Kapil will fit well into the new office.
(a) One (b) Two (c) Three Directions (10-14): Each of the questions given below
(d) Four (e) None of these consists of a question and two statements numbered I and
6. Men start walking towards East from Point A. Before II. You have to decide whether the data provided in the
taking a right turn he moves 5 meters. Now he moves statements are sufficient to answer the question.
4 meters and then after taking a right turn he moves 2 (a) If statement I alone is sufficient to answer the
meters and reaches point D. then in which direction question, but statement II by itself is not sufficient
point A with respect to point D? to answer the question.
(a) South (b) North-West (b) If statement II by itself is sufficient to answer the
(c) South-East (d) North question, but statement I alone is not sufficient to
(e) None of these answer the question.

2 Adda247 Publications For any detail, mail us at


Publications@adda247.com
50+ Bank PO | Clerk Previous Year’s Papers 2016 – 2020

(c) If statement either I or II is sufficient to answer the Directions (16-20): Study the following information
question. carefully to answer the given question.
(d) If both the statements I and II taken together are Seven different boxes namely P, Q, R, S, T, U, V having
not sufficient to answer the question. different no. of articles in it placed one above the other. No.
Of articles is 2, 6, 9, 11, 15, 16, 20. The following
(e) If both the statements I and II taken together are
information is given below.
sufficient to answer the question. There are two boxes lies between box S and the box in
10. There are six persons P, Q, R, S, T and U sitting around which there is 11 articles. Two boxes are placed between
box P and the box in which there is 9 articles. There are
a circular table. Who is on the immediate right of T?
only three boxes lies between box T and the box which
I. Only U is sitting between T and P. have 9 articles . Box S is placed below box P. Box U is
II. P is third to the right of S. immediately below the box which has 15 articles. Box P
11. What is the code for 'book' in a code language? doesn't have 15 articles in it. Only one box is there between
box Q and box V. There are two boxes between the boxes
I. In that language `pik tik rik' means 'I like reading'
having 15 and 6 articles. Box S lies above the box having 11
and 'mik nek bek fek' means 'this book is articles. The difference between no of articles in the box S
interesting'. and the box which lies just above box S is greater than 10.
II. In the same language 'pik tik mik rik' means 'I like The number of articles in the box which lies just above box
reading book' and ‘mik juk pan’ means 'you read V is less than the no. of articles in V . Box Q does not have 6
book'. number of articles. Box U does not have less than 10
articles in it.
12. Who is Kavita's sister?
16. How many boxes is/are there between S and U?
I. Sapna is the granddaughter of Amit, who is the
(a) One (b) Two (c) Three
father of Kavita's father. (d) Four (e) None
II. The name of Kavita's sister starts with the letter 'S'.
17. How many articles are there in the box ‘S’?
13. Which of the following month does Pooja birthday? (a) 11 (b) 15 (c) 20
I. Her brother remembers that her birthday is after (d) 16 (e) Can’t be determined.
March but before October. 18. Find the pair of articles and boxes which is not correct?
II. Her mother remembers that her birthday is after (a) Q-16 (b) U-11 (c) P-2
May but before September. And birthday on that (d) V-6 (e) None of these
month which has more than 30 days. 19. Which of the following condition is correct regarding
14. What is the rank of Sohan in a class of 12 students? the box which contains 2 articles in it with respect to
I. Mahesh is fourth from the top. Sagar is fourth rank T?
(a) There are only one box between the box T and the
below Mahesh. Sohan is either immediate above or
box which contains 20 articles.
immediate below Sagar. (b) Box T is immediately above the box which contains
II. Riti is second from the top and Harish is sixth from 2 articles.
the bottom. Meena is immediate below of Riti. (c) Box which contains 2 articles is immediately above
Sagar is fifth rank below of Meena. box T.
(d) All of the above is true
15. Most of the private companies have decided against (e) None of the above is true.
awarding annual increase in the salaries of their
20. Box P contains how many articles?
employees for the previous year due to current
(a) 15 (b) 16 (c) 6
economic situations. (d) 11 (e) None of these.
Which of the following assumptions is/are implicit in
the statement? Direction (21-25): In each of the questions below are
given four statements followed by three conclusions
(I) Majority of the employees may leave their job to
numbered I & II. You have to take the given statements to
protest against the decision. be true even if they seem to be at variance with commonly
(II) These companies may announce hike in salaries known facts. Read all the conclusions and then decide
next year. which of the given conclusions logically follows from the
(a) Only I (b) Only II given statements disregarding commonly known facts.
(c) Both I and II (d) Either I or II (a) If only conclusion II is true
(e) Neither I nor II (b) If only conclusion I is true

3 Adda247 Publications For any detail, mail us at


Publications@adda247.com
50+ Bank PO | Clerk Previous Year’s Papers 2016 – 2020

(c) If both conclusion I and II are true III. Waste water recycling plant is also being planned
(d) If either conclusion I or II is true in which recycled water will be used for washing
(e) If neither conclusion I nor II is true and horticulture purpose.
IV. Initiatives such as LED light fittings to reduce
21. Statements: Some roses are jasmines.
power consumption and use of five percent bio-
Some jasmines are lilies.
diesel in diesel locos.
All lilies are marigolds.
V. Introduction of water audit to assess consumption
All marigolds are sunflowers.
and wastage at major stations.
Conclusions: I. All lilies are sunflowers.
(a) All of these (b) All except I
II. Some jasmines are marigolds.
(c) All except I and V (d) All except V
22. Statements: Some flats are houses. (e) None of these
Some houses are bungalows.
28. Statement: The government is set to overhaul annual
No bungalow is hotel. targets for public sector lenders, ending a focus on size
All hotels are restaurants. that has long encouraged banks to inflate their loans
Conclusions: I. No bungalow is restaurant. and deposits at the year-end to hit growth objectives.
II. Some houses are hotels. Banking and government sources said that the new
23. Statements: All papayas are fruit. targets, to be discussed at meeting with top state bank
No fruit are orange. officials, would focus on efficiency, with objectives set
Some orange are honey around return on assets, or return on equity, and
Conclusions: I. Some honey are not fruit. controlling bad debts.
II. No papaya are orange. Which of the following points can be inferred from the
given information? (An inference is something that is
24. Statements: Some flats are houses. not directly stated but can be inferred from the given
Some houses are bungalows. information.)
No bungalow is hotel. (a) Loans and deposits of the banks will go up.
All hotels are restaurants. (b) Some short-term loans will be discouraged by the
Conclusions: I. Some houses are not hotels. banks.
II. Some houses are not restaurants. (c) Credit ratings of the banks will be boosted up.
25. Statements: Some roses are jasmines. (d) Financial stability of the banks will be lost.
Some jasmines are lilies. (e) All can be inferred.
All lilies are marigolds.
All marigolds are sunflowers. 29. From word ‘OUTSHINE’ with the help of 1st, 2nd, 3rd ,
Conclusions: I. Some sunflowers are jasmines. 5th and 7th letter make meaningful English word and
II. No marigolds are roses. answer that what is the third letter in a new word. If
more than one word is possible then give answer Z and
26. How Many such pairs of letters are there in the word if no word is possible then give answer X.
FUNCTION, each of which have as many letters (a) T (b) X (c) U
between them in the word as they have between them (d) Z (e) N
in the English alphabet?
(a) None (b) One (c) Two 30. Out of five persons A, O, M, E and P. M is younger than
(d) Three (e) More than three A and O but older than E. A is older than O and E. P is
not the youngest. Then which person is the Youngest?
27. Statement: Reinforcing its commitment to green (a) O (b) E (c) P
initiatives, railways will explore its future course of (d) A (e) None of these
action to prevent pollution. Issues related to
environment like solid waste management, pollution Directions (31-35): Study the following information
control, water management and energy management carefully to answer the given questions
will be discussed at a workshop on ‘Environmental Seven persons A, B, C, D, E, F and G are belongs to three
challenges before Indian Railways and solutions’. different departments Production, Marketing and HR with
Which of the following points will strengthen the at least 2 of them in any of these department. Each of them
decision taken by the Railways? belongs to different cities i.e. Patna, Chandigarh, Kolkata,
I. Introduction of bio-fuel and solar energy in train Delhi, Jaipur, Ranchi and, Mumbai.
operation. B works in Marketing department with E. E belongs to
II. Installation of rain water harvesting facilities at Mumbai. Those who work in Production department do
major stations. not belong to Patna and Delhi. The one who belongs to

4 Adda247 Publications For any detail, mail us at


Publications@adda247.com
50+ Bank PO | Clerk Previous Year’s Papers 2016 – 2020

Chandigarh works only with G in department HR. The one Car A is in odd number ranking and is not 3rd lowest in
who belongs to Jaipur does not work in the same covering distance. Car which is of yellow colour is
department with either E or G. A does not works in immediate more than in covering distance than Car A.
Marketing department. A belongs to Ranchi. D and F are There are only two cars between car A and the one which
work in the same department. F does not belong to Kolkata. is of green colour in covering distance. The Car which is of
The one who belongs to Delhi does not work in Marketing orange colour is in odd number ranking but greater than
department. Car D in travelling. D is covering third lowest distance.
Third lowest car in travelling covers a distance of 1218 km.
31. In which department A, D and F work? Only three cars are between car C and the one which is of
(a) Cannot be determines (b) Production orange colour. The one which is of green colour is
(c) Marketing (d) HR immediate more than in covering distance than car C. E
(e) None of these covers a distance of 1456 km. The car which is of red colour
is immediate more than in covering distance than car
32. Who belongs to Patna?
which is of blue colour. Car G is in odd number ranking.
(a) B (b) A (c) C Only one car is in between car B and E. Car B covers more
(d) D (e) None of these distance than car E. Neither car C nor A is of black colour.
33. In which of the following city does ‘F’ belong? Car E is not of green colour.
(a) Ranchi (b) Delhi (c) Mumbai 36. Car A is of which colour?
(d) Jaipur (e) None of these (a) White (b) Blue (c) Green
34. Which of the following combination is right? (d) Orange (e) Red
(a) A – HR : Chandigarh 37. Which of the following combinations is true with
(b) G – Production: Jaipur respect to the given arrangement?
(c) F – HR : Delhi (a) White–C (b) Orange – F (c) Blue - G
(d) D – Production: Kolkata (d) Yellow - D (e) Black–B
(e) None of these 38. Which car is second highest in covering distance?
35. C works in which department? (a) B (b) Car which is of red colour
(a) Production (b) Marketing (c) Car which is of blue colour
(c) HR (d) Cannot be determined (d) D (e) F
(e) None of these 39. If B+E cover a distance of 3216km, then what is the
Directions (36-40): Study the given information carefully possible score of A?
(a) More than G. (b) 1520 km
to answer the given question.
(c) 1368 km (d) Less than 1456km
A, B, C, D, E, F and G are seven cars which are running on
(e) None of these
tracks and travelling and covering different distance but
not necessarily in the same order. Each car has different 40. What is the colour of Car F?
colours i.e. Red, Orange, Green, Black, Yellow, White and (a) White (b) Blue (c) Green
Blue but not necessarily in the same order. (d) Orange (e) Red

QUANTITATIVE APTITUDE

41. In a mixture of 40 litre, the ratio of milk and water is 4 (a) 69 years (b) 70 years (c) 73 years
: 1. If some quantity of mixture taken out and then 4 (d) 77 years (e) None of these
litre of milk and 4 litre of water is added to the mixture
then the ratio of milk and water become 8 : 3. Find the 43. A boat takes 6 min more to cover 21 km downstream
quantity of mixture which was taken out initially ? when the river flowing at the speed of 𝑥 km/hr then
(a) 10 litre (b) 15 litre (c) 12 litre the time taken to cover the same distance downstream
(d) 18 litre (e) None of these when the river flowing at the rate of (𝑥 + 1) km/hr.
42. If the present age of A is twice the present age of B. 18 find the value of 𝑥 if the speed of boat in still water is
years ago age of C is half the age of B six years hence. If 12 km/hr ?
the average of the present age of A, B and C is 42 years. (a) 1.5 (b) 2 (c) 2.5
Find the age of A, 9 years hence? (d) 1 (e) 3
5 Adda247 Publications For any detail, mail us at
Publications@adda247.com
50+ Bank PO | Clerk Previous Year’s Papers 2016 – 2020

44. There are 36 tickets numbered from 1 to 36. If two same month. Then find the total no. of books published
tickets are drawn at random without replacement one by company B in January.
by one than find the probability that both tickets have (a)625 (b) 650 (c) 600
a number which is multiple of 5 ? (d) 700 (e) None of these
1 1 1
(a) (b) (c) 49. If no. of books distributed by company B in November
40 48 28
1 2
(d) (e) None of these are14 7 % less than book distributed by same company
30
in September, while total books published in
45. A certain sum was put on simple interest at 20% per
November are 400, then what percent of books
annum for 2 years. Then the amount received after 2
published in November by company B are distributed?
years put on compound interest 12% per annum for 2
(a)25.5% (b) 26.5% (c) 27.5%
more years. If the amount received at the end of 4 years
(d) data inadequate (e) None of these
is 43904 Rs. Then find the amount invested at starting?
(a) 20,000 Rs. (b) 25,000 Rs. 50. Find the total no. of books distributed in March by both
(c) 30,000 Rs. (d) 40,000 Rs. companies together?
(e) None of these (a) 157 (b) 187 (c) 167
(d) 177 (e) None of these
Directions (46 – 50): The following table shows the no. of
books published and percentage of book distributed Directions (51-55): In each of these questions a number
among them for 2 companies A and B in different months. series is given. In each series only one number is wrong.
Find out the wrong number.
51. 4, 10, 23, 43, 70, 108, 145
(a) 10 (b) 145 (c) 43
(d) 70 (e) 108
52. 8, 5, 7, 10, 21, 53.5, 161.50
46. If total no. of books published in May by both (a) 5 (b) 10 (c) 7
companies are 1625, while total books distributed by (d) 21 (e) 53.5
both are 766, then find difference between no. of books 53. 5, 8, 13, 40, 161, 806, 4837
published by company A and Company B in may. (a) 161 (b) 40 (c) 13
(a)250 (b) 275 (c) 225 (d) 8 (e) 806
(d) 300 (e) None of these
54. 6, 14, 78, 294, 842, 1806, 3534
47. The no. of Books distributed by company A in January (a) 294 (b) 842 (c) 78
are approximately what percent of no. of books (d) 14 (e) 3534
distributed by company B in July?
(a)65% (b) 70% (c) 75% 55. 3, 4, 6, 24, 192, 3072
(d) 73% (e) 80% (a) 4 (b) 6 (c) 192
(d) 24 (e) 3072
48. If book distributed by company B in January are 2 less
than double of books distributed by company A in the
Directions (56-60): Given below is the pie chart showing the distribution of no. of students of a school who are involved
in 5 different games. The table shows the ratio of boys to girls among them. Study them carefully and answer the following
questions.
Total no. of students = 1800

Swimming
Tennis 12%
25%
Cricket
13%

Hockey Football
30% 20%

6 Adda247 Publications For any detail, mail us at


Publications@adda247.com
50+ Bank PO | Clerk Previous Year’s Papers 2016 – 2020

Game Boys : Girls


Football 5:4
Hockey 7:3
Tennis 2:3
Swimming 3:5
Cricket 2:1

56. The number of girls in swimming are what percent of (a) 50 : 81 (b) 45 : 79 (c) 40 : 83
the number of girls in Tennis ? (d) 40 : 81 (e) None of these
(a) 45% (b) 60% (c) 65% 59. Total no. of girls in all 5 games together is
(d) 50% (e) None of these approximately what percent of the total number of
students ?
57. What is the sum of the number of boys in cricket and (a) 48% (b) 40% (c) 45%
number of boys in Hockey ? (d) 35% (e) 55%
(a) 534 (b) 532 (c) 525
60. Find the different between number of boys in
(d) 540 (e) None of these Swimming to that in Football.
58. What is the ratio of number of boys in football to (a) 144 (b) 124 (c) 114
(d) 122 (e) None of these
number of girls in Tennis and swimming together?

Directions (61-65): The first line graph shows the total distance travelled by 5 different trains on Monday, while the
second graph shows the corresponding time taken to cover that distance.
6 500 405
5 400 285 315
4 300 245
3 180
200
2
100
1
0
0 A B C D E
A B C D E
Distance travelled (in km)
Time taken (in hrs)

61. If the average of the speed of train A on Monday and (a) 62% (b) 64% (c) 68%
Tuesday is 75 km/hr, while the time of travelling is (d) 66% (e) 70%
same on both the days, then find the total distance 1
travelled by train A on Tuesday. 65. If distance travelled by train D is increased by 11 9 %
(a) 290 km (b) 240 km (c) 280 km on Tuesday, while the time of travelling of train D on
(d) 250 km (e) None of these Tuesday is 4 hrs, then find the speed of train D on
62. What is the ratio of speed of train B to speed of train D Tuesday.
on Monday ? (a) 72 km/hr (b) 90 km/hr
(a) 4 : 5 (b) 3 : 4 (c) 2 : 3 (c) 112.5 km/hr (d) 100 km/hr
(d) 3 : 5 (e) None of these (e) None of these
63. The speed of the slowest train is approximately what
percent of the speed of fastest train on monday? 66. A man bought a mobile and a laptop for Rs. 78000. He
(a) 48% (b) 54% (c) 60% sold the mobile at a gain of 25% and the laptop at a loss
(d) 64% (e) 70% of 15%, thereby gaining 5% on the whole. Find the cost
price of mobile.
64. If the speed of train B is decreased by 25% on Tuesday,
and of train A increases by 20% on Tuesday. Then (a) Rs. 39000 (b) Rs. 34000
speed of train B on Tuesday approximately what (c) Rs. 30000 (d) Rs. 38000
percent of speed of train A on Tuesday ? (e) Rs. 32000

7 Adda247 Publications For any detail, mail us at


Publications@adda247.com
50+ Bank PO | Clerk Previous Year’s Papers 2016 – 2020

67. A and B started a business with the investments in the 73. The different between the simple and the compound
ratio of 5 : 3 respectively. After 6 months from the start interest compounded every six months at the rate of 10
of the business, C joined them and the respective ratio per cent per annum at the end of two years is Rs.
between the investments of B and C was 2 : 3. If the 124.05. What is the sum?
annual profit earned by them was Rs. 12300, what was (a) Rs. 10000 (b) Rs. 60000
the difference between B’s share and C’s share in the (c) Rs. 12000 (d) Rs. 8000
profit ? (e) None of these
(a) Rs. 900 (b) Rs. 800 (c) Rs. 600 74. The ratio of the quantities of sugar, in which sugar
(d) Rs. 400 (e) Rs. 700 costing Rs. 20 per kg. and Rs. 15 per kg. should be
mixed so that here will be neither loss nor gain on
68. A group of 30 men, working 4 hours a day can do a
selling the mixed sugar at the rate of Rs. 16 per kg.
piece of work in 10 days. Find the number of days in
(a) 2 : 1 (b) 1 : 2 (c) 4 : 1
which another group of 45 men working 8 hrs a day (d) 1 : 4 (e) None of these
can do twice the work. Assume that 2 men of the first
group do as much work in 2 hours as 4 men of the 75. If the numerator of a fraction is increased by 20% and
second group do in 1 hr. the denominator is increased by 25%, the fraction
3
1
(a) 6 3 days
2
(b) 6 3 days
5
(c) 5 6 days obtained is 5. What was the original fraction?
5 4 3
1 (a) (b) (c)
(d) 3 days (e) None of these 7 7 8
6
(d) Can't be determined (e) None of these
69. Manish borrowed some money at the rate of 7 per cent
Directions (76-80): Equation number I and II are given.
per annum for the first three years, 9 per cent per
You have to solve both the questions and answer.
annum for the next six years and 10 per cent per
annum for the period beyond nine years. If the total 76. I. 3𝑥 2 + 29𝑥 + 56 = 0
interest paid by him at the end of fifteen years is Rs II. 2𝑦 2 + 15𝑦 + 25 = 0
4050, how much money did he borrow?( if simple (a) x < y (b) x > y (c) x ≥ y
interest be reckoned) (d) x ≤ y
(a) Rs 2800 (b) Rs 3600 (c) Rs 3000 (e) Relationship between x and y cannot be established
(d) Rs 3500 (e) None of these 77. I. 4𝑥 2 − 29𝑥 + 45 = 0
II. 3𝑦 2 − 19𝑦 + 28 = 0
70. The average marks of 100 students were found to be
(a) x < y (b) x > y (c) x ≥ y
40. Later on it was discovered that a score of 53 was
(d) x ≤ y
misread as 83. Found the correct average (e) Relationship between x and y cannot be established
corresponding to the correct score.
(a) 38.7 (b) 39 (c) 39.7 78. I. 3𝑥 2 − 13𝑥 + 12 = 0
(d) 41 (e) None of these II. 3𝑦 2 − 7𝑦 + 2 = 0
(a) 𝑥 > 𝑦 (b) 𝑥 < 𝑦 (c) 𝑥 ≥ 𝑦
71. A circus tent is cylindrical up to a height of 3 m and (d) Relationship between 𝑥 and 𝑦 cannot be established
conical above it. If its diameter is 105m and the slant (e) 𝑥 ≤ 𝑦
height of the conical part is 63 m, then the total area of
2 79. I. 20𝑥 2 − 9𝑥 + 1 = 0
the canvas required to make the tent is (take 𝜋 = ) II. 12𝑦 2 − 7𝑦 + 1 = 0
7
(a) 11385 m2 (b) 10395 m2 (c) 9900 m2 (a) 𝑥 ≥ 𝑦 (b) 𝑥 ≤ 𝑦 (c) 𝑥 > 𝑦
(d) 990 m2 (e) None of these (d) Relationship between 𝑥 and 𝑦 cannot be established
2
72. A man buys pulse for Rs. 4800. He sells th of it at a (e) 𝑥 < 𝑦
5
profit of 25%. At what per cent gain should he sell
3
80. I. 𝑥 2 = 16
remaining 5th so as to make an overall profit of 19% on II. 2𝑦 2 − 17𝑦 + 36 = 0
the whole transaction? (a) x > y (b) x ≥ y (c) x < y
(a) 15% (b) 18% (c) 21% (d) Relationship between x and y cannot be established
(d) 12% (e) none of these (e) x ≤ y

8 Adda247 Publications For any detail, mail us at


Publications@adda247.com
50+ Bank PO | Clerk Previous Year’s Papers 2016 – 2020

ENGLISH LANGUAGE

Directions (81-90): Read the following passage carefully the GSLV, the Mk-III can carry three astronauts and have
and answer the questions given below it. Certain words are more space to carry out experiments. The next
given in bold to help you locate them while answering developmental flight, therefore, will be crucial.
some of the questions.
81. Which of the following features make the newly
The Indian Space Research Organisation has crossed a launched vehicle GSLV Mark-III distinctive?
significant milestone with the successful developmental (I) It is the first time that India has launched a
flight of the country’s heaviest Geosynchronous Satellite satellite weighing over 3.1 tonnes.
Launch Vehicle, the GSLV Mark-III. This is the first time a
(II) The launching capacity of satellites up to four
satellite weighing over 3.1 tonnes has been launched from
India to reach the geostationary orbit about 36,000 km tones almost doubles the current launch capacity
from Earth. The Mk-III can launch satellites weighing up to of India.
four tonnes, which almost doubles India’s current launch (III) It is the first time that an indigenously developed
capacity. With communication satellites becoming heavier lithium-ion battery was used to power the
(up to six tonnes), the capability for larger payloads is vital. satellite.
This can be done by switching over to electric propulsion (a) Only (I) is correct
for orbit rising and to keep the satellite in the right position
(b) Only (II) is correct
and orientation in the orbit through its lifetime (that is,
station keeping). The switch-over would reduce the weight (c) Both (I) and (II) are correct
of the vehicle as it can do away with nearly two tonnes of (d) Both (II) and (III) are correct
propellants and carry heavier satellites. Towards this end, (e) All are correct
ISRO has started testing electric propulsion in a small way;
82. How, according to the passage, excess load of newly
the South Asia Satellite (GSAT-9) that was launched last
month used electric propulsion for station keeping. On launched satellite vehicle can be mitigated?
Monday, an indigenously developed lithium-ion battery (a) By switching over to electric propulsion for orbit-
was used for the first time to power the satellite. Another rising and station keeping.
key achievement is the use of an indigenously developed (b) By replacing the heavier equipments by lighter
cryogenic stage, which uses liquid oxygen and liquid ones.
hydrogen; the 2010 GSLV launch using an indigenous (c) By using the cryogenic stage which uses liquid
cryogenic stage ended in failure. It can now be said without
oxygen and liquid hydrogen.
hesitation that India belongs to the elite club of countries
that have mastered cryogenic technology. In the December (d) By proper studies of aerodynamic behaviour of the
2014 experimental flight of the GSLV Mk-III, a passive vehicle.
cryogenic stage was used. Though the cryogenic stage was (e) Both (a) and (b)
not meant to be ignited, the launch provided invaluable
data on aerodynamic behaviour of the vehicle. 83. Which of the following statements justify the author’s
view, “This will make India self-reliant in launching
The Mark-III will be operational with the success of one heavier satellites”?
more developmental flight, which is set to take place within
(I) The successful launch of the country’s heaviest
a year. This will make India self-reliant in launching
heavier satellites, bringing down costs substantially. Till Geosynchronous Satellite Launch Vehicle, the
now, heavier communication satellites have been launched GSLV Mark-III is one of the greatest milestones in
on Europe’s Ariane rockets; in fact, ISRO will soon be using the history of ISRO.
Ariane rockets to launch two of its heavier satellites. But as (II) ISRO is set to launch yet another developmental
has been the case with lighter satellites, it is likely that flight within a year.
other countries will soon turn to ISRO for the launch of (III) India now belongs to the elite club of countries
heavier satellites at a lower cost. With fewer propulsion
that have mastered cryogenic technology.
stages and, therefore, control systems, the Mk-III is far
more reliable than the GSLV and the PSLV. Combined with (a) Only (I) is correct
its ability to carry eight to 10 tonnes into a low Earth orbit, (b) Both (I) and (II) are correct
the Mk-III can be considered for human-rating certification (c) Both (II) and (III) are correct
(to transport humans) once some design changes are (d) Both (I) and (III) are correct
made. Compared with the two-member crew capacity of (e) All are correct

9 Adda247 Publications For any detail, mail us at


Publications@adda247.com
50+ Bank PO | Clerk Previous Year’s Papers 2016 – 2020

84. Which of the following statements is false in context of Directions (91-95): Read the passage carefully and
the passage? answer the questions given below it. Certain words/
(a) The South Asia Satellite (GSAT-9) used electric phrases have been given in bold to help you locate them
propulsion for station keeping. while answering some of the questions.
(b) In the December 2014 experimental flight of the Many people believe that Science and Religion are contrary
GSLV Mk-III, a passive cryogenic stage was used. to each other. But this notation is wrong. As a matter of fact,
(c) ISRO will soon be using Ariane rockets to launch both are complementary to each other. The aim of both
two of its heavier satellites. these institutions is to explain different aspects of life,
(d) The switch-over to electric propulsion would universe and human existence. There is no doubt that the
method of science and religion are different. The method of
reduce nearly two tonnes of propellants and carry
science is observation, experimentation and experience.
heavier satellites.
Science takes its recourse to progressive march towards
(e) All are true.
perfection. The tools of religion are faith, institution and
85. How according to the passage Mk-III is more reliable spoken word of the enlightened. In general, while science
than the GSLV and the PSLV? is inclined towards reason and rationality, spiritualism is
(I) The Mk-III can carry three astronauts as the essence of religion.
compared to GSLV which has two-member crew In earlier times when man appeared on earth he was
capacity. overawed at the sight of violence of powerful aspects of
(II) The Mk-III has fewer propulsion stages and nature. In certain cases, usefulness of different natural
control systems than GSLV and PSLV. objects of nature overwhelmed man. Thus, began the
(III) The Mk-III has more space to carry out worship of forces of nature fire, the sun, the rivers, the
experiments than GSLV and PSLV. rocks, the trees, the snakes, etc. The holy scriptures were
(IV) The Mk-III costs lighter than GSLV and PSLV in written by those who had developed harmony between
terms total production cost. external nature and their inner self. Their objective was to
(a) Both (I) and (II) ennoble, elevate and liberate the human spirit and mind.
(b) Both (II) and (III) But the priestly class took upon itself the monopoly of
(c) Only (I), (II) and (IV) scriptural knowledge and interpretation to its own
(d) Only (I), (II) and (III) advantage. Thus, whole entire human race was in chains.
(e) All (I), (II), (III) and (IV) Truth was flouted and progressive liberal and truthful
ideas, ideas expressing doubt and scepticism were
Directions (86-88): Choose the word/group of words suppressed and their holders punished. It was in these
which is most similar in meaning to the word/group of trying circumstances that the science emerged as a saviour
words printed in bold as used in passage. of mankind. But its path was not smooth and safe. The
scientists and free thinkers were tortured. This was the
86. Propulsion fate of Galileo, Copernicus, Bruno and others. But, by and
(a) Plethora (b) Impulse (c) Satiety by Science gained ground.
(d) Clogging (e) Revulsion
91. Why does man worship the forces of nature?
87. Crew (a) The holy scriptures advocate the worship of forces
(a) Interior (b) Crowd (c) Oodles of nature.
(d) Bevy (e) Laity (b) The worship elevates and liberates the human
88. Propellant spirit and mind.
(c) The worship makes man believe in faith and
(a) Mulligan (b) Antecedent (c) Mainspring
intuition.
(d) Motif (e) Ardour
(d) Forces of nature teach us spiritualism.
Directions (89-90): Choose the word/group of words (e) None of these
which is most opposite in meaning to the word/group of 92. Which of the following statement is TRUE in the
words printed in bold as used in passage. context of the passage?
89. Elite (a) Science and religion are antagonistic to each other.
(a) Patriciate (b) Gentry (c) Dregs (b) Science encourages worshipping of the nature.
(d) Skim (e) Unusual (c) Religion is essential for external peace and
harmony.
90. Operational (d) Regimental religion was replaced by scientific
(a) Viable (b) Realizable (c) Virtuous principles.
(d) Expedient (e) Severed (e) Science is essential for inner peace of mind.

10 Adda247 Publications For any detail, mail us at


Publications@adda247.com
50+ Bank PO | Clerk Previous Year’s Papers 2016 – 2020

93. According to the passage, science and religion both Directions (101 – 105): In each of the following questions
(a) rely on the spoken word of the enlightened given below there are two blanks, each blank indicates that
(b) emerged out of the fear of man something has been omitted. Choose the set of words for
(c) emerged from the desire of man to worship the each blank which best fits to the meaning of the sentence as
forces of nature
a whole.
(d) employ different methods of enquiry
(e) work at the cross-purpose of each other
101. SEBI’s predominant concern, apart from ____________ the
94. Which of the following statements is NOT TRUE in the information available to investors, seems to be to
context of the passage? ____________ rating agencies from resorting to collusion
(a) Man worships the forces of nature.
in reaching decisions.
(b) Methods of science and religion are different.
(a) regulating, delaying
(c) Regimental religion got degenerated into
orthodoxy. (b) overhauling, circumventing
(d) Galileo and Bruno were disciples of Copernicus. (c) magnifying, deter
(e) The holy scriptures were written by people who (d) improving, prevent
had tremendous inner strength. (e) retrogressing, retard
95. According to the passage, at the present juncture, there 102. Repeated failures have not ______________ the business of
is a need to rating agencies, primarily due to the lack of alternative
(a) encourage spiritualism as much as possible service providers who can _______________ out investors.
(b) teach people to worship the forces of nature
(a) concerned, serve (b) assumed, gratify
(c) free man from all sorts of bondages
(c) pretended, favour (d) affected, help
(d) explain to the people different aspects of life and
universe (e) perturbed, pamper
(e) judiciously mix the principles of science and the 103. The legislature of Spain’s north-eastern region last
spirit of religion
week passed a law to back the October 1 vote for
Directions (96-100): Read each sentence to find out ___________, consistent with the narrow electoral
whether there is any grammatical or idiomatic error in it. mandate it ____________ on a promise of self-
The error, if any, will be in one part of the sentence. The determination.
number of that part is the answer. If there is ‘No error’, the (a) rejection, disbursed
answer is (e). (Ignore errors of punctuation, if any.)
(b) secession, received
96. Festivals are prime occasions (a)/ for spluring on (c) repudiation, credited
presents and owing to improved economic situation (d) abrogation, sanctioned
(b)/ the youths is gung-ho (c)/ about breaking all (e) evasion, recognized
previous records. (d)/ no error. (e)
104. Global commodities have ___________ a bull run over the
97. It is important to recruit personnel at (a)/ different
last 12 months, ____________hopes of the beginning of the
levels in the organization so that (b)/ ensuing human
resource gap is bridged (c)/ at least for the critical next supercycle in commodity prices.
operations. (d)/ No error. (e) (a) insisted, provoking (b) professed, agitating
(c) witnessed, reviving (d) noticed, dissuading
98. Not only has the commerce ministry fixed (a)/
(e) understood, rectifying
extraordinarily minimum prices for onion exports (b)/
but it has also made licencesmandatory (c)/ for every 105. After 39 __________ successful launches, the Indian Space
consignment. (d)/ No error. (e) Research Organisation had almost made it ______________
99. Mother tongue is as natural (a)/ for the development that launching satellites was indeed child’s play when
of man’s mind (b)/ as mother’s milk is (c)/ for the it used its workhorse rocket, the Polar Satellite Launch
development of the infant’s body. (d)/ No error. (e) Vehicle.
100. A small piece (a)/ of bread is (b)/ certainly better than (a) pertinent, perform (b) infrequent, operate
(c)/ being nothing to eat. (d)/ No error. (e) (c) recurrent, exert (d) coherent, conduct
(e) consecutive, appear
11 Adda247 Publications For any detail, mail us at
Publications@adda247.com
50+ Bank PO | Clerk Previous Year’s Papers 2016 – 2020

Directions (106-110): Rearrange the following sentences small and affordable household biogas plants is simply
(A), (B), (C), (D), (E) and (F) in the proper sequence to form being thrown away. It is also ironic that while some
a meaningful paragraph; then answer the questions given countries such as Rwanda and Kenya have introduced stiff
penalties for the __(117)___ of flimsy plastic bags, India is
below them.
doing little to __(118)__ them from drifting into suburban
(A) The Australian this week became the first player ever garbage mountains, rivers, lakes and the sea, and being
to be penalized for mock fielding, in a local match. ___(119)___ by cattle feeding on dumped ___(120)___.
(B) The batsman was confused for a moment, stopped in
his tracks, and then completed the run. 111. (a) sparsity (b) desertion (c) poverty
(C) The umpire awarded the batsman five runs for (d) absence (e) paucity
“deliberate distraction, deception or obstruction of a 112. (a) settled (b) equipped
batsman”. (c) surrounded (d) confined
(D) Connoisseurs of cricketing trivia would do well to (e) embedded
remember the name MarnusLabuschagne.
(E) This trick is a common one in Indian club cricket, 113. (a) supplies (b) forms (c) classifies
especially when there is overgrown grass in the (d) demands (e) extends
monsoon. 114. (a) manufacture (b) accumulation
(F) Labuschagne dived to stop a cover drive that went past (c) generation (d) foundation
him, quickly got up and pretended to throw the ball (e) conception
that he did not actually have in his hands.
115. (a) awed (b) misled (c) lost
106. Which of the following should be the first sentence (d) absorbed (e) rapt
after rearrangement?
(a) B (b) D (c) A 116. (a) feed (b) deliver (c) nurture
(d) F (e) C (d) provide (e) produce

107. Which of the following should be the third sentence 117. (a) interest (b) favor (c) gain
after rearrangement? (d) selection (e) use
(a) A (b) D (c) F 118. (a) reduce (b) discourage (c) prevent
(d) E (e) C (d) facilitate (e) reject
108. Which of the following should be the fourth sentence 119. (a) absorbed (b) grazed (c) ravaged
after rearrangement? (d) ingested (e) grasped
(a) B (b) E (c) F
(d) C (e) D 120. (a) limit (b) refuse (c) exclude
(d) evade (e) cease
109. Which of the following should be the last sentence
after rearrangement?
(a) D (b) E (c) A
(d) B (e) C
110. Which of the following should be the second sentence
after rearrangement?
(a) C (b) A (c) B
(d) D (e) E
Directions (111-120): In the following passage, some of
the words have been left out, each of which is indicated by a
number. Find the suitable word from the options given
against each number and fill up the blanks with appropriate
words to make the paragraph meaningfully complete.
In the __(111)___ of stakeholders at the local body level,
recoverable resources __(112)__ in discarded materials are
lost due to dumping. Organic refuse, which __(113)__ about
50% of all garbage, readily lends itself to the ___(114)__ of
compost or production of methane for household use or
power generation. But it is a major opportunity __(115)___.
Organic waste that could help green cities and __(116)__

12 Adda247 Publications For any detail, mail us at


Publications@adda247.com
50+ Bank PO | Clerk Previous Year’s Papers 2016 – 2020

Solutions

REASONING ABILITY

Directions (1-5): 11. (b); From Statement II- it is clear that the code for ‘Book’
is “mik”.
12. (d); From Statement I and II- We can clearly say that there
is two possible case that either Sapna is Kavita’s
sister or Sapna is Kavita’s cousin sister.

1. (b); 2. (e); 3. (a);


4. (b); 5. (b);
6. (b);
13. (d); From Statement I, we can say that Pooja’s birthday is
either in the month of July or in August.
14. (e); From Statement I and II- we can clearly say that
Sohan’s rank is fourth from the bottom or ninth from
the top.
7. (d);

15. (e); Since, nothing is being mentioned in the paragraph


about the employees leaving their jobs or company
announcing pay hikes next year. SO, neither of the
two statements are implicit.
8. (c);
Direction (16-20):
Boxes Articles
T 16
P 2
V 6
S 20
Q 9
9. (a); All that the speaker implies is that Kapil is efficient
R 15
because he has twelve years of practice, and so option
U 11
(a) is correct. (b) is eliminated because the word
‘identical’ is not mentioned in the paragraph. (c) is 16. (b); 17. (c); 18. (a);
eliminated because we are talking about typing not 19. (b); 20. (e);
21. (c);
the whole ‘job profile’. (d) is eliminated as nothing is
mentioned about Kapil’s speed and accuracy and we
cannot assume efficiency as speed and accuracy.
Similarly (e) cannot be the answer as nothing is
implied about fitting into the office.

10. (d); From Statement I and II- We cannot say that who
22. (e);
among the following sits on the immediate right of T
as there are two possibilities that either S or U sits on
the immediate right of T.
23. (c);

13 Adda247 Publications For any detail, mail us at


Publications@adda247.com
50+ Bank PO | Clerk Previous Year’s Papers 2016 – 2020

24. (b); year) to finance your temporary working capital


needs.

29. (b); The 1st, 2nd, 3rd , 5th and 7th letter of the word
‘OUTSHINE’ are O, U, T, H and N. Using these letter no
25. (b);
meaningful word can be possible.

30. (b); From the above given statements, we can clearly say
the E is the youngest among all.

Directions (31-35):

Persons Department City


26. (c); There are only two letters in the given word in which
have as many letters between them in the word as A Production Ranchi
they have between them in the English alphabet. B Marketing Patna
C HR Chandigarh
D Production Kolkata
E Marketing Mumbai
27. (d); All the points in the options promote green initiatives F Production Jaipur
except auditing of water consumption as it is an
G HR Delhi
examination of system records and equipment that
may be used to identify, quantify and verify how 31. (b); 32. (a); 33. (d);
much water passes through the system and where it 34. (d); 35. (c);
goes.
28. (b); ‘Short-term loans’ refer to the loans scheduled to be Directions (36-40):
repaid in less than a year. When your business does
not qualify for a line of credit from a bank, you might
36. (d); 37. (a); 38. (a);
still have success in obtaining money from them in
the form of a one-time, short-term loan (less than a 39. (b); 40. (c);

QUANTITATIVE APTITUDE

4 21 21 6
41. (b); Milk → × 40 = 32 litre 43. (b); − =
5 x+12 x+13 60
2
1
Water → × 40 = 8 litre x + 25 − 54 = 0
5 x = −27, +2
Let x liters mixture take out Required speed = 2 km/hours
4
32− ×x+4 8
∴ 1
5
= 44. (d); ∴ Required probability =
7
×
6
=
1
8− ×x+4 3
5 36 35 30
540 − 12x = 480 − 8x 45. (b); Let amount = 1000 Rs.
4x = 60 After 2 years, amount = 1.4 × 1000
x = 15 litres. = 1400 Rs.
Further after two years, amount
42. (a); A → 2x years 12 2
B → x years = 1400 (1 + ) = 1756.16
100

(C − 18) = (x + 6)
1 ∴ 1756.16 → 43904 Rs.
2 43904
x 1→ = 25
C = + 21 1756.16
2 ∴ Required amount = 1000 × 25 = 25000 Rs.
(A+B+C)
Given = 42
3 46. (b); x × 0.38 + (1625 − x) × 0.60 = 766
x
2x + x + + 21 = 42 × 3 x = 950
2
4x+2x+x 1625 − x = 675
= 105
2 required difference = 950 − 675 = 275
x = 15 × 2 = 30 years 144
Required age = 2x + 9 = 30 × 2 + 9 = 69 years 47. (d); Required percentage = × 100 ≈ 73%
198

14 Adda247 Publications For any detail, mail us at


Publications@adda247.com
50+ Bank PO | Clerk Previous Year’s Papers 2016 – 2020

48. (b); book distributed by company B 67. (a); A : B = 5 : 3 = 10 : 6


32 B:C=2:3=6:9
= 2 × [450 × ] − 2 = 286
100
100 A : B : C = 10 : 6 : 9
books published by company B = 286 × = 650 Ratio for profit
44
= (10x × 12) ∶ (6x × 12): (9x × 6) = 20 ∶ 12 ∶ 9
49. (d); Since data is not sufficient to calculate the required 12−9
value. Required difference = = 12300 = 900 Rs.
41
45 24 68. (b); let t = days taken by second group
50. (d); 260 × + 250 × = 177
100 100
2 × 30 × 4 × 10 = 45 × 8 × t
51. (e); 2
t = 6 days.
3

69. (c); Let the borrowed sum is S


∴ 70 + 34 ⇒ 104 not 108 S×7×3 S×9×6 S×10×6
+ + = 4050
100 100 100
52. (c); × 0.5 + 1,× 1 + 1,× 1.5 + 1,× 2 + 1,× 2.5 + 1. . . . . . . .. 0.21S + 0.54S + 0.6S = 4050
∴ 5 × 1 + 1 = 6 not 7 1.35S = 4050
4050
53. (d); × 1 + 1,× 2 + 1,× 3 + 3,× 4 + 1,× 5 + 1. . . . . . . . . . .. S= = 3000 Rs.
1.35
∴ 5 × 1 + 1 = 6 not 8
70. (c); Average of 100 students = 40
54. (b); +23 , +43 , +63 , +83 , +103 . . . . . . . . … Total = 40 × 100 = 4000
∴ 294 + 512 = 806 not 842 Error = 83 – 53 = 30 (high)
3970
55. (a); ×1, ×2, ×4, ×8, ×16 So correct average⇒ = 39.7
100
∴ 3 × 1 = 3 not 4
71. (a);
5 12
56. (d); Girls in Swimming = × × 1800 = 135
8 100
3 25
Girls in tennis = × × 1800 = 270
5 100
135
Required percentage = × 100 = 50%
270
2 13
57. (a); No. of boys in cricket = × × 1800 = 156
3 100
7 30
No. of boys in Hockey = × × 1800 = 378
10 100
156 + 378 = 534
200 200
58. (d); Required ratio = = = 40 ∶ 81
270+135 405
105
59. (c); Required percentage ∴ radius of cone = m
160+162+270+135+78 2
= × 100 ≈ 45% Slant height of cone = 63 m
1800
⇒ curved surface area of cone = (πrl)
60. (e); Required difference = 200 − 81 = 119 22 105
= × × 63 = 10395 m2
245 7 2
61. (c); Speed on Monday = = 70 km/hr radius of cylinder = m
105
3.5
2
Speed on Tuesday = (75 × 2) − 70 = 80 km/hr height = 3 m (given)
Distance travelled on Tuesday = 80 × 3.5 = 280 km ∴ curved surface are of cylinder = 2πrh
180 22 105
= 2× × × 3 = 990 m2
62. (a); Required average = 2.5
405 =4∶5 7 2
4.5 Total curved area of structure
57 ⇒ Curved area of cone + curved area of
63. (b); Required percentage = × 100 ≈ 54%
105 cylinder = 10395 + 990
54 = 11385 m2
64. (b); Required percentage = × 100 ≈ 64%
84 ∴ Total area of canvas = 11385 m2
65. (c); Distance travelled on Tuesday 72. (a); Let total unit = 50
10 1 x
= 405 × = 450 km 20 × + 30 × = 9.5
9 4 100
450 3x
Speed = = 112.5 km/hr = 4.5 ⇒ x = 15%
4 10
66. (a); C. P of mobile = x 73. (d); Let the sum be Rs. x.
C. P of laptop = 78000−x 5 4 x×10×2
25 15 5 Then, [x (1 + ) − x] − [ ] = 124.05
x− (78000 − x) = × 78000 ⇒ x = 39000 100 100
100 100 100
Solving the above equation, we get x = Rs. 8000.

15 Adda247 Publications For any detail, mail us at


Publications@adda247.com
50+ Bank PO | Clerk Previous Year’s Papers 2016 – 2020

74. (d); 78. (d); (i) 3x² - 13x + 12 = 0


3x² - 9x - 4x + 12 = 0
4
x = ,3
3
(ii) 3y² - 7y + 2 = 0
3y² - 6y - y + 2 = 0
Required Ratio = 1 : 4 1
y = ,2
3
x
75. (e); Let original fraction = ∴ No relation
y
120x 3 40x x 5
= ⇒ =1 ⇒ =
125y 5 25y y 8 79. (b); (i) 20x² - 9x + 1 = 0
76. (e); (i) 3x² + 29x + 56 = 0 20x² - 4x - 5x + 1 = 0
3x² + 20x + 9x + 56 = 0 x= ,
1 1
−8 4 5
x = , −7
3
(ii) 12y² - 7y + 1 = 0
(ii) 2y² + 15y + 25 = 0
2y² + 10y + 5y + 25 = 0 12y² - 3y - 4y + 1 = 0
−5 1 1
y = , −5 y= ,
2 3 4
∴ No relation ∴x≤y
77. (e); (i) 4x² - 29x + 45 = 0
4x² - 20x - 9x + 45 = 0 80. (e); (i) x=±4
9
x = ,5 (ii) 2y² - 17y + 36 = 0
4
(ii) 3y² - 19y + 28 = 0 2y² - 9y - 8y + 36 = 0
3y² - 12y - 7y + 28 = 0 9
7
y = ,4 y = ,4
2
3
∴ No relation ∴x≤y

ENGLISH LANGUAGE

81. (e); Read the first paragraph carefully, “This is the first which is set to take place within a year. This will make
time a satellite weighing over 3.1 tonnes has been India self-reliant in launching heavier satellites,
launched from India to reach the geostationary orbit bringing down costs substantially.” Hence all three
about 36,000 km from Earth. The Mk-III can launch statements justify the author’s view regarding the
satellites weighing up to four tonnes, which almost development of ISRO.
doubles India’s current launch capacity.” and “On
84. (e); All the given statements are true in context of the
Monday, an indigenously developed lithium-ion
passage.
battery was used for the first time to power the
satellite.” Hence all three given features make the 85. (d); Refer the last paragraph, “With fewer propulsion
newly launched vehicle GSLV Mk-III unique. stages and, therefore, control systems, the Mk-III
…………………………….the Mk-III can carry three
82. (a); Refer the first paragraph, “This can be done by
astronauts and have more space to carry out
switching over to electric propulsion for orbit rising
experiments.” Hence only statements (I), (II) and (III)
and to keep the satellite in the right position and
are correct in context of the passage.
orientation in the orbit through its lifetime (that is,
station keeping). The switch-over would reduce the 86. (b); Propulsion means the action of driving or pushing
weight of the vehicle as it can do away with nearly forwards. Hence ‘impulse’ is the word most similar
two tonnes of propellants and carry heavier in meaning to it.
satellites.” Hence (a) is the correct option in context Satiety means the feeling or state of being sated.
of the passage. Revulsion means a sense of disgust and loathing.
83. (e); Read the passage carefully. Refer “It can now be said 87. (d); Crew means a group of people who work on and
without hesitation that India belongs to the elite club operate a ship, aircraft etc. Bevy means a large group
of countries that have mastered cryogenic of people or things of a particular kind. Hence both
technology.” and, “The Mark-III will be operational are similar in meanings.
with the success of one more developmental flight,

16 Adda247 Publications For any detail, mail us at


Publications@adda247.com
50+ Bank PO | Clerk Previous Year’s Papers 2016 – 2020

Oodles mean a very great number or amount of 101. (d); “improving, prevent” is the correct set of words that
something. makes the sentence meaningful.
Laity means ordinary people, as distinct from Overhauling means take apart (a piece of machinery
professionals or experts. or equipment) in order to examine it and repair it if
88. (a); Propellant means a substance used as a reagent in a necessary.
rocket engine to provide thrust. Mulligan means Circumventing means avoid.
something that blows up. Hence both are similar in Retrogressing means go back to an earlier state.
meanings. Deter means go back to an earlier state.
Motif means a dominant or recurring idea in an
artistic work. 102. (d); “affected, help” is the correct set of words that
Ardour means great enthusiasm or passion. makes the sentence meaningful.
Perturbed means anxious.
89. (c); Elite means a select group that is superior in terms of
Pamper means indulge with every attention,
ability or qualities to the rest of a group or society.
comfort, and kindness
Dregs mean the most worthless part or parts of
something. Hence both are opposite in meanings. 103. (b); “secession, received” is the correct set of words that
Patriciate means a noble order or class. makes the sentence meani` `ngful.
Gentry mean people of good social position, Secession means the action of withdrawing formally
specifically the class of people next below the nobility from membership of a federation or body, especially
in position and birth. a political state.
90. (e); Operational means in or ready for use. Severed Repudiation means rejection of a proposal or idea.
means put an end to (a connection or relationship); Disbursed means pay out (money from a fund).
break off. Hence both are opposite in meanings.
104. (c); “witnessed, reviving” is the correct set of words that
Expedient means (of an action) convenient and
makes the sentence meaningful.
practical although possibly improper or immoral.
Dissuading means persuade (someone) not to take a
91. (e); Refer the first and second paragraph of the second particular course of action.
paragraph “In earlier times when man appeared on Professed means of a quality, feeling, or belief)
earth he was overawed at the sight of violence of claimed openly but often falsely.
powerful aspects of nature. In certain cases,
usefulness of different natural objects of nature 105. (e); “consecutive, appear” is the correct set of words
overwhelmed man.” that makes the sentence meaningful.
Pertinent means relevant or applicable to a
92. (d); Refer the fifth last sentence of the passage “It was in
particular matter, apposite.
these trying circumstances that the science emerged
Coherent means logical and consistent.
as a saviour of mankind.”
106. (b); The sequence after rearrangement of the sentences is
93. (d); Refer the second and third last sentences of the first
paragraph “Science takes its recourse to progressive DAFBEC
march towards perfection. The tools of religion are 107. (c); The sequence after rearrangement of the sentences is
faith, institution and spoken word of the DAFBEC
enlightened.”
108. (a); The sequence after rearrangement of the sentences is
94. (d); Sentence (d) is not correct as it has not been
DAFBEC
mentioned in the passage.
109. (e); The sequence after rearrangement of the sentences is
95. (e); Refer the last paragraph of the passage.
DAFBEC
96. (c); Use ‘are’ in place of ‘is’ because the subject of the verb
‘the youths’ is plural. 110. (b); The sequence after rearrangement of the sentences is
DAFBEC
97. (e); The sentence is grammatically correct.
111. (d); ‘absence’ is the correct word.
98. (b); Use ‘price’ in place of ‘prices’ as ‘onion’ is a thing, for Desertion means abandonment, leaving.
which ‘price’ in singular form is used. Paucity means the presence of something in only
99. (e); The sentence is grammatically correct. small or insufficient quantities or amounts.
Sparsity means to scatter or spread.
100. (d); The use of ‘being’ is superfluous.

17 Adda247 Publications For any detail, mail us at


Publications@adda247.com
50+ Bank PO | Clerk Previous Year’s Papers 2016 – 2020

112. (e); ‘embedded’ is the correct word to be filled in the Rapt means completely fascinated or absorbed by
blank. what one is seeing or hearing.
According to the theme of the paragraph, ‘embedded’ 116. (a); ‘feed’ is the correct word to be filled in the blank.
is the correct word. Organic wastes are useful for the use of small and
affordable households.
113. (b); ‘forms’ is the correct word.
117. (e); ‘use’ is the correct word.
114. (c); ‘generation’ is the correct word.
As production is used in the sentence, hence we can 118. (c); “prevent’’ is the correct word.
get the idea that generation will be the answer. 119. (d); ‘ingested’ is the correct word to be filled in the blank.
115. (c); ‘lost’ is the correct word. Ravaged means severely damaged.
Awed means filled with awe or wonder. 120. (b); ‘refuse’ is the correct word.
Misled means cause (someone) to have a wrong idea As refuse is also used elsewhere in the paragraph,
or impression. we can get a hint.

18 Adda247 Publications For any detail, mail us at


Publications@adda247.com

You might also like